282
1 Números y formas Edición 2018 Colección Hojamat.es © Antonio Roldán Martínez http://www.hojamat.es

Números y formas - Aprender y divertirse con hoja de …hojamat.es/publicaciones/numform.pdf · Este es uno de los capítulos de las Matemáticas en los que se aprecia ... también

Embed Size (px)

Citation preview

Page 1: Números y formas - Aprender y divertirse con hoja de …hojamat.es/publicaciones/numform.pdf · Este es uno de los capítulos de las Matemáticas en los que se aprecia ... también

1

Números y formas

Edición 2018

Colección Hojamat.es

© Antonio Roldán Martínez

http://www.hojamat.es

Page 2: Números y formas - Aprender y divertirse con hoja de …hojamat.es/publicaciones/numform.pdf · Este es uno de los capítulos de las Matemáticas en los que se aprecia ... también

2

PRESENTACIÓN

Desde los pitagóricos hemos usado figuras poligonales para

representar ciertos números, pero lo que comenzó siendo una nueva

forma de percibirlos se convirtió pronto en un manantial de nuevas

cuestiones. Puedes pensar en ellos sin tener en cuenta su origen

geométrico. De hecho, muchas cuestiones actuales y futuras que se

traten en nuestro blog se basarán más en sus definiciones aritméticas.

Este es uno de los capítulos de las Matemáticas en los que se aprecia

mejor el pensamiento que no busca la utilidad, sino la belleza ¿Para

qué sirve un número pentagonal? Entre todas las clases que

estudiaremos, los números cuadrados son los que permiten más

cuestiones y actividades. En esta edición les hemos dedicado, al igual

que a los triangulares, un capítulo aparte.

También hemos comenzado a incorporar nuevos tipos de números

figurados. En esta edición se han añadido los piramidales de cuatro

dimensiones y los centrados, tanto poligonales como piramidales.

Como advertiremos en todos los documentos de esta colección, el

material presentado no contiene desarrollos sistemáticos, ni pretende

ser un manual teórico. En cada tema se incluirán cuestiones curiosas o

relacionadas con las hojas de cálculo, con la única pretensión de

explicar algunos conceptos de forma amena.

Page 3: Números y formas - Aprender y divertirse con hoja de …hojamat.es/publicaciones/numform.pdf · Este es uno de los capítulos de las Matemáticas en los que se aprecia ... también

3

TABLA DE CONTENIDO

Presentación ..................................................................................................2

Cuadrados .....................................................................................................5

Eliminar bolas en un cuadrado ...................................................................5

Curiosidades bien fundamentadas .............................................................9

Sumas de los primeros cuadrados .......................................................... 11

Cuadrados en progresión aritmética ....................................................... 12

La mitad, cuadrado, el tercio, cubo ......................................................... 15

Una exploración matemática ................................................................... 16

El número 30500 ..................................................................................... 18

Piezas para cuadrados ............................................................................ 20

Carnaval de cuadrados ............................................................................ 25

Resultados curiosos de la suma de divisores cuadrados ....................... 29

Igualdad de sumas de cuadrados con un escalón .................................. 35

Cuadrados del tipo n(n+k) ....................................................................... 39

Triangulares ................................................................................................ 58

Jugamos con los triangulares .................................................................. 58

Triangulares alojados .............................................................................. 59

Carnaval de triangulares .......................................................................... 63

Triangulares de lado par .......................................................................... 69

Cuadrados y triangulares .......................................................................... 73

Cuadrados vecinos de triangulares ......................................................... 73

Triangulares y cuadrados con piezas ...................................................... 79

Números especiales que son un producto especial ................................ 84

Equilibrados entre semejantes .............................................................. 105

Damos vueltas a los triangulares cuadrados ......................................... 109

Poligonales ............................................................................................... 125

¿Eres un poligonal? ............................................................................... 125

Cubos y gnomones ................................................................................ 130

Page 4: Números y formas - Aprender y divertirse con hoja de …hojamat.es/publicaciones/numform.pdf · Este es uno de los capítulos de las Matemáticas en los que se aprecia ... también

4

Poligonales centrados ........................................................................... 135

Números piramidales............................................................................... 149

Poligonales y piramidales ..................................................................... 149

Tetraedros.............................................................................................. 155

Cuadrados ............................................................................................. 161

Pentágonos ............................................................................................ 168

Hexágonos y octógonos ........................................................................ 173

Números piramidales de cuatro dimensiones ....................................... 177

Números piramidales de cuatro dimensiones ....................................... 183

Pirámides cuadrangulares en cuatro dimensiones ............................... 189

Otros números piramidales de cuatro dimensiones .............................. 193

Números piramidales centrados ............................................................ 203

Pitágoras y sus ternas ............................................................................. 232

El sueño de Lewis Carroll ...................................................................... 232

Oblongos y pitagóricos .......................................................................... 233

Viaje de ida y vuelta a la Geometría ...................................................... 239

Centrados y pitagóricos ......................................................................... 242

Cuadrados con trozos consecutivos ...................................................... 244

Diferencia entre catetos ......................................................................... 245

Doblado pitagórico ................................................................................. 247

¿En cuántas sumas de cuadrados? ...................................................... 249

Espiral de números ................................................................................ 258

Números especiales ................................................................................ 262

Autonúmeros o números colombianos .................................................. 262

Soluciones ................................................................................................ 274

Cuadrados ............................................................................................. 274

Pitágoras y sus ternas ........................................................................... 279

Page 5: Números y formas - Aprender y divertirse con hoja de …hojamat.es/publicaciones/numform.pdf · Este es uno de los capítulos de las Matemáticas en los que se aprecia ... también

5

CUADRADOS

La teoría de los números figurados es de las más antiguas de la

Historia de las Matemáticas. En cualquier página web o manual de

carácter general se puede consultar lo más importante.

En Hojamat.es se pueden estudiar los documentos

http://hojamat.es/sindecimales/aritmetica/teoria/teorarit.htm

y

http://hojamat.es/parra/prop2011.pdf

Entre todos ellos, los cuadrados y triangulares son los más estudiados.

ELIMINAR BOLAS EN UN CUADRADO

Forma un cuadrado con bolas, situándolas en filas y columnas, las que

quieras. Después elimina 10 bolas e intenta reorganizar el resto hasta

formar otro cuadrado más pequeño, y verás que

resulta imposible, cualquiera que sea el lado del

cuadrado que has formado.

Prueba entonces a quitar sólo 6 bolas, y

observarás que tampoco puedes formar un

cuadrado con las restantes. Con otros números

sí se puede, dependiendo del lado del cuadrado.

¿Qué tienen de particular el 6 y el 10 para que ocurra esto?

Si 10 es la diferencia entre dos cuadrados n2 y m2, se tendrá que

verificar que

10=(m+n)(m-n)

Page 6: Números y formas - Aprender y divertirse con hoja de …hojamat.es/publicaciones/numform.pdf · Este es uno de los capítulos de las Matemáticas en los que se aprecia ... también

6

Pero m+n y m-n tienen la misma paridad. Esto obliga a que el número

10 también se descomponga en dos factores, ambos pares o ambos

impares, pero eso es imposible, debido a que sólo se pueden

considerar 10*1 y 5*2, que no tienen la misma paridad. Lo mismo

ocurre con el 6.

En general, todos los números de la forma 2(2k+1) con k entero, no

serán idóneos para ser diferencia de cuadrados. Expresado de otra

forma, son números múltiplos de 2 pero no de 4.

Todos los números naturales podemos escribirlos como

N=2m.ap1.bp2…zpn, siendo a,b..z primos y m eventualmente cero. El

exponente m nos permite distinguir tres casos:

(a) m=1

El número N no se puede descomponer en dos factores ambos pares

ni ambos impares. Es el caso de 2,6,10…que acabamos de estudiar

(b) m=0

El número es impar, y presentará tantas soluciones como la mitad del

su número de divisores. Por ejemplo 105= 532-522 = 192-162 = 132-82 =

112-42 que corresponden a los pares de divisores 105=

105*1=35*3=21*5=15*7. Si el número es primo admitirá una sola

diferencia de cuadrados.

(c) m>1

2m se puede descomponer en dos factores pares de tantas maneras

como indique la parte entera de (N-1)/2. Así, si m=3 sólo se admite una

descomposición, como 8 = 32-12 y si m=7 128= 332-312 = 332-312 =

182-142 . Combinando estos casos con los producidos por los factores

impares obtendremos todas las soluciones.

Se ve, pues que el número de soluciones está relacionado de cierta

forma con el de divisores, aunque pueden discrepar bastante.

Otra idea: Ser diferencia de cuadrados equivale a poder construir un

gnomon con sus unidades. En las siguientes imágenes podemos ver

uno con a par y otro con a impar.

Page 7: Números y formas - Aprender y divertirse con hoja de …hojamat.es/publicaciones/numform.pdf · Este es uno de los capítulos de las Matemáticas en los que se aprecia ... también

7

Podemos traducir esta figura mediante una propiedad: Los números

que admiten una expresión como diferemcia de cuadrados

también se pueden representar como una suma de impares

consecutivos.

Número de descomposiciones

Entre los números menores que 1000 hay uno que es igual a diez

diferencias distintas de cuadrados ¿Cuál?

Para encontrarlo acudiremos a la hoja de cálculo:

El número menor que 1000 que se puede representar como diez

diferencias de cuadrados es el 960.

En la siguiente tabla se han inluido todas las formas de descomponer

960 en dos factores. Después se han seleccionado los de la misma

paridad, y mediante suma y diferencia se han obtenido A y B tales que

960 = A2 – B2

Pero, ¿cómo encontramos el 960? Aparte de un análisis bastante

pesado de los factores de los números menores que 1000 podemos

usar este código en Basic:

Factores de 960 Paridad A B Comprobación

2 480 0 241 239 960

3 320 1

4 240 0 122 118 960

5 192 1

6 160 0 83 77 960

8 120 0 64 56 960

10 96 0 53 43 960

12 80 0 46 34 960

15 64 1

16 60 0 38 22 960

20 48 0 34 14 960

24 40 0 32 8 960

30 32 0 31 1 960

Igual paridad 10

Page 8: Números y formas - Aprender y divertirse con hoja de …hojamat.es/publicaciones/numform.pdf · Este es uno de los capítulos de las Matemáticas en los que se aprecia ... también

8

Sub numerodiferencias Dim i,j dim a for i=1 to 1000 (Números a probar) a=0 (Contador de factores de la misma paridad) for j=1 to sqr(i) (Se buscan los factores hasta la raíz cuadrada) if esdivisible(i,j) then (Comprueba si es factor) if paridad(j)=paridad(i/j) then a=a+1 (Igual paridad, se incrementa a) end if next j msgbox(i) msgbox(a) next i End Sub Con una macro definida por este código obtenemos una tabla con el

número de descomposiciones de igual paridad similar a la de la imagen

Así se ha encontrado que 960 admite 10 descomposiciones, y que los

múltiplos de 2 que no lo son de 4, ninguna.

Los códigos que necesitarás también son:

public function esdivisible(a,b) as boolean if int(a/b)=a/b then esdivisible=True else esdivisible=False end function public function paridad(n) paridad=n-int(n/2)*2 end function

32 2

33 2

34 0

35 2

36 2

37 1

38 0

39 2

40 2

41 1

42 0

Page 9: Números y formas - Aprender y divertirse con hoja de …hojamat.es/publicaciones/numform.pdf · Este es uno de los capítulos de las Matemáticas en los que se aprecia ... también

9

El tema da todavía más de sí:

En el caso de varias soluciones podemos relacionarlas con los

cuadriláteros con lados de medida entera inscriptibles en una

circunferencia. Así, los lados 49, 47, 26 y 22 forman uno de esos

cuadrados.

En efecto, si dos diferencias son iguales, como 492

– 472 = 262 – 222, también se verificará que 492 +

222= 262 + 472, que se puede interpretar como dos

triángulos rectángulos que comparten un mismo

diámetro por lo que formarán un cuadrilátero

inscriptible.

CURIOSIDADES BIEN FUNDAMENTADAS

Todos los que publicamos sobre números naturales incluimos en algún

momento curiosidades aritméticas. A veces no nos damos cuenta de

tres hechos que influyen en la aparición de las mismas restando un

poco de su importancia matemática.

(A) Algunas son meras coincidencias sin valor matemático alguno,

como 2592 =2592

(B) Otras dependen del sistema de numeración empleado, como todas

las que usan los conceptos de capicúa o de cifras invertidas. Por

ejemplo 122=144 y 212 = 441

(C) En algunos casos no existe tal curiosidad numérica, sino que es el

reflejo de una propiedad algebraica expresada de forma que se oculte

su origen.

El otro día, releyendo “Los números mágicos del Dr. Matrix” de Martin

Gardner, recordé la clave algebraica de esta serie de curiosidades:

32+42 = 52

102+112+122 = 132+142

Page 10: Números y formas - Aprender y divertirse con hoja de …hojamat.es/publicaciones/numform.pdf · Este es uno de los capítulos de las Matemáticas en los que se aprecia ... también

10

212+222+232+242 = 252+262+272

Según el autor, basta tomar como primer sumando el cuadrado de

n(2n+1), siendo n el número de términos del segundo miembro. Por

tanto, la siguiente igualdad comenzará con el cuadrado de 4(2*4+1) =

36: 362+… + 402 = …

¿Sabrías demostrarlo algebraicamente sin trabajar demasiado? Busca

algún atajo, si no, mejor lo dejas, que directamente puede resultar muy

largo. Eso sí, practicarás el Álgebra hasta hartarte de ella. Quizás

debas esperar antes de seguir leyendo.

¿Cómo demostrar esta propiedad?

32+42 = 52

102+112+122 = 132+142

212+222+232+242 = 252+262+272

Es bueno que distingamos entre comprobar y demostrar.

(1) En el siguiente documento del Club Mensa

http://www.mensa.es/carrollia/c63.pdf

puedes estudiar una demostración que requiere mucho cálculo

algebraico, pero que al final llega, por demostración, a que el primer

cuadrado debe ser el del número n(2n+1).

(2) Si recuerdas que la suma de los n primeros números cuadrados es

igual n(n+1)(2n+1)/6, puedes sustituir cada miembro de la igualdad

como una diferencia entre este tipo de sumas. Luego, hay que

desarrollar paréntesis y cuadrados hasta llegar a una misma expresión

algebraica en ambos. Tomamos como primer cuadrado el de n(2n+1),

es decir, 2n2+2n

La calculadora online Wiris nos puede ayudar.

Page 11: Números y formas - Aprender y divertirse con hoja de …hojamat.es/publicaciones/numform.pdf · Este es uno de los capítulos de las Matemáticas en los que se aprecia ... también

11

Observa la igualdad de resultados: 24n5+60n4+50n3+15n2+n.

(3) Lo anterior no es una demostración, sino una comprobación de que

el inicio en n(2n+1) es correcto. Para demostrarlo podemos seguir

basándonos en S= n(n+1)(2n+1)/6.

Si usamos como variable N el número anterior a una suma de este

tipo y llamamos K al número de sumandos, se puede demostrar que

(N+1)2+(N+2) 2+…+(N+K) 2 = 2K3+(6N+3)K2+(6N2+6N+1)K.

Si aplicamos esta fórmula por una parte a N y K (primer miembro) y

por otra a N+K y K-1 (segundo miembro), al igualarlas, y simplificar

mucho (¡mucho!), llegamos a una ecuación de segundo grado de

soluciones enteras, que nos exige que N=K(2K-3), que es equivalente

al de n(2n+1) con un cambio de variables.

Todo lo propuesto es muy costoso de desarrollar, pero te queda la

satisfacción de no tener que creértelo sólo porque esté escrito en un

blog.

SUMAS DE LOS PRIMEROS CUADRADOS

Estudiando un tema determinado me he encontrado con esta relación

que no conocía:

12+22+32+42+52+…+232+242 = 702

No sé si estará publicada ya, pero la presento aquí por su elegancia y

por mi sospecha de que no existen casos similares. He buscado hasta

50000 y no he encontrado otro cuadrado que sea suma de los K

primeros cuadrados.

¿Ocurrirá algo parecido con los números triangulares?

1+3+6+10+15…+N(N+1)/2 = K(K+1)/2

Page 12: Números y formas - Aprender y divertirse con hoja de …hojamat.es/publicaciones/numform.pdf · Este es uno de los capítulos de las Matemáticas en los que se aprecia ... también

12

La respuesta es afirmativa

Hemos descubierto cuatro casos entre 1 y 100000, sin contar el trivial

1=1, en los que la suma de los primeros triangulares produce otro

triangular.

El primero es 1+3+6 = 10

¿Cuáles son los otros tres?

Ya puestos a calcular, nos hemos planteado si sumando los primeros

números triangulares podremos obtener un cuadrado, o, a la inversa, si

sumando los primeros cuadrados la suma será un número triangular.

(Ver Soluciones)

CUADRADOS EN PROGRESIÓN ARITMÉTICA

No es difícil encontrar ternas de cuadrados perfectos que estén en

progresión aritmética, tales como 1, 25 y 49, o 4, 100 y 196. ¿Cómo

podríamos encontrar más ternas con una hoja de cálculo? Se podría

organizar una tabla de doble entrada con los cuadrados perfectos, y

después someter a su media aritmética a una condición ¿Cuál?

Page 13: Números y formas - Aprender y divertirse con hoja de …hojamat.es/publicaciones/numform.pdf · Este es uno de los capítulos de las Matemáticas en los que se aprecia ... también

13

En la imagen puedes ver el resultado de una búsqueda similar, en la

que se han marcado con un 1 los cuadrados perfectos pertenecientes

a una terna como la propuesta. Si te animas a construir un buscador

semejante podrás encontrar muchas más ternas. Ponte a prueba:

¿Con qué otros dos cuadrados forma progresión aritmética el número

10404, cuadrado de 102?

Para concretar las ternas pedidas hemos recurrido a una exploración

sistemática. Es una forma válida de trabajar en Matemáticas (así se

encuentran los números primos), pero que alguien puede pensar que

es algo perezosa. A continuación aportamos un análisis algo más

profundo.

Enfoque algebraico

Llamemos n a la raíz cuadrada del término central, que sería n2.

El tercer cuadrado tendrá la forma (n+h)2 y el primer cuadrado (n-k)2

con k>h ¿por qué?

Las diferencias entre ellos serán iguales, luego

(n+h)2 - n2= n2 - (n-k)2

Simplificando: 2nh+ h2 = 2nk - k2

Despejando n: n = (h2 + k2) / (2(k-h))

Como k>h, llamamos m=k-h, y entonces queda:

n = (h2 + (h+m)2) / (2m) = (2 h2 + 2mh + m2) / (2m)

Esto obliga a que m sea par, y la podemos sustituir por 2p

n = (2 h2 + 4ph + 4p2) / (4p) = h2/(2p) + h + p (1)

Esto nos da un procedimiento de generación de ternas de cuadrados:

Elegimos cualquier entero p y buscamos un número par h cuyo

cuadrado sea divisible entre p y cuyo cociente sea mayor que el mismo

p (para que n-k sea positivo), y mediante la fórmula (1) calculamos n.

Seguidamente encontramos los valores de n+h y n-k = n-h-2p

Page 14: Números y formas - Aprender y divertirse con hoja de …hojamat.es/publicaciones/numform.pdf · Este es uno de los capítulos de las Matemáticas en los que se aprecia ... también

14

Ejemplo: p=5, h=10, n=100/10 + 10 + 5 = 25; (n+h)=35: (n-k)=25-10-

5*2=5.

Por tanto, los cuadrados en progresión aritmética buscados son: 25,

625 y 1225.

Notas

- Si tres cuadrados están en progresión aritmética, sus diferencias

mutuas son siempre múltiplos de 24. Intenta demostrarlo, que es un

reto muy interesante. Si no lo logras, en las Soluciones tienes una

demostración basada en los restos cuadráticos.

- No existen cuatro cuadrados en progresión aritmética, ni en mayor

número.

- Tampoco existen tres cubos en progresión aritmética.

- Leonard Dickson en su libro "History of the Theory of Numbers"

(1919), propone estas fórmulas:

x = 2 v2/u – u

y = 2 v2/u + u + 2 v

z = 2 v2/u + u + 4 v

donde u divide a 2 v2 con un cociente mayor que u.

Este método coincide con el que se propone en este libro. Por ejemplo,

para h=12 y p=6 las soluciones son n=30, n+h=42, n-k=6, y con la

propuesta de Dickson se logra la misma solución con 2v2=72 y u=6:

x=72/6-6=6; y=72/6+6+12=30; z=72/6+6+24=42.

- No es difícil crear un código de búsqueda, si dispones de la función

ESCUAD, para saber si un número es cuadrado:

Se supone que se buscan soluciones hasta un número N

For i = 2 To N

a = i * i

For k = 1 To a / 2

If escuad(a - k) And escuad(a - 2 * k) Then

Page 15: Números y formas - Aprender y divertirse con hoja de …hojamat.es/publicaciones/numform.pdf · Este es uno de los capítulos de las Matemáticas en los que se aprecia ... también

15

Msgbox(a): Msgbox(a-k): Msgbox(a-2*k)

End If

Next k

Next i

En el lenguaje PARI se puede usar este código:

isinteger(n)=(n==truncate(n))

isquare(n)= { local(f,m,p=0); if(n==1,p=1,f=factor(n); m=gcd(f[, 2]);

if(isinteger(m/2),p=1));return(p) }

{ for (n=2, 100, a=n*n;for(k=1,a/2,if (isquare(a-k) && isquare(a-2*k),

write("final.txt",a," ",a-k," ",a-2*k)))) }

LA MITAD, CUADRADO, EL TERCIO, CUBO

Encuentra los primeros números naturales N que admiten estas dos

descomposiciones:

N= 2n2 = 3m3

Es necesario recorrer los posibles factores primos de m y de n y sus

exponentes.

Una solución es N=41472, pero existe otra menor.

Si has aprendido a hacerlo, prueba con

N= 2n2 = 5m5

Una solución es un número “muy redondo”

Si tu ánimo no ha sufrido merma, aborda el que

N= 3n3 = 5m5

Quizás la primera solución tenga nueve cifras

Page 16: Números y formas - Aprender y divertirse con hoja de …hojamat.es/publicaciones/numform.pdf · Este es uno de los capítulos de las Matemáticas en los que se aprecia ... también

16

UNA EXPLORACIÓN MATEMÁTICA

En la entrada número 120 del interesante blog de Claudio

(http://simlementenumeros.blogspot.com) se hacía una propuesta que

esencialmente consistía en buscar los números que son cuadrados

perfectos y que su doble aumentado en una unidad también lo es,

como 144=122 y 144*2+1=289 = 172. En un primer comentario, se

proponían las soluciones 02, 22, 122, 702, 4082 y 23782 con una ley de

formación an+1 = 6an – an-1

Una entrada posterior contenía un enlace a una página de sucesiones

de números enteros muy popular.

Navegando un poco y siguiendo enlaces sucesivos al propuesto

descubrí que las soluciones 0, 4, 144, 4900, 166464,… divididas entre

4 coincidían con los números enteros que son cuadrados y triangulares

a la vez (se puede prescindir del 0): 1, 36, 1225, 41616,…

Recordé que estos números se encuentran mediante la ecuación de

Pell 8x2+1=y2, una de cuyas soluciones es x=1, y=3. De esta forma se

aclaró bastante la cuestión, y por su posible interés, se desarrolla a

continuación el proceso que seguí, mediante una serie de propuestas

encadenadas complementarias a las de Claudio.

(1) Demuestra que si 2n2 + 1 = m2, entonces n2/4 es también cuadrado

y triangular.

(2) Demuestra que los números x que son cuadrados y triangulares a

la vez coinciden con los valores de x2 que son soluciones de la

ecuación de Pell 8x2+1=y2

(3) Una de las soluciones de la ecuación citada es x1=1, y1=3. Según la

teoría correspondiente a las ecuaciones de Pell, las demás soluciones

de esta ecuación vienen dadas por la igualdad

Page 17: Números y formas - Aprender y divertirse con hoja de …hojamat.es/publicaciones/numform.pdf · Este es uno de los capítulos de las Matemáticas en los que se aprecia ... también

17

Usa esta propiedad para encontrar las soluciones de x, que serán 1, 6,

35, 204,…., que elevadas al cuadrado coincidirán con los números

cuadrados y triangulares a la vez 1, 36, 1225, 41616,…que, a su vez,

multiplicados por 4, resultarán ser las soluciones de 2n2 + 1 = m2, 4,

144, 4900, 166464…

(4) Usa la fórmula del apartado anterior para demostrar esta fórmula

doble de recurrencia:

yn=8xn-1+3yn-1, xn=3xn-1+yn-1

o, en forma matricial:

Su aplicación reiterada nos permitirá encontrar los valores (1,3), (6,17),

(35,99)…

Convierte esas dos fórmulas de recurrencia en una sola para xn, y te

resultará

xn=6xn-1-xn-2, que coincide con la propuesta en el blog para sus dobles

0, 2, 12, 72,…

(5) ¿Por qué el cociente entre las x parece tender a esta expresión?

Nos podemos basar en que qn = 6-1/qn-1, que nos lleva, tomando

límites para n tendiendo a infinito, a la ecuación x = 6 -1/x, una de

cuyas soluciones es

que es la que vale al ser creciente la sucesión.

Page 18: Números y formas - Aprender y divertirse con hoja de …hojamat.es/publicaciones/numform.pdf · Este es uno de los capítulos de las Matemáticas en los que se aprecia ... también

18

EL NÚMERO 30500

Mantener un blog de números con cierta periodicidad supone un gran

esfuerzo en la búsqueda de temas y su posterior desarrollo. Por eso,

suelo usar “disparadores de ideas”: páginas web, libros o revistas que

tratan un tema matemático y que en sus desarrollos se incluyen

fórmulas o propiedades que me sugieren (¡ajá!) un desarrollo sobre

ellas.

Una de las páginas que visito con frecuencia es la popular The On-Line

Encyclopedia of Integer Sequences, de dirección http://oeis.org/

La otra tarde escribí en ella el número 30500 (eso recordaba yo, pero

no fue así) y descubrí que era suma de los cuadrados de cuatro

números primos consecutivos. Así que me interesé por el tema, y como

hacía tiempo que no le daba vueltas a una cuestión, lo elegí con ese fin

y así lo presento:

(1) La serie de números que son suma de cuadrados de primos, salvo

el primero, que contiene el cuadrado de 2, todos son pares, como era

de esperar, pero ¿existe alguna terminación en 0, 2, 4, 6 u 8 que no

puedan presentar nunca? Intenta justificarlo. Es una cuestión bastante

sencilla.

(2) Más sencilla todavía: Salvo el primero, todos son múltiplos de 4 (es

sencilla pero no trivial, podían ser pares no múltiplos de 4)

(3) Cuando volví a la página de secuencias enteras y probé 30500 me

di cuenta de que estaba recordando mal. Ese número no presenta la

propiedad deseada. Después de varios intentos encontré que el valor

probado había sido otro terminado también en 500, pero con unos

pocos miles menos. ¿Qué número es y qué cuadrados de números

primos consecutivos lo forman? (Hay otro más pequeño que también

termina en 500)

(4) Si dispusiéramos de la función primprox(N) “próximo primo después

de N” podríamos preparar un algoritmo que nos devolviera la serie de

este tipo de números. Esta función PRIMPROX está publicada en el

Page 19: Números y formas - Aprender y divertirse con hoja de …hojamat.es/publicaciones/numform.pdf · Este es uno de los capítulos de las Matemáticas en los que se aprecia ... también

19

Anexo del libro “Números y hoja de cálculo I” de nuestra colección

Hojamat.es

Bastaría iniciar cuatro variables a=2, b=3, c=5, d=7, los cuatro primeros

primos, y después ir copiando una en otra de esta forma: a=b, b=c,

c=d, d=primprox(d). Así se recorrerían todas las cuaternas de números

primos consecutivos y bastaría elevarlos al cuadrado y sumar. Así se

ha construido esta lista en hoja de cálculo:

Orden Primer primo Suma

1 2 87

2 3 204

3 5 364

4 7 628

5 11 940

6 13 1348

7 17 2020

8 19 2692

9 23 3700

10 29 4852

11 31 5860

12 37 7108

13 41 8548

14 43 10348

15 47 12220

(5) Las consideraciones anteriores nos permiten construir un algoritmo

que nos determine, dado un número múltiplo de 4 si es suma de

cuadrados de primos consecutivos o no. ¿Qué pasos tendría?

(6) Si repitiéramos el estudio con sumas de tres cuadrados en lugar de

cuatro, no obtendríamos ningún resultado, salvo el que comienza con

52, termina en 5. ¿Por qué?

Si lo intentáramos con sumas de dos primos, ningún resultado termina

en 6, y a partir del cuarto, tampoco en 4. ¿Cuál es la razón?

Page 20: Números y formas - Aprender y divertirse con hoja de …hojamat.es/publicaciones/numform.pdf · Este es uno de los capítulos de las Matemáticas en los que se aprecia ... también

20

PIEZAS PARA CUADRADOS

En otra parte de esta publicaión hemos construido triángulos

concatenando cifras. Vamos a intentarlo con cuadrados. Este tema

está estudiado, y ya hay más casos publicados en OEIS. Los

recorremos.

n//n

Es difícil que un número concatenado consigo mismo produzca un

cuadrado. Los pocos casos que aparecen ya están publicados:

1322314049613223140496, 2066115702520661157025,

2975206611629752066116, 4049586776940495867769,

5289256198452892561984,… http://oeis.org/A092118

La razón de que se descubran tan pocos es la siguiente: el número

concatenado n//n es en realidad n*(10^c+1), siendo c el número de

cifras de n, por lo que n<10^c.

Por ejemplo, 7878=78*(10^2+1)=78*101. Si deseamos que n//n sea un

cuadrado, 10^c+1 ha de contener algún cuadrado como factor, porque

si es libre de cuadrados, es imposible que n aporte los factores que

quedan para completar un cuadrado, puesto que es menor que 10^c.

Habrá que buscar números del tipo 10^k+1 que no sean libres de

cuadrados.

Si factorizamos, por ejemplo, desde 11 hasta 10^50+1, descubrimos

que sólo en cuatro casos contiene un cuadrado (copiamos la tabla

parcialmente)

100000000001=11^2*23*4093*8779

1000000000000000000001=7^2*11*13*127*2689*459691*909091

1000000000000000000000000000000001=7*11^2*13*23*4093*8779*5

99144041* 183411838171

Page 21: Números y formas - Aprender y divertirse con hoja de …hojamat.es/publicaciones/numform.pdf · Este es uno de los capítulos de las Matemáticas en los que se aprecia ... también

21

1000000000000000000000000000000000000001=7*11*13^2*157*859

*6397*216451*1058313049* 388847808493.

Para completar un cuadrado como el que se pide, n deberá contener

los factores primos que no figuran al cuadrado (la parte libre) y

además, si acaso, otros factores adicionales elevados al cuadrado. Ya

vimos en su día que si multiplicamos N por la parte libre de N

conseguiremos el mínimo múltiplo cuadrado de N

(http://hojaynumeros.blogspot.com.es/2011/12/emparedado-de-

cuadrados-2.html).

Cumplido esto, deberá tener el número de cifras adecuado. Por

ejemplo, en el primer caso

N=23*4093*8779*k^2 y si queremos que tenga 11 cifras, el valor

mínimo de k es 4, con lo que nos da la primera solución:

n=23*4093*8779*16=13223140496, que engendra el cuadrado

1322314049613223140496, primer término de http://oeis.org/A092118

Si tomamos k=5 obtenemos el segundo término: n=20661157025, que

engendra el segundo cuadrado 2066115702520661157025

Para k=6 se engendra el tercer cuadrado: 2975206611629752066116 y

para k=7, 8 o 9 se engendran los términos cuarto a sexto. A partir de

este valor se sobrepasan las cifras.

Así que el caso 100000000001 engendra seis términos.

Pasamos al siguiente:

1000000000000000000001=7^2*11*13*127*2689*459691*909091

El valor adecuado de n será del tipo

n=11*13*127*2689*459691*909091*k^2

Para k=1 y k=2 no se llega al número de cifras mínimo. Para k=3 nos

resulta el séptimo término:

183673469387755102041183673469387755102041.

Page 22: Números y formas - Aprender y divertirse con hoja de …hojamat.es/publicaciones/numform.pdf · Este es uno de los capítulos de las Matemáticas en los que se aprecia ... también

22

No están publicados más términos. Para k=4, 5 y 6 nos resultan

términos inéditos:

326530612244897959184326530612244897959184

510204081632653061225510204081632653061225

734693877551020408164734693877551020408164

No deseamos marear a nuestros lectores, por lo que no abordamos los

siguientes casos. Sólo dejamos una muestra:

132231404958677685950413223140496132231404958677685950413

223140496

2n//n

Se puede seguir el mismo razonamiento y descomponer en factores

los números del tipo 2*10^c+1 que contienen cuadrados

20000000000000000001=3*7^2*83*1663*985694468327

2000000000000000000000000000000001=3*43^2*245169227*147063

8299531951365929

Con el primero

32653061224489795921632653061224489796

73469387755102040823673469387755102041

130612244897959183686530612244897959184

Con el segundo

216333153055705786911844240129800108166576527852893455922

120064900

261763115197404002163331530557058130881557598702001081665

765278529

311519740400216333153055705786912155759870200108166576527

852893456

Page 23: Números y formas - Aprender y divertirse con hoja de …hojamat.es/publicaciones/numform.pdf · Este es uno de los capítulos de las Matemáticas en los que se aprecia ... también

23

365603028664142779881016765819362182801514332071389940508

382909681

424012979989183342347214710654408212006489994591671173607

355327204

486749594375338020551649540292050243374797187669010275824

770146025

553812871822606814494321254732288276906435911303407247160

627366144

625202812330989724175229853975122312601406165494862087614

926987561

700919415900486749594375338020552350459707950243374797187

669010276

780962682531097890751757706868578390481341265548945375878

853434289

865332612222823147647376960519200432666306111411573823688

480259600

Los primeros están publicados en http://oeis.org/A115529

n//2n

No explicamos ya el procedimiento. Los candidatos son:

12=2^2*3

100000000000000000000002=2*3*7^2*19961*17040030781111603

100000000000000000000000000000000000000000000000000000000

02=2*3*89^2*

353891*184629530872289*3220312754723112768886882952137673

Con el primero obtenemos el 36=6^2

Con el segundo:

Page 24: Números y formas - Aprender y divertirse con hoja de …hojamat.es/publicaciones/numform.pdf · Este es uno de los capítulos de las Matemáticas en los que se aprecia ... también

24

816326530612244897959216326530612244897959184

1836734693877551020408236734693877551020408164

3265306122448979591836865306122448979591836736

Y con el tercero, verdaderos gigantes:

504986744097967428355005681100871102133568993813912384800

100997348819593485671001136220174220426713798762782476960

0

Publicados en http://oeis.org/A115527

Concatenación con diferencias constantes

Casi todos los casos están estudiados. Aquí ya no disponemos del

análisis de los factores de 2*10^c+1 o similares. Sólo podemos acudir

a la búsqueda, porque al añadir un sumando al número todo el

planteamiento anterior falla.

n//n+1

Los primeros ejemplos los buscaremos con hoja de cálculo (no

mostraremos el código) y con PARI.

Hemos añadido la raíz cuadrada de la concatenación. Esta sucesión

está publicada en http://oeis.org/A030465 y llama a los primeros

números de Sastry.

El código PARI adecuado es

concatint(a,b)=eval(concat(Str(a),Str(b)))

{for(n=1,10^7,a=concatint(n,n+1);if(issquare(a),print(a)))}

N N//N+1 Raíz

183 183184 428

328 328329 573

528 528529 727

715 715716 846

6099 60996100 7810

13224 1322413225 36365

40495 4049540496 63636

Page 25: Números y formas - Aprender y divertirse con hoja de …hojamat.es/publicaciones/numform.pdf · Este es uno de los capítulos de las Matemáticas en los que se aprecia ... también

25

N+1//n

Los primeros ejemplos son

También publicado en http://oeis.org/A054214

Adapta tú el código PARI para encontrar más.

n//n+2

El número par 7874 es el más pequeño que cumple que concatenado

con el siguiente par 7876 produce un cuadrado: 78747876=8874^2.

Este caso ya está publicado en http://oeis.org/A115426.

n+2//n

Es el problema simétrico del anterior y también está estudiado en

http://oeis.org/A115431

Aquí paramos, porque otras concatenaciones resultan menos

atractivas. No obstante, con lo que ya has leído puedes emprender

búsquedas por tu cuenta.

CARNAVAL DE CUADRADOS

Consideremos el conjunto de divisores de un número natural N que

son cuadrados perfectos. Sabemos que el mayor de ellos es la parte

cuadrada del número), a la que designaremos como PC(N).

(ver http://hojaynumeros.blogspot.com.es/2011/05/parte-cuadrada-y-

parte-libre.html)

Si descomponemos N en factores primos

N N//N+1 Raíz

81 8281 91

8241 82428241 9079

9801 98029801 9901

Page 26: Números y formas - Aprender y divertirse con hoja de …hojamat.es/publicaciones/numform.pdf · Este es uno de los capítulos de las Matemáticas en los que se aprecia ... también

26

(1)

para encontrar la parte cuadrada basta elevar a cada factor primo al

mayor número par contenido en cada uno de los exponentes, es decir

(2)

Así, por ejemplo, para encontrar la parte cuadrada de

26460=22*33*5*72 bastará truncar cada exponente a un número par,

con lo que quedaría PC(26460)= 22*32*72=1764. A la raíz cuadrada de

esa parte se le suele llamar Raíz Interna del número N

(ver http://hojaynumeros.blogspot.com.es/2011/12/emparedado-de-

cuadrados-2.html)

En este caso la raíz interna de 26460 sería 42=2*3*7.

Todo esto lo recordamos para poder estudiar mejor los divisores

cuadrados de un número. Se pueden considerar las siguientes

afirmaciones:

Los divisores cuadrados de N coinciden con los de su parte

cuadrada.

Si k es divisor cuadrado de N, todos sus exponentes en (1) serán

pares, pero ninguno sobrepasará al correspondiente en PC(N), luego

será también divisor de esa parte cuadrada. Inversamente, todo divisor

de PC(N) lo es también de N.

El número de divisores cuadrados de N coincide con el de los

divisores de la raíz interna de N.

Esto es así porque si extraemos la raíz cuadrada a todos los divisores

cuadrados de N, es claro que permanecerán los mismos factores

primos pero con sus exponentes reducidos a la mitad, que es la misma

operación sufrida por la raíz interna.

Page 27: Números y formas - Aprender y divertirse con hoja de …hojamat.es/publicaciones/numform.pdf · Este es uno de los capítulos de las Matemáticas en los que se aprecia ... también

27

En el ejemplo elegido, si esa raíz interna es 42, poseerá ocho

divisores, por ser igual a 2*3*7 (aplicando la fórmula del número de

divisores resultaría (1+1)(1+1)(1+1)=8). Efectivamente, si buscamos

todos los divisores cuadrados de 26460 nos resultan estos ocho: 1764,

441, 196, 49, 36, 9, 4 y 1, que son los cuadrados de los divisores de

42: 42, 21, 14, 7, 6, 3, 2 y 1

Existe una correspondencia biyectiva entre los divisores

cuadrados de N y los divisores de su raíz interna, de forma que

cada uno de los primeros es el cuadrado de otro del segundo

conjunto.

Por ejemplo, para N=1200, su parte cuadrada es 400, su raíz interna

20, y se da la correspondencia entre los divisores de 20 y los divisores

cuadrados de 20.

Esto nos da, como hemos visto, un procedimiento para contar los

divisores cuadrados de un número, pero también para sumarlos, si

recordamos la fórmula de la función sigma_2, que suma los cuadrados

de los divisores (ver http://hojaynumeros.blogspot.com.es/2011/03/la-

familia-de-las-sigmas-2.html)

Aplicamos esa fórmula a la raíz interna. Esto es importante, porque esa

raíz determina el número de divisores cuadrados. En nuestro ejemplo

lo haríamos así:

SDC(26460)=(2^4-1)/(2^2-1)* (3^4-1)/(3^2-1)* (7^4-1)/(7^2-

1)=5*10*50=2500

Comprueba: 1764+441+196+49+36+9+4+1=2500

Page 28: Números y formas - Aprender y divertirse con hoja de …hojamat.es/publicaciones/numform.pdf · Este es uno de los capítulos de las Matemáticas en los que se aprecia ... también

28

Si deseas comprobar este resultado con otros números, con este

codigo PARI puedes sumar todos los divisores cuadrados:

print(sumdiv(26460,d,d*issquare(d)))

Sustituyes el ejemplo 26460 por otro número cada vez que lo desees.

Con el Basic de las hojas de cálculo también lo puedes calcular

mediante esta función:

public function sumdivcuad(n)

dim i,p,a,s

p=1

s=0

for i=1 to sqr(n)

a=i*i

if n/a=n\a then s=s+a

next i

sumadivcuad=s

end function

Comprueba de varias formas que el número 84000 posee sólo seis

divisores cuadrados cuya suma es 546. Usa también la fórmula basada

en sigma_2 ((2^6-1)/(2^2-1)*(5^4-1)/(5^2-1)=21*26=546)

Como otras variantes de la función sigma, esta suma de divisores

cuadrados es una función multiplicativa, por lo que basta definirla para

pr, siendo p un factor primo. Para ello, según (2) tomamos como

exponente de su raíz interna (r – r MOD 2)/2, con lo que la suma de los

divisores cuadrados será

Por ejemplo, la suma de divisores cuadrados de 2048=211 será igual a

(2^12-1)/(2^2-1)=4095/3=1365. Comprobamos: 1024+256+64+16+4+1

= 1365.

Page 29: Números y formas - Aprender y divertirse con hoja de …hojamat.es/publicaciones/numform.pdf · Este es uno de los capítulos de las Matemáticas en los que se aprecia ... también

29

En el caso particular de que r sea igual a 2 o a 3 la suma de divisores

cuadrados será p2+1. Es muy fácil razonarlo y lo usaremos más

adelante.

Otro caso particular se da cuando la raíz interna está libre de

cuadrados, tipo RI(N)=p*q*r*s…, la suma buscada será

(1+p2)(1+q2)(1+r2)(1+s2)…Sería el caso, por ejemplo, del número

60500, cuya parte cuadrada es 12100 y la raíz interna 110=2*5*11,

libre de cuadrados, por lo que la suma de divisores cuadrados de

60500 debería ser (1+22)(1+52)(1+112)=5*26*122=15860. En efecto,

los divisores cuadrados de 60500 suman

12100+3025+484+121+100+25+4+1=1586

RESULTADOS CURIOSOS DE LA SUMA DE

DIVISORES CUADRADOS

Engendramos cuadrados

La siguiente sucesión presenta varias propiedades respecto a la suma

de los divisores cuadrados que merece la pena destacar

1764, 60516, 82369, 529984, 2056356, 2798929, 3534400, 18181696,

38900169, 96020401, 97121025, 335988900, 455907904, 457318225,

617820736, 1334513961, 1599200100, 2176689025, 3279852900,

4464244225, 8586616896…

(publicada en https://oeis.org/A232554)

Page 30: Números y formas - Aprender y divertirse con hoja de …hojamat.es/publicaciones/numform.pdf · Este es uno de los capítulos de las Matemáticas en los que se aprecia ... también

30

Todos ellos son cuadrados tales que la suma de sus divisores

cuadrados, incluidos ellos mismos, también es un cuadrado. Sí,

puedes volver a leerlo si no lo has captado. En la siguiente tabla

puedes comprobar esta propiedad:

En la primera columna figuran los elementos de la sucesión. Hemos

prescindido del 1, que también cumpliría la misma propiedad. En la

siguiente su descomposición en factores primos, que ya analizaremos.

Como en el caso anterior sugeríamos sumar los divisores cuadrados

mediante la función sigma_2 aplicada a la raíz interna, hemos

calculado dicha suma en las siguientes columnas, comprobando

mediante su raíz cuadrada que se trata de cuadrados perfectos.

Finamente también se han calculado los factores de esas raíces.

Se pueden generar con este código en lenguaje PARI:

{for(n=1,10^5,m=n*n;k=sumdiv(m,d,d*issquare(d));if(issquare(k)&

&k>>1,print(m)))}

Factorización

Podemos observar que ningún término de la sucesión es potencia de

un solo primo.

Con dos factores primos distintos sólo se dan tres casos, que puedes

buscar en la tabla, y los primos que intervienen son 7, 41 y 239,

curiosamente pertenecientes a la sucesión de primos p para los que

p^2+1 no está libre de cuadrados

A(n) Factores Raiz sigma_2 Raiz Factores

1764 [2,2][3,2][7,2] 42 2500 50 [2,1][5,2]

60516 [2,2][3,2][41,2] 246 84100 290 [2,1][5,1][29,1]

82369 [7,2][41,2] 287 84100 290 [2,1][5,1][29,1]

529984 [2,6][7,2][13,2] 728 722500 850 [2,1][5,2][17,1]

2056356 [2,2][3,2][239,2] 1434 2856100 1690 [2,1][5,1][13,2]

2798929 [7,2][239,2] 1673 2856100 1690 [2,1][5,1][13,2]

3534400 [2,6][5,2][47,2] 1880 4884100 2210 [2,1][5,1][13,1][17,1]

18181696 [2,6][13,2][41,2] 4264 24304900 4930 [2,1][5,1][17,1][29,1]

38900169 [3,8][7,2][11,2] 6237 45024100 6710 [2,1][5,1][11,1][61,1]

96020401 [41,2][239,2] 9799 96079204 9802 [2,1][13,2][29,1]

97121025 [3,6][5,2][73,2] 9855 113635600 10660 [2,2][5,1][13,1][41,1]

335988900 [2,2][3,2][5,2][13,2][47,2] 18330 488410000 22100 [2,2][5,2][13,1][17,1]

455907904 [2,6][17,2][157,2] 21352 607622500 24650 [2,1][5,2][17,1][29,1]

457318225 [5,2][7,2][13,2][47,2] 21385 488410000 22100 [2,2][5,2][13,1][17,1]

617820736 [2,6][13,2][239,2] 24856 825412900 28730 [2,1][5,1][13,2][17,1]

1334513961 [3,8][11,2][41,2] 36531 1514610724 38918 [2,1][11,1][29,1][61,1]

1599200100 [2,2][3,2][5,2][31,2][43,2] 39990 2313610000 48100 [2,2][5,2][13,1][37,1]

2176689025 [5,2][7,2][31,2][43,2] 46655 2313610000 48100 [2,2][5,2][13,1][37,1]

3279852900 [2,2][3,2][5,2][23,2][83,2] 57270 4747210000 68900 [2,2][5,2][13,1][53,1]

4464244225 [5,2][7,2][23,2][83,2] 66815 4747210000 68900 [2,2][5,2][13,1][53,1]

8586616896 [2,6][3,8][11,2][13,2] 92664 13011964900 114070 [2,1][5,1][11,1][17,1][61,1]

Page 31: Números y formas - Aprender y divertirse con hoja de …hojamat.es/publicaciones/numform.pdf · Este es uno de los capítulos de las Matemáticas en los que se aprecia ... también

31

(ver el documento de Rafael Parra

http://hojamat.es/parra/NumerosLDC.pdf

y la sucesión https://oeis.org/A224718).

En el caso de los tres citados, 7^2+1=2*25^2, 41^2+1=2*29^2 y

239^2+1=2*13^4. Si ahora los multiplicamos dos a dos, obtendremos

un factor 2*2=4 multiplicado por dos cuadrados, luego será cuadrado

perfecto, como se pedía.

Otra curiosidad es que las sumas de cuadrados son todas pares y

muchas de ellas múltiplos de 100. Sus raíces son pares hasta donde

hemos buscado. Queda ahí abierta una cuestión para estudiarla con

más ciencia que nosotros.

Sucesión derivada

Si multiplicamos los términos de esta sucesión por otro número libre

de cuadrados resultará otra sucesión formada por números no

cuadrados con suma de divisores cuadrados propios que resulta ser

cuadrada:

3528, 5292, 8820, 10584, 12348, 17640, 19404, 22932, 24696, 26460,

29988, 33516, 37044, 38808, 40572, 45864, 51156, 52920, 54684,

58212, 59976, 61740, 65268, 67032, 68796, 72324, 74088, 75852,

81144, 82908, 89964, 93492, 97020…(publicada en

https://oeis.org/A232555)

Podemos construir todos los múltiplos de ese tipo hasta una cota, por

ejemplo un millón y después ordenarlos en sucesión. Así lo hemos

hecho y casi todos los primeros son múltiplos de 1764.

En realidad esta sucesión es parte de otra más amplia en la que

aparecen todos los casos, y no sólo estos múltiplos que hemos

considerado. Son estos:

Números cuya suma de divisores cuadrados propios es otro

cuadrado mayor que 1

900, 3528, 4900, 5292, 8820, 10404, 10584, 12348, 17640, 19404,

22932, 24696, 26460, 29988, 33516, 37044, 38808, 40572, 45864,

Page 32: Números y formas - Aprender y divertirse con hoja de …hojamat.es/publicaciones/numform.pdf · Este es uno de los capítulos de las Matemáticas en los que se aprecia ... también

32

51156, 52920, 54684, 58212, 59976, 61740, 65268, 67032, 68796,

72324, 74088, 75852, 79524, 81144, 81796, 82908, 89964, 93492,

97020…(publicada en https://oeis.org/A232556)

En ellos la suma de divisores cuadrados propios es otro cuadrado. Por

ejemplo, la suma en el caso de 5292 es

1764+441+196+49+36+9+4+1=2500=50^2,

que también es un cuadrado.

Aunque los hemos buscado con funciones de hoja de cálculo, se

puede intentar también con PARI. Prueba si quieres este código:

{for(n=1,10^5,k=sumdiv(n,d,d*issquare(d)*(d<n));if(issquare(k)&&k

>>1,print(n)))}

Todos los encontrados son múltiplos de 4 y al menos poseen tres

factores primos distintos. De ellos, algunos son también cuadrados:

900, 4900, 10404, 79524, 81796, 417316, 532900, 846400, 1542564,

2464900, 3232804, 3334276, 3496900, 12432676, 43850884,

50836900, 51811204, 71470116, 107453956, 236975236, 253892356,

432889636, 544102276, 864948100, 1192597156, 1450543396,

1554094084, 2024820004, 2165413156…(publicada en

https://oeis.org/A232557)

No son cuadrados el resto: 3528, 5292, 8820, 10584, 12348,…que

resultan ser los múltiplos de la primera sucesión que ya tratamos.

Resumimos:

Sucesiones de cuadrados

(1) Pueden formar un cuadrado sumándoles todos sus divisores

cuadrados propios. Nos resultaría la primera sucesión: 1764, 60516,

82369, 529984,…(A232554)

(2) Forman un cuadrado sólo la suma de divisores propios, sin

sumarles el número dado. Tendríamos la sucesión: 900, 4900, 10404,

79524, 81796,…(A232557)

Page 33: Números y formas - Aprender y divertirse con hoja de …hojamat.es/publicaciones/numform.pdf · Este es uno de los capítulos de las Matemáticas en los que se aprecia ... también

33

Sucesiones de no cuadrados

(3) Números cuyos divisores cuadrados suman otro cuadrado. Son

3528, 5292, 8820, 10584,…(A232555) Son múltiplos de elementos de

la sucesión (1)

Sin condicionamiento

(4) La unión de la sucesión (2) con la (3) (A232556)

Formamos palindrómicos

Con la suma de divisores cuadrados podemos formar números

palindrómicos. Es una simple curiosidad, pero está inédita, que

sepamos. Hay dos formas, con divisores cuadrados propios o con

todos:

Con divisores propios

Estos son los números en los que la suma de divisores cuadrados

propios es un número palindrómico de al menos dos cifras (para

eliminar casos triviales):

144, 324, 1089, 1936, 5929, 13225, 30752, 46128, 58564, 76880,

92256, 107632, 125316, 138384, 149769, 153760, 154449, 169136,

199888, 215264, 230640, 261392, 292144, 322896, 338272, 342225,

353648, 378225, 399776, 405769, 445904, 461280, 476656, 507408,

522784, 538160, 568912, 584288, 599664,…

(Los hemos publicado en https://oeis.org/A232892)Si expresamos el

resultado en una tabla de dos columnas, vemos los resultados

palindrómicos a la derecha:

Page 34: Números y formas - Aprender y divertirse con hoja de …hojamat.es/publicaciones/numform.pdf · Este es uno de los capítulos de las Matemáticas en los que se aprecia ... también

34

Llama la atención la frecuencia con la que aparece el valor 20202, y

prolongando la tabla veríamos muchos más. La razón de esto es que el

primer caso, 30752=25*312, tiene como divisores cuadrados

15376+3844+961+16+4+1=20202, que provienen de los factores

24*312 =15376 y entonces, si multiplicamos ese número por factores

libres de cuadrados se volverá a dar el mismo caso. En efecto, según

la tabla, los siguientes son: 46128=15376*3, 76880=15376*5,

92256=15376*6, 107632=15376*7,…

La pregunta es por qué no funciona este razonamiento en los primeros

casos de la tabla. La respuesta es que esos números son cuadrados y

si los multiplicamos por un libre de cuadrados, se convertirían ellos

mismos en divisores cuadrados propios, y eso alteraría la suma.

Un código PARI para encontrarlos puede ser

reverse(n)=concat(Vecrev(Str(n)))

palind(n)=(Str(n)==reverse(n)&&n>10)

{for(n=1,10^5,k=sumdiv(n,d,d*issquare(d)*(d<n));if(palind(k),print(

n)))}

Con todos los divisores cuadrados

Los primeros números con esta propiedad son

15376, 30752, 46128, 76880, 92256, 107632, 153760, 169136,

199888, 215264, 230640, 261392, 292144, 322896, 338272, 353648,

399776, 445904, 461280, 476656, 507408, 522784, 538160, 568912,

584288, 599664, 630416, 645792, 661168, 707296, 722672, 784176,

144 66

324 131

1089 131

1936 626

5929 171

13225 555

30752 20202

46128 20202

58564 15251

76880 20202

92256 20202

107632 20202

125316 48784

Page 35: Números y formas - Aprender y divertirse con hoja de …hojamat.es/publicaciones/numform.pdf · Este es uno de los capítulos de las Matemáticas en los que se aprecia ... también

35

814928, 845680, 876432, 891808, 907184, 937936, 953312,

999440,…

(Los hemos publicado en https://oeis.org/A232893)

Todos producen la suma de cuadrados 20202, que ya vimos, y todos

son múltiplos del primero 15376 con cociente libre de cuadrados. Esta

situación llega hasta el número 2217121, que ya no es múltiplo de

15376 y la suma palindrómica que produce es 2217122, ya que sus

únicos divisores cuadrados son él mismo y la unidad.

Código PARI:

reverse(n)=concat(Vecrev(Str(n)))

palind(n)=(Str(n)==reverse(n)&&n>10)

{for(n=1,10^5,k=sumdiv(n,d,d*issquare(d));if(palind(k),print(n)))}

Otras sumas

Podemos intentar lograr números de otros tipos, como triangulares u

oblongos, pero los resultados son tan abundantes que pierden su

interés. En el caso de los oblongos los primeros resultados son

múltiplos de 144. Ahí tienes una exploración.

IGUALDAD DE SUMAS DE CUADRADOS CON

UN ESCALÓN

Repasando algunas propiedades curiosas me encontré en hojamat.es

con esta:

365=102+112+122 = 132+142

La pregunta inmediata que me surgió fue la de si existían otros

números con la misma propiedad o similar. Los encontré en OEIS

(http://oeis.org/) pero no descubriré dónde por ahora, aunque los

lectores experimentados sabrán hallarlos. Con esto quiero aclarar que

lo que consigamos está ya descubierto, pero el objetivo (tan frecuente

Page 36: Números y formas - Aprender y divertirse con hoja de …hojamat.es/publicaciones/numform.pdf · Este es uno de los capítulos de las Matemáticas en los que se aprecia ... también

36

en este blog) es intentar la concurrencia de métodos y el uso de la hoja

de cálculo.

Nos acercaremos al problema con el planteamiento de dos preguntas:

¿Existen más números en los que la suma de tres cuadrados

consecutivos coincida con los dos siguientes?

¿Qué ocurrirá si aumentamos o disminuimos el número de

cuadrados?

Es probable que hayas pensado en el 25=32+42=52, luego parece que

sí existen casos similares. Lo vemos.

Acercamiento con la hoja de cálculo

Si concretamos un número de inicio n y un número de cuadrados igual

a k+1 en el primer miembro y a k en el segundo, con estas sencillas

líneas podemos descubrir si existen otros casos:

For i=1 to 10000 (por ejemplo)

‘calcula el primer miembro

a = 0

For l = 0 To k

a = a + (i + l) ^ 2

Next l

‘calcula el segundo miembro

b = 0

For l = k + 1 To 2 * k

b = b + (i + l) ^ 2

Next l

‘Los compara y si son iguales lo comunica

If a = b Then

Msgbox(n)

Msgbox(a)

End If

Next i

Hemos tomado como tope 10000, pero después habrá quizás que

ampliar. Implementa esto como rutina en tu hoja de cálculo y

Page 37: Números y formas - Aprender y divertirse con hoja de …hojamat.es/publicaciones/numform.pdf · Este es uno de los capítulos de las Matemáticas en los que se aprecia ... también

37

descubrirás que para cada k existe una solución y sólo una.

Recogemos en una tabla los primeros resultados:

Ahora ya descubrimos que los resultados coinciden con los recogidos

en http://oeis.org/A059255, pero no podemos dejarlo así, porque en la

tabla aparecen números triangulares y múltiplos de 5. Algo habrá

detrás. Intentamos descubrirlo.

Un poco de Álgebra

Si sospechamos que las soluciones son únicas para cada valor de k,

es probable que exista una relación algebraica sencilla. En efecto,

aunque los principios son algo farragosos, con paciencia algebraica

llegaremos a la meta. No damos todos los detalles y te dejamos

practicar:

Primera suma de cuadrados A

Suponemos que comienza en n y termina en n+k (k+1 sumandos), es

decir:

Segunda suma de cuadrados B

Observa cómo lo hemos escrito, para que te aproveches de la fórmula

para la suma de números naturales consecutivos.

k n a

1 3 25

2 10 365

3 21 2030

4 36 7230

5 55 19855

6 78 45955

7 105 94220

Page 38: Números y formas - Aprender y divertirse con hoja de …hojamat.es/publicaciones/numform.pdf · Este es uno de los capítulos de las Matemáticas en los que se aprecia ... también

38

Desarrolla cada suma separando los coeficientes de n2, de n y los

independientes. Como esta tarea te puede llevar a la desesperación,

usa las dos populares fórmulas:

Calcula A-B para igualarla a

cero y ve encontrando los

coeficientes:

De n2 te deberá resultar a=1.

Es fácil verlo.

De n, si sabes usar la primera

fórmula ofrecida, con algún retoque, te dará b=-2k2

El coeficiente independiente es un poco más complejo de encontrar

correctamente. Puedes usar la suma de cuadrados de los primeros

naturales. Deberá resultar c=-2k3-k2

Así que la ecuación para calcular n quedaría así:

Su discriminante es el cuadrado de 2k(k+1), lo que nos garantiza una

solución entera. Tomamos la positiva y, efectivamente n=k(2k+1), que

es el número triangular de orden 2k, como habíamos sospechado al

principio.

Para cada valor de k, la igualdad de cuadrados pretendida ocurre

para n=k(2k+1), el número triangular correspondiente a 2k, y es

por tanto la solución única.

Hemos resuelto con rigor lo que sospechábamos tras el uso de la hoja

de cálculo. Esto es imprescindible: las herramientas informáticas sólo

proponen o dan pistas, pero no demuestran nada. A veces olvidamos

esta limitación.

Expresión de la suma

Ahora podemos calcular el valor de las dos sumas. Sustituimos k(2k+1)

en una de ellas, y sacando factor común nos resulta

Page 39: Números y formas - Aprender y divertirse con hoja de …hojamat.es/publicaciones/numform.pdf · Este es uno de los capítulos de las Matemáticas en los que se aprecia ... también

39

A(k)=k(k+1)(2k+1)(12k2+12k+1)/6. Por ejemplo, para k=3 resulta

3*4*7*(12*9+12*3+1)=2030.

El problema se ha reducido a una cuestión algebraica.

CUADRADOS DEL TIPO N(N+K)

En enero de 2018 se publicaron en Twitter varios resultados sobre una

propuesta de Republic of Math @republicofmath, uno de los cuales

insertamos a continuación:

En este blog acudimos con frecuencia a la técnica de “dar vueltas” a

una cuestión de la que tengamos noticia. En este caso concreto

estudiaremos de forma algebraica y algorítmica la cuestión de qué

números n cumplen que n(n+k) es un cuadrado para un número k

dado. Por ejemplo, si k=16, existen dos números, 2 y 9, que producen

un cuadrado mediante dicha expresión. En efecto:

2(2+16)=2*18=36=62; 9(9+16)=9*25=225=152

Para encontrar soluciones correspondientes a un valor de k

acudiremos a búsqueda en hoja de cálculo y PARI para después

abordar un estudio teórico. Es una metodología muy frecuente en este

blog: comenzar con una búsqueda sin apoyo teórico y después ir

buscando regularidades o fundamentaciones algebraicas.

Estudio con hoja de cálculo

Es fácil programar una función que nos indique si un número n produce

un cuadrado en la expresión n(n+k) para un k dado. La que insertamos

Page 40: Números y formas - Aprender y divertirse con hoja de …hojamat.es/publicaciones/numform.pdf · Este es uno de los capítulos de las Matemáticas en los que se aprecia ... también

40

a continuación nos servirá para valores de k pequeños. En otros casos

los errores de truncamiento de los cálculos nos pueden llevar a

conclusiones falsas. Por eso es interesante acudir a un lenguaje de

más exactitud como PARI y dar una vuelta de Álgebra a esta cuestión.

La función más simple que podemos proponer es esta:

Function escuadprod(n, k) As Boolean

‘Depende de dos variables y devuelve VERDADERO o FALSO

Dim a

a = n * (n + k) ‘Construimos el producto pedido

If a = Int(Sqr(a)) ^ 2 Then escuadprod = True Else escuadprod =

False

‘Si a equivale al cuadrado de la parte entera de su raíz cuadrada, vale

End Function

Con ella y un bucle de búsqueda se pueden encontrar las soluciones

para un valor de k dado. En la imagen figuran las correspondientes a

k=21, que son 4, 7, 27 y 100.

Las comprobamos:

4(4+21)=4*25=100=102

7(7+21)=7*28=196=142

27(27+21)=27*48=1296=362

100(100+21)=100*121=12100=1102

¿Se podrán encontrar más? Lo veremos en el estudio algebraico.

Page 41: Números y formas - Aprender y divertirse con hoja de …hojamat.es/publicaciones/numform.pdf · Este es uno de los capítulos de las Matemáticas en los que se aprecia ... también

41

Para mayor velocidad y exactitud trasladamos vuestra función a PARI

(lo puedes realizar fácilmente con otro lenguaje). Este sería el listado

para k=21.

escp(n,k)=issquare(n*(n+k))

k=21;for(i=1,10000,if(escp(i,k),print(i)))

Hemos acotado la búsqueda en 10000, pero para valores de k

superiores deberíamos ampliar la búsqueda. Ya trataremos esto más

adelante.

El resultado es:

Coinciden las soluciones 4, 7, 27 y 100.

Estudio algebraico

Llamemos m2 al cuadrado que debe producir el producto n(n+k).

En ese caso se cumplirá: n(n+k)=m2

Resolviendo la ecuación de segundo grado resultante:

𝑛2 + 𝑘𝑛 − 𝑚2 = 0

𝑛 =−𝑘 + √𝑘2 + 4𝑚2

2

El radicando ha de ser un cuadrado perfecto, llamémosle p2, lo que nos

lleva a que

p2 - 4m2 = k2

(p+2m)(p-2m)=k2

Page 42: Números y formas - Aprender y divertirse con hoja de …hojamat.es/publicaciones/numform.pdf · Este es uno de los capítulos de las Matemáticas en los que se aprecia ... también

42

Si encontramos dos valores enteros para p y m, sustituyendo en la

resolución de la ecuación:

𝑛 =𝑝 − 𝑘

2

Bastará entonces descomponer k2 en dos factores, sean A y B, e

igualar:

Resultará p+2m=A, p-2m=B, p=(A+B)/2, m=(A-B)/4

Deberemos buscar entonces A y B de forma que su diferencia sea

múltiplo de 4 y distinto de cero, para evitar la solución cero.

Veamos el caso de 21. Su cuadrado 441 admite estas

descomposiciones en productos:

441*1=147*3=63*7=49*9=21*21

Los cuatro primeros se diferencian en un múltiplo de 4, y nos queda:

441*1: m=(441-1)/4=110; p=(441+1)/2=221; n=(p-k)/2=(221-21)/2=100

147*3: m=(147-3)/4=36; p=(147+3)/2=75; n=(75-21)/2=27

63*7: m=(63-7)/4=14; p=(63+7)/2=35; n=(35-21)/2=7

49*9: m=(49-9)/4=10; p=(49+9)/2=29; n=(29-21)/2=4

Obtenemos las soluciones sabidas 100, 27, 7 y 4. Hemos desechado la

solución cero, que se obtendría de 21*21.

Hemos deducido que el valor de p equivale a p=(A+B)/2, siendo A y B

factores de k2, luego p<=k2/2, y, por tanto, el valor buscado n=(p-k)/2,

tendrá como amplia cota k^2/4:

𝑛 ≤𝑘2

4

Podíamos ajustar más, pero no es necesario. Por ejemplo, para el caso

k=2018, PARI nos da la solución (por cierto, única) de n=508032, ya

publicada en Twitter

Page 43: Números y formas - Aprender y divertirse con hoja de …hojamat.es/publicaciones/numform.pdf · Este es uno de los capítulos de las Matemáticas en los que se aprecia ... también

43

Esta solución cumple 508032<2018^2/4=1018081

Por tanto, en PARI deberíamos buscar hasta esa cantidad:

escp(n,k)=issquare(n*(n+k))

k=2018;for(i=1,1018081,if(escp(i,k),print(i)))

Obtendríamos la misma solución única:

¿Por qué es única?

Seguimos el proceso algebraico:

2018^2=4072324=4072324*1=2036162*2=1018081*4=4036*1009=201

8*2018

El único par válido es 2036162*2, luego p=(2036162+2)/2=1018082 y

n=(1018082-2018)/2=508032

Es única la solución 508032

Terminamos con una consideración:

Si n convierte n(n+k) en un cuadrado, es decir, es solución para

un k dado, el número rn será solución para rk.

Es una propiedad muy importante, que se justifica en estas simples

igualdades:

Si n(n+k)=m2, entonces rn(rn+rk)=(rm)2

Número de soluciones para cuadrados del tipo n(n+k)

En el anterior apartado comenzamos a “dar vueltas” a la cuestión de

qué números n cumplen que n(n+k) es un cuadrado para un número

k dado. Ya presentamos la función escuadprod(n,k) para averiguar si

un número n forma cuadrado mediante n(n+k). Después se tradujo a

Page 44: Números y formas - Aprender y divertirse con hoja de …hojamat.es/publicaciones/numform.pdf · Este es uno de los capítulos de las Matemáticas en los que se aprecia ... también

44

PARI y se desarrolló un procedimiento algebraico para encontrar las

soluciones del problema planteado.

Comenzaremos por contar las soluciones que presenta cada valor de

k, para pasar luego a algunos casos interesantes.

Función para contar soluciones

Mediante procedimientos similares a los desarrollados en los anteriores

apartados, podemos crear la función numcuadprod(k) que cuente las

soluciones para un valor concreto de k. El siguiente código para VBA

de Excel resuelve la cuestión.

Function numcuadprod(k)

Dim a, i, r, c

c = 0 ‘Contador de soluciones

r = k ^ 2 / 4 ‘Cota de búsqueda

For i = 1 To r

a = i * (i + k)

If a = Int(Sqr(a)) ^ 2 Then c = c + 1 ‘Si se cumple, se incrementa el

contador

Next i

numcuadprod = c

End Function

Si prefieres la exactitud del lenguaje PARI, puedes usar este código:

numcp(k)={local(c=0,

a=0,r=k^2/4,i=0);for(i=1,r,a=i*(i+k);if(issquare(a),c+=1)); c}

print(numcp(960))

Lo hemos particularizado para k=960

Aquí tienes el resultado en Excel

Page 45: Números y formas - Aprender y divertirse con hoja de …hojamat.es/publicaciones/numform.pdf · Este es uno de los capítulos de las Matemáticas en los que se aprecia ... también

45

Y aquí en PARI

Con la función escuadprod(n,k) que estudiamos anteriormente

podemos buscar esas 40 soluciones. Son:

En Excel:

Y en PARI

Casos particulares

Estudiaremos a continuación algunos casos particulares en los que no

es complicado contar soluciones.

Múltiplos de 3

Todos tienen al menos una solución. Si k=3m, una solución para n es

m, ya que m(m+3m)=4m2=(2m)2

Un razonamiento similar valdría para múltiplos de 8, ya que

m(m+8m)=(3m)2

Por tanto los múltiplos de un número anterior a un cuadrado

presentarán la misma situación.

Page 46: Números y formas - Aprender y divertirse con hoja de …hojamat.es/publicaciones/numform.pdf · Este es uno de los capítulos de las Matemáticas en los que se aprecia ... también

46

Números que no presentan soluciones

Los números 1, 2 y 4 no admiten soluciones, porque sus cuadrados no

se pueden descomponer en dos factores A y B cuya diferencia sea

múltiplo de 4.

Números primos mayores que 2

Si k es primo, A=k2 y B=1. Todos los cuadrados de primos mayores

que 2 son de la forma 4m+1, ya que (2k+1)2=4(k2+k)+1,

(2k+3)2=4(k2+3k+2)+1, y por tanto A-B será múltiplo de 4. En este caso

tendremos que p=(k2+1)/2 y n=((k2+1)/2-k)/2=(k2-2k+1)/4=(k-1)2/4, o

bien:

𝑛 = (𝑘 − 1

2)

2

Así lo comunica @republicofmath

Aquí añadimos que esta solución también es válida para todos los

impares, aunque sólo tiene que ser única para los primos (recuérdese

el 21).

En el caso de 21, n=(21-1)^2/4=400/4=100, pero existen otras.

Podemos listar las soluciones para los primeros primos y comprobar

esa fórmula:

Page 47: Números y formas - Aprender y divertirse con hoja de …hojamat.es/publicaciones/numform.pdf · Este es uno de los capítulos de las Matemáticas en los que se aprecia ... también

47

Potencias de 2 mayores que 4

Si k=2r, con r>2, el número de soluciones será r-2

En efecto, k2 en ese caso se podría descomponer de la forma A=2a,

B=2b, con la condición de que a+b=2r a>b y que la diferencia A-B sea

múltiplo de 4, para lo que b ha de ser mayor que 1, ya que

A-B=2a-2b=(2a-b-1)*2b

En esa diferencia el paréntesis es impar, luego sólo será múltiplo de 4

si lo es 2b, es decir, con b>1

Esto reduce los valores de b a 2, 3, 4,…r-1, ya que el valor r produciría

la igualdad A=B. Contamos casos, y, efectivamente, son r-2

Lo vemos para el 16:

S k=16, r=4, 2r=8, y los valores posibles de b, 2 y 3

Si b=2, B=4, A=64, p=(A+B)/2=34, n=(34-16)/2=9, y se cumple

9(9+16)=225=152

Si b=3, B=8, A=32, p=(A+B)/2=20, n=(20-16)/2=2, y tenemos

2(2+16)=36=62

Con la función numcuadprod(n) podemos comprobar estos

resultados:

Se observa la coincidencia de valores en los resultados de la función

numcuadprod y la expresión r-2

Page 48: Números y formas - Aprender y divertirse con hoja de …hojamat.es/publicaciones/numform.pdf · Este es uno de los capítulos de las Matemáticas en los que se aprecia ... también

48

Semiprimos pares mayores que 4

Estos números presentan una sola solución, ya que si k=2h, k^2=4h2,

con h primo impar y la única forma de descomponer k en dos factores

adecuados es A=2h2, B=2, con lo que su diferencia será 2*(h2-1). El

paréntesis será múltiplo de 4, ya que h2 es de la forma 4m+1. Así que

las soluciones se formarán así: p=(2h2+2)/2=h2+1 y de ahí, n=(p-

k)/2=(h2+1-2h)/2=

(h-1)2/2=(k-2)2/8

En resumen

𝑛 =(𝑘−2)2

8

Lo puedes comprobar con la tabla de los primeros

semiprimos pares:

Estos razonamientos confirman lo visto con

anterioridad, cuando se afirmaba que 2018

(semiprimo par) sólo presentaba una solución,

508032.

Números de la forma k=2m*p, con m>1 y p primo mayor que 2

En este caso k2=22m*p2

Al descomponer k2 en factores, el primo p puede pertenecer a ambos,

o bien p2 pertenecerá a uno de ellos y al otro no.

Primer caso

Si el primo figura en ambos factores, sean A=2rp y B=2sp, aparecerán

tantos factores como indique 2m, ya que p no añadirá ninguno más.

Como vimos en el apartado de potencias de 2, se producirán m-2

soluciones válidas.

Page 49: Números y formas - Aprender y divertirse con hoja de …hojamat.es/publicaciones/numform.pdf · Este es uno de los capítulos de las Matemáticas en los que se aprecia ... también

49

Segundo caso

Si p2 figura en un factor y en otro no, ambos factores han de ser

múltiplos de 4, luego de todas las potencias de 2 que dividen a 2m,

sean 21, 22, 23,… 22m-2, 22m-1, 22m, habrá que eliminar la primera, que no

es múltiplo de 4, y las dos últimas, que impedirían que fuera múltiplo de

4 el otro factor, luego sólo nos quedan 2m-3 posibilidades.

Sumamos y resultan 3m-5 el número de soluciones que admite

k=2m*p.

Lo comprobamos con un ejemplo. Sea k=80=24*5. Según la fórmula,

deben aparecer 2*4-5=7 soluciones. Lo desarrollamos:

K2=6400

Descomponemos en productos válidos.

Primer caso

Debe entrar el 5 en ambos factores. Las posibilidades son: 320*20 y

160*40, en total 4-2 soluciones, que serían:

A=320, B=20, p=(320+20)/2=170, n=(170-80)/2=45. Compruebo:

45(45+80)=752

A=160 B=40, p=(160+40)/2=100, n=(100-80)/2=10. Se cumple:

10(10+80)=900=302

Segundo caso

El primo 5 sólo figura en un factor. Nos quedarían los productos válidos

1600*4, 800*8, 400*16, 200*32, 100*64, que darían lugar (omitimos los

cálculos) a las soluciones 361, 162, 64, 18 y 1. Serían 5 soluciones, es

decir 2*4-3, según la fórmula.

Resumimos ambos casos y resultan las siete soluciones (3*4-5) que

podemos obtener con la función escuadprod:

Page 50: Números y formas - Aprender y divertirse con hoja de …hojamat.es/publicaciones/numform.pdf · Este es uno de los capítulos de las Matemáticas en los que se aprecia ... también

50

Números impares

En el caso de los impares el cálculo se simplifica. Sólo necesitamos

conocer su descomposición factorial, en la que todos los números

primos serán impares. Todos ellos serán del tipo 4k+1 o 4k+3, pero sus

cuadrados, cuartas o sextas potencias serán todos del tipo 4k+1 y sus

diferencias múltiplos de 4. Si en el emparejamiento un primo del tipo

4k+3 se toma una vez, el otro factor también lo contendrá, y se

compensa el 3 al restar, resultando también un múltiplo de 4.

En este caso k2 tendrá la forma k2=p2tq2ur2vs2w…, con p, q, r, s primos y

t, u, v y w sus exponentes primitivos. Se sabe que entonces su número

de divisores será (1+2t)(1+2u)(1+2v)…Este producto es la función

TAU, la que cuenta divisores.

Todos esos factores se deberán emparejar, ya que sus diferencias

siempre serán múltiplos de 4. Un par tendrá dos factores iguales, y se

deberá eliminar, con lo que nos quedan (TAU(k2)-1)/2 productos

posibles, y soluciones para k.

Ejemplo

Si k=165=3*5*11, según la función numcuadprod, se deberán esperar

13 soluciones, número que coincide con la fórmula que acabamos de

obtener: =(TAU(165^2)-1)/2=(33-1)/2=26/2=13. En efecto, al sacar los

divisores de 1652 podemos obtener trece pares válidos. Reproducimos

los cálculos habituales para sacar todas las soluciones para k=165:

Page 51: Números y formas - Aprender y divertirse con hoja de …hojamat.es/publicaciones/numform.pdf · Este es uno de los capítulos de las Matemáticas en los que se aprecia ... también

51

El último par se desecha por tener los factores idénticos, y nos quedan

13 soluciones en la última columna. Coinciden con las obtenidas

mediante búsqueda ordenada con la función escuadprod ya vista

anteriormente:

Un ejemplo con k conteniendo potencias:

Sea k=33*52*7=4725. Su cuadrado tendrá de exponentes 6, 4 y 2.

Calculamos TAU=(1+6)(1+4)(1+2)=7*5*3=105

El número de soluciones será (TAU-1)/2=(105-1)/2=52, que coincide

con el que nos devuelve numcuadprod:

Page 52: Números y formas - Aprender y divertirse con hoja de …hojamat.es/publicaciones/numform.pdf · Este es uno de los capítulos de las Matemáticas en los que se aprecia ... también

52

Caso general

Ya hemos acumulado experiencia para abordar el caso general, que

resume en cierto modo lo descubierto hasta ahora. Mediante búsqueda

empírica y razonamiento posterior, creemos que podemos acudir a

este procedimiento:

(1) Encontramos la valuación del número k respecto a 2, es decir, el

exponente máximo del 2 contenido en el número dado, llamémosle m.

Esto quiere decir que el número k se descompone en parte par, 2m, y

parte impar. Entonces:

k2=22m*v, siendo v la parte impar.

(2) Hallamos la función TAU de la parte impar v, y aplicamos la fórmula

vista en párrafos anteriores (TAU(k2)-1)/2 , y al resultado le llamaremos

t.

(3) En ese caso, el número de soluciones para nuestra condición de

que sea cuadrado n(n+k) vendrá dada por

N=t*(2m-3)+m-2 si m>2. Si no, lo tratamos como impar.

El primer sumando proviene de combinar todas las soluciones de la

parte impar (la TAU t) con todas las de 2m (vimos que con un solo

primo resultaban 2m-3) y el segundo de la partición que se desechó en

la parte impar por tener dos factores iguales (k*k).

Esto hay que tomarlo como una explicación, no como demostración,

que requeriría un conteo más sistemático de todos los casos. Lo

vemos, por ejemplo, con 336=24*3*7. En este caso m=4 y la parte

impar es 441=32*72, cuya TAU vale (1+2)(1+2)=9 y t=(9-1)/2=4. Por

tanto, según la expresión que hemos presentado, n=4*(2*4-3)+4-

2=4*5+2=22

En efecto, la función numcuadprod nos devuelve 22:

Page 53: Números y formas - Aprender y divertirse con hoja de …hojamat.es/publicaciones/numform.pdf · Este es uno de los capítulos de las Matemáticas en los que se aprecia ... también

53

Según el razonamiento de más arriba, el segundo sumando, 4-2=2

proviene de tomar en la parte impar de 212 el par 21*21. Y así es en

este ejemplo, ya que se corresponderían a los productos

1344*84=21*64*21*4=336*336 y a 672*168=21*32*21*8=336*336

Los restantes 20 productos válidos se corresponderán con todas las

combinaciones válidas de los productos de la parte par con los de la

parte par. Por curiosidad, los copiamos aquí. Ninguno de ellos presenta

el factor común 21 en ambos factores:

28224*4, 14112*8, 9408*12, 7056*16, 4704*24, 4032*28, 3528*32,

3136*36, 2352*48, 2016*56, 1764*64, 1568*72, 1344*84, 1176*96,

1008*112, 784*144, 672*168, 576*196, 504*224 y

448*252.

Las 22 soluciones para n respecto al valor de k=336 las

podemos obtener, ordenadas de menor a mayor, con la

función escuadprod:

Tomemos una al azar, como 847:

847*(847+336)=1002001=10012. Resulta un cuadrado,

luego es una solución válida.

Hemos construido la función numcuadprod2 recogiendo

el algoritmo propuesto. Con ella se puede verificar la

coincidencia de los dos métodos que podemos utilizar, el

de la búsqueda simple (numcuadprod) y el basado en las

consideraciones desarrolladas en este apartado (numcuadprod2)

En esta tabla de datos tomados al azar se observa la equivalencia:

Page 54: Números y formas - Aprender y divertirse con hoja de …hojamat.es/publicaciones/numform.pdf · Este es uno de los capítulos de las Matemáticas en los que se aprecia ... también

54

Casos publicados

Analizamos ahora los casos que se publicaron en Twitter en enero de

2018:

Según nuestro procedimiento, como es impar, buscamos la función

TAU de su cuadrado:

2019=3*673, luego TAU(20192)=(1+2)(1+2)=9, y el número de

soluciones será (9-1)/2=4

Estas cuatro soluciones provendrán de los productos válidos del

cuadrado de 2019:

Hemos seguido lo sugerido en párrafos anteriores:

- Buscar productos cuya diferencia sea múltiplo de 4

– Llamar A y B a los factores

Page 55: Números y formas - Aprender y divertirse con hoja de …hojamat.es/publicaciones/numform.pdf · Este es uno de los capítulos de las Matemáticas en los que se aprecia ... también

55

– Encontrar su promedio p

– Aplicar la fórmula n=(p-k)/2.

Podemos observar la coincidencia entre nuestro cálculo y el publicado.

Aquí 2020=22*5*101. Es el caso en el que el exponente de 2 es 2,

luego el número de casos será 4, pues la parte impar sólo presenta

dos factores y el cuadrado de 2 no incrementa ese número.

Los productos válidos de 20202 son:

Para 2310=2*3*5*7*11 bastará calcular el número de casos de la parte

impar de su cuadrado. Es fácil ver que TAU(23102)=3*3*3*3=81, luego

el número de casos será (81-1)/2=40, tal como se afirma en la

publicación que analizamos.

Reproducir los cuarenta casos es muy pesado. Deberíamos factorizar

el cuadrado de 2310, 5336100 y buscarle todos los divisores:

5336100 2668050 1778700 1334025 … 10 9 7 6 5 4 3 2 1

Page 56: Números y formas - Aprender y divertirse con hoja de …hojamat.es/publicaciones/numform.pdf · Este es uno de los capítulos de las Matemáticas en los que se aprecia ... también

56

Después formaríamos pares con ellos y nos quedaríamos con los que

presentan una diferencia múltiplo de 4. No lo haremos con todos, sólo

con los primeros:

Si siguiéramos el procedimiento con todos los pares de factores

coincidiríamos con lo publicado de forma total.

Problema inverso

Hasta ahora hemos buscado el valor de n que consigue que n(n+k)

sea un cuadrado. Si planteamos el problema inverso, si dado un n ver

si existe un k que cumpla la misma condición, con lo visto en párrafos

anteriores tenemos la respuesta, pues vimos que n(n+3n) es un

cuadrado, luego existe solución para todo n.

También vimos más arriba que 8n, 15n, 24n,…son todas soluciones

para k, luego existen infinitas. La más pequeña suele ser 3n, pero con

muchas excepciones, como puedes ver en la

siguiente tabla, en la que las hemos marcado en

rojo:

Esos casos se producen en los números que

contienen cuadrados. Analizamos la situación:

(a) Si n es libre de cuadrados, será producto de

varios primos, todos elevados a exponente 1.

Por tanto, el cuadrado resultante de n(n+k) ha de

ser múltiplo de los cuadrados de esos primos,

luego el paréntesis también lo será, lo que obliga

a que k sea múltiplo de n. De ahí que la solución

mínima sea 3n.

(b) Si n contiene un cuadrado mayor que 1, será,

por ejemplo, n=r2*s, lo que nos lleva a la

Page 57: Números y formas - Aprender y divertirse con hoja de …hojamat.es/publicaciones/numform.pdf · Este es uno de los capítulos de las Matemáticas en los que se aprecia ... también

57

situación de que r2*s*(r2*s+k) sea un cuadrado. Esto se consigue

dando un valor a k que convierta el paréntesis en otro cuadrado. Para

ello se puede convertir r2 en (r+1)2. Sabemos que la diferencia entre

esos dos cuadrados es 2r+1, luego el valor de k que nos conviene es

(2r+1)*s, ya que r2*s*(r2*s+(2r+1)*s)= r2*s*(r+1)2*s= r2*s2*(r+1)2, luego

hemos conseguido el cuadrado. Lo vemos con algún ejemplo:

N=20=22*5. Hacemos k=(2*2+1)*5=25 y queda

20(20+25)=20*45=900=302

N=24=22*6. Si k=(2*2+1)*6=30, tenemos 24(24+30)=1296=362

No hemos analizado si existe en el caso de números que contienen

cuadrados alguna solución menor que la presentada. Lo importante es

que todos los valores de n admiten infinitas soluciones para k.

Page 58: Números y formas - Aprender y divertirse con hoja de …hojamat.es/publicaciones/numform.pdf · Este es uno de los capítulos de las Matemáticas en los que se aprecia ... también

58

TRIANGULARES

JUGAMOS CON LOS TRIANGULARES

El estudio de cuestiones aritméticas deriva pronto a cálculos

algebraicos, generalmente tediosos, y, en algunos casos, también a

esquemas geométricos. Estos dos caminos, el algebraico y el visual se

complementan perfectamente. Los números figurados, por su propia

definición, son buenos elementos de unión entre ellos. Veamos un

ejemplo con números triangulares:

“Llamamos T(n) al enésimo numero triangular. ¿Qué obtenemos si

sumamos los cuadrados de un número triangular T(n) y de su siguiente

T(n+1)?

Orientación algebraica

(1) Conjetura: Diseñamos una tabla de números triangulares en una

hoja de cálculo y en una columna adjunta calculamos la suma de

cuadrados pedida para todos los casos posibles. Fácilmente se

descubre una ley de formación. No indicamos el resultado, tan sólo que

es un número triangular. ¿Cuál?

(2) Cálculo: Mediante cálculos algebraicos se puede verificar la

conjetura. Basta desarrollar la expresión y comprobar su resultado con

el imaginado. En la imagen tienes un desarrollo efectuado con la

calculadora Wiris. La conjetura está un poco escondida.

Page 59: Números y formas - Aprender y divertirse con hoja de …hojamat.es/publicaciones/numform.pdf · Este es uno de los capítulos de las Matemáticas en los que se aprecia ... también

59

Orientación geométrica

(3) Podemos atrevernos a pensar

que si T(n) es un número triangular,

su cuadrado se podrá representar

por otro número triangular idéntico a

él, pero sus elementos no serán

puntos o bolitas, sino triángulos más

pequeños. Sería “un triángulo de

triángulos”.

Si no acertaste la conjetura por medio del Álgebra, esta imagen te la

sugerirá con más facilidad. Las bolitas rojas corresponden al cuadrado

de T(4) y las verdes al de T(3). Si no sientes una pequeña emoción al

analizarla es que no te gustan de verdad las Matemáticas.

TRIANGULARES ALOJADOS

Si tomamos 40 cubos, los podemos apilar en forma de prisma con

base un triángulo isósceles y rectángulo, o en términos aritméticos, un

número triangular mayor que 1. Excluimos la unidad porque en ese

caso se pierde la forma triangular.

Page 60: Números y formas - Aprender y divertirse con hoja de …hojamat.es/publicaciones/numform.pdf · Este es uno de los capítulos de las Matemáticas en los que se aprecia ... también

60

Este ejemplo es válido porque 40=4*10, y 10 es el cuarto número

triangular.

No todos los números enteros se pueden representar así, pues han de

ser múltiplos de un número triangular y eso no siempre ocurre. Por

ejemplo, el 14, ya que entre sus divisores no figuran 3, 6 ó 10, que son

los triangulares menores que él (recuerda que excluimos el 1). Esto ya

nos divide el conjunto de los números naturales entre los que tienen

divisores triangulares mayores que 1 y los que no.

Los segundos, que no admiten la representación propuesta, son 1, 2,

4, 5, 7, 8, 11, 13, 14, 16, 17, 19, 22, 23, 25, 26, 29, 31, 32, 34, 35, 37,

38… (http://oeis.org/A112886) y les llamaremos libres de

triangulares. Verás que están los primos, algunos semiprimos,

potencias de primos y otros a los que volveremos más adelante.

Parte triangular y parte libre de triángulos

Sabemos que los primeros admiten un divisor triangular pero, como

pueden ser varios, nos quedaremos con el mayor: llamaremos parte

triangular (PTR) de un número al mayor divisor triangular que

posea. Si has leído sobre estos temas, te recordará esto a la parte

cuadrada y la parte libre de un número

(http://hojaynumeros.blogspot.com.es/2011/05/parte-cuadrada-y-parte-

libre.html)

El mayor divisor triangular puede ser 1 o el mismo número, como se

comprueba en la lista de todos ellos (http://oeis.org/A115017):

N 1, 2, 3, 4, 5, 6, 7, 8, 9, 10, 11, 12, 13, 14, 15, 16, 17, 18, 19, 20,

21, 22…

PTR 1, 1, 3, 1, 1, 6, 1, 1, 3, 10, 1, 6, 1, 1, 15, 1, 1, 6, 1, 10, 21,

1…

En ella están los libres de triangulares, que son los que se

corresponden con un 1, como el 4 y el 5, los triangulares, cuya PTR

son ellos mismos, como 6 y 10, y el resto, en el que se tiene una parte

triangular y otra libre ambas mayores que la unidad. Es el caso de 12 o

Page 61: Números y formas - Aprender y divertirse con hoja de …hojamat.es/publicaciones/numform.pdf · Este es uno de los capítulos de las Matemáticas en los que se aprecia ... también

61

40. La parte libre de estos últimos está recogida en

http://oeis.org/A121289

Una idea: dos números con la misma parte libre y partes triangulares

consecutivas formarán un prisma cuadrado. Imagina el prisma de la

primera imagen y su complementario.

Búsqueda de la parte triangular

Un algoritmo simple es el de ir recorriendo los números naturales k,

formar con ellos los triangulares mediante k(k+1)/2 e ir verificando si el

número dado N es múltiplo de alguno. El mayor de todos ellos será la

PTR(N).

Previamente es bueno calcular el orden del máximo triangular que es

menor o igual que N, para acortar el ciclo de búsqueda. Se deja a los

lectores la demostración de que ese orden k se calcula mediante

En hoja de cálculo sería =ENTERO((RAIZ(8*N+1)-1)/2)

Por ejemplo, para N=14534, k=169 y el mayor triangular menor que N,

169*170/2 = 14365. A nosotros nos interesaría el 169, porque entre 2 y

169 estaría el orden del triangular buscado. Todo esto se puede

plasmar en una función:

Public Function partetriang(n) Dim p, i, t, tr p = Int((Sqr(n * 8 + 1) - 1) / 2) ‘Calcula el máximo orden t = 1 For i = 2 To p tr = i * (i + 1) / 2 ‘forma todos los triangulares menores o iguales a n If n / tr = n \ tr Then t = tr ‘si es divisor, toma nota Next i partetriang = t ‘se queda con el mayor End Function

Page 62: Números y formas - Aprender y divertirse con hoja de …hojamat.es/publicaciones/numform.pdf · Este es uno de los capítulos de las Matemáticas en los que se aprecia ... también

62

El algoritmo busca los triangulares entre el menor 3 y el mayor k(k+1)/2

y se va quedando con los divisores. El último encontrado será PTR(A).

Si en lugar de recoger el valor de i*(i+1)/2 hubiéramos ido recogiendo i,

nos hubiera resultado el orden de PTR.

Los tienes en http://oeis.org/A083312

¿Qué números dan alojamiento a un triangular?

Para que N tenga un divisor triangular mayor que 1 se ha de poder

escribir de la forma N=k(k+1)*M/2 con k>1. Esto da lugar a varias

interpretaciones:

(a) N tiene un divisor triangular mayor que 1, si y sólo si 2N posee dos

divisores consecutivos mayores que 1.

Es condición necesaria, pues la expresión de 2N sería 2N=k(k+1)*M

con k>1, con lo que k y k+1 son los divisores pedidos.

Por ejemplo, el triangular 21 divide a N=8883, con lo que el doble

17766=6*7*423 contiene a los consecutivos 6 y 7.

La condición es suficiente: Si 2N posee dos divisores consecutivos h y

h+1 con h>1, estos serán primos entre sí, luego su MCM(h,h+1) será

su producto h(h+1). Como 2N es múltiplo de h y h+1, lo será de su

MCM, es decir de su producto. Por tanto 2N=h(h+1)P y será múltiplo

del triangular h(h+1)/2, ya que uno de los dos h o h+1 es par.

Los números 2N de este tipo los tienes en http://oeis.org/A132895.

Son el doble de un número libre de triángulos.

Sería interesante que pensaras en un algoritmo que descubriera esos

números.

(c) Los semiprimos N=p*q son números libres de triángulos salvo que

uno de sus factores sea 3, o bien q=2p±1

En efecto, si N=p*q con p y q primos, 2N=2pq ha de contener dos

divisores consecutivos. Si p o q fueran iguales a 3, ya se cumpliría,

porque 2N=2*3*k, pero entonces N sería múltiplo de 3.

Page 63: Números y formas - Aprender y divertirse con hoja de …hojamat.es/publicaciones/numform.pdf · Este es uno de los capítulos de las Matemáticas en los que se aprecia ... también

63

Si ni p ni q son iguales a 3, lo serán a 2 o a un primo mayor que 3. Si

por ejemplo p=2 entonces 2N=2*2*q y q se ve obligado a ser 3, con lo

que pasamos al primer caso. Seria múltiplo de 3.

Así que sólo nos queda que N=p*q con p y q primos mayores que 3 y

no son números consecutivos (porque son impares). En ese caso es

claro que 2N=2pq no podría tener divisores consecutivos salvo que

q=2p+1 o bien q=2p-1 (o simétricamente, p=2q+1 o 2q-1). En el primer

caso p sería un primo de Sophie Germain.

Recuerda que los primos de Sophie Germain son aquellos en los que

2*p+1 también es primo: 2, 3, 5, 11,…

(d) Los números primarios (potencias de primos) están libres de

triángulos salvo el caso N=3k

Esta es trivial: Si N=pk con k>1, entonces 2N=2pppp…sólo contendría

divisores consecutivos en el caso 2N=2*3*3*3...

¿Se te ocurren más propiedades? A nosotros por ahora no.

CARNAVAL DE TRIANGULARES

Este capítulo se ha desbordado, como una serpentina que al arrojarla

ya no puede volver a ser rollo. Comenzamos a estudiar variantes de

estudios anteriores y a la multiplicidad de divisores triangulares le

siguió su suma, las coincidencias en esa suma y la reconstrucción de

otro triangular. Por ello comenzamos con un esquema:

Número de divisores triangulares (http://oeis.org/A007862)

Cálculo general

Números con un solo divisor triangular propio (http://oeis.org/A203468)

Suma de divisores triangulares (http://oeis.org/A185027)

Curiosidades

Números con suma de triangulares también triangular

(http://oeis.org/A209309)

Números triangulares con la misma propiedad (http://oeis.org/A209310)

Page 64: Números y formas - Aprender y divertirse con hoja de …hojamat.es/publicaciones/numform.pdf · Este es uno de los capítulos de las Matemáticas en los que se aprecia ... también

64

Otras curiosidades menores

Caso en el que la suma de divisores triangulares es otro divisor

(http://oeis.org/A209311)

Número de divisores triangulares

Vimos en otro apartado que el número 40 posee una parte triangular

igual a 10, que le permite ser representado como un prisma triangular.

Esta representación 40=4*T4 es única (no consideramos el triangular 1,

por lo que no volveremos a citarlo). Ningún otro número triangular

menor o igual que 40 (3, 6, 10, 15, 21, 28 o 36) lo divide salvo el 10.

Esto por lo que se refiere al 40, pero existen otros números que

admiten varias representaciones. El 30 admite cuatro: 30=10*T2 = 5*T3

= 3*T4 = 2*T5

No es difícil contar los divisores triangulares que posee un número N

(al menos, el 1). Basta cambiar el algoritmo que publicamos

anteriormente para que cuente en lugar de quedarse con el mayor

Public Function numdivtriang(n) Dim p, i, t, tr p = Int((Sqr(n * 8 + 1) - 1) / 2) ‘Calcula el máximo orden t = 1 For i = 2 To p tr = i * (i + 1) / 2 ‘forma todos los triangulares menores o iguales a n If n / tr = n \ tr Then t = t+1 ‘si es divisor, incrementa el contador Next i numdivtriang = t ‘se queda con el mayor End Function

Page 65: Números y formas - Aprender y divertirse con hoja de …hojamat.es/publicaciones/numform.pdf · Este es uno de los capítulos de las Matemáticas en los que se aprecia ... también

65

Esta función cuenta el 1, por lo que para 30 dará 5 posibilidades y para

40 sólo 2. En la siguiente tabla parcial lograda con hoja de cálculo lo

puedes comprobar

30 5 31 1 32 1 33 2 34 1 35 1 36 4 37 1 38 1 39 2 40 2

Este resultado lo tienes en http://oeis.org/A007862 y es interesante leer

los comentarios que se incluyen.

Números con un solo divisor triangular propio mayor que 1

El caso del 40 no es único. Hay muchos números que sólo pueden

representarse de una sola forma como un prisma triangular con base y

altura mayores que uno (para evitar trivialidades). Son estos:

6, 9, 15, 20, 21, 27, 33, 39, 40, 50, 51, 56, 57, 69, 70, 80, 81, 87, 93,

99, 100, 111, 112, 117, 123, 129, 130, 141, 153, 159, 160, 170, 171,

177, 182, 183, 190, 196, 200, 201, 207, 213, 219, 224, 230, 237, 243…

Los hemos publicado en http://oeis.org/A203468

Prueba con cualquiera de ellos, el 182=2*7*13. Puedes usar la

propiedad que vimos de que su doble ha de tener dos divisores

consecutivos. 364=2*2*7*13 y su conjunto de divisores es {364, 182,

91, 52, 28, 26, 14, 13, 7, 4, 2, 1}. Los únicos divisores consecutivos son

13 y 14, que dan lugar a un único divisor triangular de 182, el 91.

Por cierto, su consecutivo 183 presenta la misma situación: su único

divisor triangular es el 3. No es el único par de consecutivos contenido

en la sucesión. Por ejemplo, tenemos 170 y 171.

Page 66: Números y formas - Aprender y divertirse con hoja de …hojamat.es/publicaciones/numform.pdf · Este es uno de los capítulos de las Matemáticas en los que se aprecia ... también

66

Dentro de esta sucesión figuran números triangulares. Todos ellos

presentarán tres divisores triangulares: ellos, un divisor propio y la

unidad. Así, 351 tiene como únicos divisores triangulares 1, 3 y el

propio 351.

Suma de divisores triangulares

Además de considerar la suma de todos los divisores de un número,

puede resultar curioso sumar sólo los de un tipo. Por ejemplo, el

número 720 tiene como suma de divisores 2418, pero si sólo

consideramos los que son cuadrados, sumarían 210=144+36+

16+9+4+1 y con los triangulares 236=120+45+36+15+10+6+3+1. Se

pueden considerar otros tipos de divisores: los pares, los oblongos…

Un algoritmo un poco burdo, pero que funciona, es el de recorrer todos

los posibles divisores y someter a cada uno a una condición antes de

incorporarlo a la suma. Aquí tienes el que hemos usado para

cuadrados y triangulares:

Public Function sumadiv(nume, tipo) 'tipos '0 da todos los divisores '1 los cuadrados '2 los triangulares Dim i, s s = 0 For i = 1 To nume If esmultiplo(nume, i) Then If tipo = 0 Then s = s + i If tipo = 1 And escuad(i) Then s = s + i If tipo = 2 And estriangular(i) Then s = s + i End If Next i sumadiv = s End Function Con un algoritmo similar hemos publicado en OEIS la función que

recoge la suma de los divisores triangulares de los primeros números

naturales:

Page 67: Números y formas - Aprender y divertirse con hoja de …hojamat.es/publicaciones/numform.pdf · Este es uno de los capítulos de las Matemáticas en los que se aprecia ... también

67

1, 1, 4, 1, 1, 10, 1, 1, 4, 11, 1, 10, 1, 1, 19, 1, 1, 10, 1, 11, 25, 1, 1, 10,

1, 1, 4, 29, 1, 35, 1, 1, 4, 1, 1, 46, 1, 1, 4, 11, 1, 31, 1, 1, 64, 1, 1, 10, 1,

11, 4, 1, 1, 10, 56, 29, 4, 1, 1, 35, 1, 1, 25, 1, 1, 76…

(http://oeis.org/A185027)

Curiosidades

Esta sucesión da lugar a varias curiosidades:

La suma de triangulares puede ser triangular. Excluimos el caso en

que sea igual a 1 por trivial. Estos son los números que lo cumplen:

6, 12, 18, 24, 48, 54, 96, 102, 110, 114, 138, 162, 174, 186, 192, 204,

220, 222, 228, 246, 258, 282, 315, 318, 348, 354, 364, 366, 372, 384,

402, 414, 426, 438, 440, 444, 456, 474, 486, 492, 498, 516, 522, 534,

550…

También la acabamos de publicar (http://oeis.org/A209309)

Por ejemplo, 444 tiene como divisores triangulares 6, 3 y 1, y su suma

es 10 que es triangular. Más complejo sería el caso de 1320, cuyos

divisores triangulares, 120, 66, 55, 15, 10, 6, 3 y 1 suman 276, que es

triangular igual a 23*24/2.

Similares a esta, pero menos exigentes, son estas condiciones:

(1) La suma de los divisores ordinarios es triangular

1, 2, 5, 8, 12, 22, 36, 45, 54, 56, 87, 95, 98, 104, 116, 152, 160, 200,…

A045746

(2) La que es triangular es la suma de las partes alícuotas, y

mayor que 1

2,4,6,14,16,18,24,25,28,33,36,51,54,66,91,112

(3) Números triangulares en los que la suma de sus divisores

propios es también triangular

1, 3, 6, 28, 36, 66, 91, 231, 496, 8128, 14196, 15225, 129795,

491536… (A083675)

Page 68: Números y formas - Aprender y divertirse con hoja de …hojamat.es/publicaciones/numform.pdf · Este es uno de los capítulos de las Matemáticas en los que se aprecia ... también

68

(4) Números triangulares cuya suma de divisores es también

triangular

1, 36, 45, 23220, 105111, 135460, 2492028, 5286126, 6604795,

14308575, 45025305, 50516326, 54742416, 99017628, 108125865,

152486916 (A083674)

Ahora viene la nuestra, la más exigente:

Números triangulares cuya suma de divisores triangulares es

mayor que 1 y triangular

6, 4186, 32131, 52975, 78210, 111628, 237016, 247456, 584821,

750925, 1464616, 3649051, 5791906, 11297881, 16082956,

24650731, 27243271, 38618866, 46585378, 51546781, 56026405,

76923406, 89880528, 96070591…(http://oeis.org/A209310)

El estudio del código PARI de esta sucesión te enseñará técnicas

útiles:

(PARI) istriangular(n)=issquare(8*n+1) {t=0; for(n=1, 10^8, if(istriangular(n), k=sumdiv(n, d, istriangular(d)*d) ; if(istriangular(k)&&k>>1, t+=1; write("b209310.txt", t, " ", n))))}

Y por último, para no cansar (si es que has llegado hasta aquí), la

última curiosidad

Números en los que la suma de divisores triangulares es mayor

que 1 y divisor del número

285, 1302, 1425, 1820, 2508, 3640, 3720, 4845, 4956, 5016, 5415,

7125, 7280, 9100, 9114, 9912, 11685, 12255, 12740, 14508, 15105,

16815, 17385, 18200, 19095, 19824, 20235, 20805, 22134, 22515,

23655, 23660, 24021, 24738…

http://oeis.org/A209311

Aquí tienes dos ejemplos:

285.-Divisores triangulares:1, 3 y 15 y su suma, 19, es divisor de 285

Page 69: Números y formas - Aprender y divertirse con hoja de …hojamat.es/publicaciones/numform.pdf · Este es uno de los capítulos de las Matemáticas en los que se aprecia ... también

69

1302.- Divisores triangulares: 21 + 6 + 3 + 1 = 31 que es divisor de 1302.

Código PARI

istriangular(n)=issquare(8*n+1) {t=0; for(n=1, 10^7, k=sumdiv(n, d, istriangular(d)*d); if(n/k==n\k&&k>>1, t+=1; print(t, " ", n)))}

Nos queda algo en el tintero, porque en esta última el cociente puede

ser también triangular, pero esto queda para otra ocasión.

TRIANGULARES DE LADO PAR

Los números triangulares 3, 10, 21, 36,…son aquellos cuyo número de

orden es par: 3=T(2)=2*3/2; 10=T(4)=4*5/2; 21=T(6)=6*7/2,…Si

aplicamos la expresión algebraica de un número triangular, la de estos

será

T(2n)=2n(2n+1)/2=n(2n+1)=2n2+n

Los podemos representar como formados por filas de triángulos de 3

elementos separados por otros elementos aislados. En la imagen

hemos representado el 36, es decir T(8)

Observa que está formado por 10 triángulos de tres elementos y 6

puntos aislados. Nos sugiere que un número triangular de orden par

equivale a triangular de orden mitad multiplicado por 3 más su

triangular anterior, es decir:

T(2n)=3T(n)+T(n-1)

Page 70: Números y formas - Aprender y divertirse con hoja de …hojamat.es/publicaciones/numform.pdf · Este es uno de los capítulos de las Matemáticas en los que se aprecia ... también

70

Es fácil demostrarlo por inducción: T(2)=3*T(1)+T(0)=3*1+0=3;

T(4)=3*T(2)+T(1)=3*3+1=10…

Probemos con T(2(n+1))=T(2n)+(2n+1)+(2n+2) por definición de

número triangular. Si aceptamos la hipótesis para n, tendremos:

T(2(n+1))=3*T(n)+T(n-1)+(n+1+n+1+n+1)+n=3*T(n)+3*(n+1)+T(n-

1)+n=3*T(n+1)+T(n), luego la hipótesis se cumple para n+1.

La fórmula T(2n)=3T(n)+T(n-1) es válida

Adaptamos una demostración visual contenida en

http://math.berkeley.edu/~rbayer/09su-55/handouts/ProofByPicture-

printable.pdf

Así se ve mejor la relación.

En realidad, estos números son los triangulares que no pueden ser

hexagonales. Se sabe que todo hexagonal es triangular, porque su

expresión es H(n)=n(2n-1)=2n(2n-1)/2=T(2n-1), pero el número de

orden del triangular es 2n-1, impar, luego los que no son hexagonales

formarán la sucesión que estamos estudiando: 3, 10, 21, 36,…, que

está contenida en http://oeis.org/A014105

Expresión como resta entre una suma de pares y otra de impares

En la página OEIS enlazada se destacan estas relaciones:

3=4-1

10=6+8-1-3

21=8+10+12-1-3-5

Page 71: Números y formas - Aprender y divertirse con hoja de …hojamat.es/publicaciones/numform.pdf · Este es uno de los capítulos de las Matemáticas en los que se aprecia ... también

71

36=10+12+14+16-1-3-5-7

No se justifican, y esto es una invitación a que lo hagamos nosotros.

En primer lugar generalizamos. Llamamos a nuestra sucesión TT(n)

TT(n)=T(2n)=SP(2(n+1),n)-SI(1,n)

Con SP(2(n+1),n) deseamos expresar que se toman n números pares

a partir de 2(n+1) y con SI(1,n) que se suman los primeros n impares.

Lo intentamos demostrar por inducción:

TT(n+1)=TT(n)+2n+1+2n+2, como ya sabemos por los párrafos

anteriores. Si usamos la hipótesis para n queda:

TT(n+1)=2(n+1)+2(n+2)+…+2(2n)-1-3-5-7…- (2n-1)+2n+1+2n+2

Para construir la nueva suma de pares hay que añadir

2(2n+1)+2(2n+2) y eliminar 2(n+1). La diferencia es 4n+2+4n+4-2n-

2=6n+4, que ha de salir de los nuevos sumandos 2n+1+2n+2=4n+3,

que equivalen a 6n+4-(2n+1), siendo el paréntesis el nuevo impar que

habría que restar, luego la estructura de la fórmula se mantiene y es

correcta.

Usamos el álgebra

TT(n)=T(2n)=n(2n+1)=2n2+n

SP(2(n+1),n)=(2(n+1)+2(2n))*n/2=3n2+n

SI(1,n)=n2 como es sabido.

Por tanto, se verifica la diferencia.

Demostración visual

Ahí te la dejamos para el caso de 36. Analízala e intenta reproducirla

para otros casos:

Page 72: Números y formas - Aprender y divertirse con hoja de …hojamat.es/publicaciones/numform.pdf · Este es uno de los capítulos de las Matemáticas en los que se aprecia ... también

72

Esta construcción sólo es posible porque el triángulo es de orden par.

Otros desarrollos

Se cumple que TT(n)=T(2n)=3+7+11+15+…(4n-1), es decir, que es la

suma de impares tomados de 4 en 4 a partir de 3. Si sabes verlo, en la

anterior imagen se muestra esa suma con claridad. Puedes justificarlo

algebraicamente:

3+7+11+15+…(4n-1)=(3+4n-1)*n/2=(4n+2)*n/2=n(2n+1)=TT(n)

Este desarrollo se puede escribir así: TT(n)=22-12+42-32+62-52+82-72…,

que es una forma elegante de terminar este tema..

Page 73: Números y formas - Aprender y divertirse con hoja de …hojamat.es/publicaciones/numform.pdf · Este es uno de los capítulos de las Matemáticas en los que se aprecia ... también

73

CUADRADOS Y TRIANGULARES

CUADRADOS VECIN OS DE TRIANGULARES

Sabemos que hay números que son triangulares y cuadrados a la vez:

1, 36, 1225, 41616,…, pero, ¿existirán pares de números consecutivos

tales que uno sea triangular y el otro cuadrado?

Dejamos como propuesta encontrar pares de números consecutivos

tales que uno sea triangular y el otro cuadrado mediante el método de

formar una columna de triangulares en una hoja de cálculo, y junto a

esa columna formar otra sumando o restando una unidad a los

anteriores, y finalmente analizando que resulte un cuadrado perfecto.

Como este método lo hemos desarrollado varias veces en este blog lo

dejamos así, como propuesta.

Si sabes escribir código de macros en Calc o en Excel, puedes usar

estas dos funciones y una macro de búsqueda (son válidas para

ambas hojas de cálculo)

Public Function escuadrado(n) As Boolean

If n = Int(Sqr(n)) ^ 2 Then escuadrado = True Else escuadrado =

False

End Function

Public Function estriangular(n) As Boolean

If 8 * n + 1 = Int(Sqr(8 * n + 1)) ^ 2 Then estriangular = True Else

estriangular = False

End Function

Sub busqueda()

For i = 1 To 1000000

If escuadrado(i) And estriangular(i + 1) Then MsgBox (i)

Next i

End Sub

Page 74: Números y formas - Aprender y divertirse con hoja de …hojamat.es/publicaciones/numform.pdf · Este es uno de los capítulos de las Matemáticas en los que se aprecia ... también

74

Tal como está escrito el código, encontrará números cuadrados tales

que al sumarles una unidad se convierten en triangulares. Una

búsqueda del 1 a 1000000 obtendríamos los pares:

Cuadrado más uno igual a triangular

10 9

325 324

11026 11025

374545 374544

Para comprobar consulta http://oeis.org/A164055

En lenguaje PARI:

isinteger(n)=(n==truncate(n))

isquare(n)= { local(f,m,p=0); if(n==1,p=1,f=factor(n); m=gcd(f[, 2]);

if(isinteger(m/2),p=1));return(p) }

{ for (n=2, 100, a=n*n;for(k=1,a/2,if (isquare(a-k) && isquare(a-2*k),

write("final.txt",a," ",a-k," ",a-2*k)))) }

{istriang(n)=isquare(8*n+1)}

{for(n=2,10^7,if(isquare(n)&&istriang(n+1),print1(n," "))) }

Si sustituimos la expresión i+1 en el código por i-1, obtendremos:

Triangulares más uno cuadrados

0 1

3 4

15 16

120 121

528 529

4095 4096

17955 17956

139128 139129

609960 609961

Como el 0 no es triangular, desechamos la primera solución. Consulta

http://oeis.org/A006454

Page 75: Números y formas - Aprender y divertirse con hoja de …hojamat.es/publicaciones/numform.pdf · Este es uno de los capítulos de las Matemáticas en los que se aprecia ... también

75

Pero esto es fiarse demasiado de la máquina. Para encontrar

triangulares y cuadrados consecutivos podemos intentar usar las

técnicas algebraicas.

Acudimos a la fórmula de los números triangulares para plantear la

igualdad pedida

212

)1(k

nn

en la que el doble signo de 1 se justifica porque el triangular puede ser

mayor o menor que el cuadrado. Si desarrollamos y exigimos que el

discriminante de la ecuación sea cuadrado perfecto, llegaremos a la

ecuación de Pell x2-8y2=-7 en el primer caso y a la x2-8y2=9 en el

segundo (intenta desarrollarlo así y te resultarán esas dos ecuaciones)

Podemos llegar a las mismas ecuaciones recordando que un número

triangular multiplicado por 8 y añadiéndole una unidad se convierte en

cuadrado perfecto. De esta forma llegamos a la ecuación de Pell de

forma mucho más rápida.

m2− 1

8± 1= k

2

Con el signo + del 1 llegamos a m2-8k2 = -7 y con el menos a

m2-8k2 = 9

Así que el problema desemboca en la resolución de las ecuaciones x2-

8y2=-7 ; x2-8y2=9

No todas las ecuaciones de Pell tienen solución. Estas dos sí las

tienen, como se puede ver por tanteo: 52-8*22=-7 ; 92-8*32=9, que nos

dan las primeras soluciones del problema:

Si x=5 y=4 obtenemos el número triangular 3 y el cuadrado 4 que son

consecutivos

Si x=9 y=3 aparecerán el triangular 10 y el cuadrado 9, consecutivos,

pero con el triangular mayor.

Page 76: Números y formas - Aprender y divertirse con hoja de …hojamat.es/publicaciones/numform.pdf · Este es uno de los capítulos de las Matemáticas en los que se aprecia ... también

76

Estas soluciones coinciden con las primeras obtenidas con la hoja de

cálculo.

Pero ¿y las demás soluciones?

Si la ecuación de Pell hubiera sido x2-8y2=1, una solución trivial sería

X0=3, Y0=1. Nos podemos aprovechar de esto de la siguiente forma:

La identidad 32-8*12 = 1 la podemos escribir como un producto en el

anillo Q(√8):

Igualmente, la ecuación x2-8y2=-7 la podemos escribir como

Si ahora multiplicamos ambas ecuaciones obtendremos:

y agrupando términos

o bien

De esta forma hemos obtenido una fórmula de recurrencia para las

siguientes soluciones:

xn =3xn-1+8yn-1 yn=xn-1+3yn-1

Esta fórmula valdría igualmente para el caso x2-8y2=9

Lo hemos desarrollado en una hoja de cálculo:

Page 77: Números y formas - Aprender y divertirse con hoja de …hojamat.es/publicaciones/numform.pdf · Este es uno de los capítulos de las Matemáticas en los que se aprecia ... también

77

Para el primer caso:

Para el segundo:

Esta segunda tabla coincide con la obtenida con hoja de cálculo

anteriormente:

10 9

325 324

11026 11025

374545 374544

pero en el primer caso ¡sólo hemos obtenido la mitad!

Con la hoja salían más:

0 1

3 4

15 16

120 121

528 529

4095 4096

17955 17956

Page 78: Números y formas - Aprender y divertirse con hoja de …hojamat.es/publicaciones/numform.pdf · Este es uno de los capítulos de las Matemáticas en los que se aprecia ... también

78

139128 139129

609960 609961

La causa de que no hayamos obtenido todas las soluciones está en la

agrupación de términos que efectuamos:

Igualmente, la ecuación x2-8y2=-7 la podemos escribir como

Si ahora multiplicamos ambas ecuaciones obtendremos:

y agrupando términos

En esta agrupación de la ecuación (1) habíamos multiplicado el primer

factor por el tercero y el segundo por el cuarto para que nos diera la

suma por diferencia igual a la diferencia de cuadrados, pero si

llegamos a multiplicar primero por cuarto y segundo por tercero,

habríamos llegado a

y de ahí obtendríamos las soluciones que faltan. En el segundo caso

coincidían las dos posibilidades y por eso no echamos a faltar ninguna

solución.

Page 79: Números y formas - Aprender y divertirse con hoja de …hojamat.es/publicaciones/numform.pdf · Este es uno de los capítulos de las Matemáticas en los que se aprecia ... también

79

Enseñanza: Hay que agotar todas las posibilidades y no alegrarnos

demasiado con el éxito obtenido.

TRIANGULARES Y CUADRADOS CON

PIEZAS

Quien ha entrado en el mundo de la programación elemental sabe qué

es la operación de concatenar cadenas (“strings”): situar sus

caracteres uno detrás del otro. Si lo representamos por &, equivaldría a

que “Pablo “&”Pérez”= “Pablo Pérez”. En las hojas de cálculo

disponemos de la función CONCATENAR, que une varios textos de

celdas en uno =CONCATENAR(A12;B22;G1).

Más difícil es concatenar números naturales, de forma que el resultado

sea otro verdadero número en el que cada cifra tenga su valor relativo.

Una forma se basa en esta función CONCATENAR. Para ello debemos

convertir los números en cadenas, con la función TEXTO, después,

concatenarlos, y finalmente, usar la función VALOR para devolverles el

carácter numérico. Tiene un inconveniente, y es que TEXTO ha de ir

acompañado de un formato, y esto lo complica todo. En PARI no existe

ese problema, por lo que puedes definir la concatenación entre

números mediante

concatint(a,b)=eval(concat(Str(a),Str(b)))

Un método más matemático, y es el que adoptaremos para la hoja de

cálculo es el de multiplicar el número de la izquierda por una potencia

de 10 adecuada y sumar luego el de la derecha. Así, concatenar 255

con 182 equivaldría al número 255*10^3+182=255182

¿Qué exponente ha de tener esa potencia de 10? El número de

cifras del que está a la derecha. Para encontrar ese número podemos

usar el logaritmo decimal, de esta forma: =ENTERO(LOG(N;10))+1.

Por tanto, una concatenación numérica vendría dada por la fórmula

CONCAT(A;B)=A*10^((ENTERO(LOG(B;10))+1))+B

Page 80: Números y formas - Aprender y divertirse con hoja de …hojamat.es/publicaciones/numform.pdf · Este es uno de los capítulos de las Matemáticas en los que se aprecia ... también

80

Esta fórmula fallaría para B=0, por lo que habría que retocarla con un

condicional, pero no lo haremos. Basta que se sepa que existe esa

función numérica, y en la práctica usaremos una rutina en Basic.

Se producen muchas curiosidades cuando concatenamos números

naturales. Veamos algunas. Es evidente que nos movemos en

cuestiones curiosas y no teóricas. Comenzamos generando números

triangulares. Ya veremos más adelante otros casos.

Estudiamos algunas concatenaciones concretas:

Producir triangulares

Intentaremos concatenar un número n consigo mismo o con otros

relacionados con él a fin de conseguir un número triangular. Por

ejemplo, 426 concatenado consigo mismo produce el triangular

426426. Para entender mejor lo que sigue, recuerda que todo número

triangular se puede expresar como N(N+1)/2, es decir, la mitad de un

oblongo N(N+1). En este caso, 426426=923*924/2

Triangular concatenando n//n

En primer lugar probaremos a concatenar un número consigo mismo

para producir un triangular. Ya están publicados en

http://oeis.org/A068899

55, 66, 5050, 5151, 203203, 255255, 426426, 500500, 501501,

581581, 828828, 930930, 39653965, 50005000, 50015001, 61566156,

3347133471, 5000050000, 5000150001, 6983669836, 220028220028,

500000500000, 500001500001…

Si te llaman la atención los ejemplos del tipo 500…500 y

500…1500…1, piensa que no son nada extraordinarios:

500500=1000*1001/2, que es un triangular

50015001=10001*10002/2 es otro. Investiga casos similares.

Triangular concatenando 2n//n

De esta forma se generan los siguientes:

Page 81: Números y formas - Aprender y divertirse con hoja de …hojamat.es/publicaciones/numform.pdf · Este es uno de los capítulos de las Matemáticas en los que se aprecia ... también

81

21, 105, 2211, 9045, 222111, 306153, 742371, 890445, 1050525,

22221111, 88904445, 107905395, 173808690, 2222211111,

8889044445, 12141260706, 15754278771, 222222111111,

888890444445, 22222221111111, 36734701836735,

65306123265306, 88888904444445, 163718828185941…

¿Es siempre triangular 2222…1111…? Sí, porque sus dobles se

descomponen como 4444…2222=6666..6*6666…7, es decir, números

oblongos formados por productos de números consecutivos. En lo que

sigue acudiremos varias veces al hecho de que el doble de un

triangular es un oblongo, k(k+1).

Se puede demostrar que cada vez que se añade la cifra a los factores

aparecen 4444..2222. Lo razonamos con 666*667=444222 pero para

más cifras se comprende igual: En efecto, si 666*667=444222, al

añadir una cifra tenemos:

6666*6667=(6000+666)(6000+667)=36000000+6000*1333+666*667=

43998000+444222=44442222.

Observa que si aumentamos las cifras, 1333 se convertiría en

133….33 y el sumando final 43998000 en 4399…8000… con lo que el

efecto de reconstruir 44444 y 2222 sería el mismo.

Algo similar ocurre con la subsucesión 9045, 890445,

88904445,…engendrada por los oblongos 134*135, 1334*1335,

13334*13335,…Son casualidades que ocurren al dividir las potencias

de 10 en tercios o en sextos.

Si deseas reproducir los resultados puedes usar este código en PARI

concatint(a,b)=eval(concat(Str(a),Str(b)))

istriang(x)=issquare(8*x+1)

{for(n=1,10^5,a=concatint(2*n,n);if(istriang(a),print(a)))}

La función istriang usa la propiedad de que ocho veces un triangular

más la unidad es un número cuadrado

(fácil: n(n+1)/2*8+1=4n2+4n+1=(2n+1)2, un cuadrado)

Page 82: Números y formas - Aprender y divertirse con hoja de …hojamat.es/publicaciones/numform.pdf · Este es uno de los capítulos de las Matemáticas en los que se aprecia ... también

82

Hemos publicado esta sucesión en https://oeis.org/A226742

Concatenación inversa n//2n

¿Y si concatenáramos en sentido contrario, primero el número y

después su doble? Pues, aunque menos llamativo, también se

construyen triangulares. Son estos:

36, 1326, 2346, 3570, 125250, 223446, 12502500, 22234446,

1250025000, 2066441328, 2222344446, 2383847676, 3673573470,

125000250000, 222223444446, 5794481158896, 12500002500000,

12857132571426, 22222234444446, 49293309858660…

Intenta razonar la aparición de estos números, con un método similar al

usado en el anterior caso: 36, 2346, 223446, 22234446,… es porque

sus dobles se descomponen como 666…68*666…69. Observa

también esta otra subsucesión: 125250, 12502500, 12500025000, que

provienen de los oblongos 500*501, 5000*5001,…¿Y el resto? Te lo

dejamos por si encuentras una pauta.

Código PARI para este caso:

concatint(a,b)=eval(concat(Str(a),Str(b)))

istriang(x)=issquare(8*x+1)

{for(n=1,10^7,a=concatint(n,2*n);if(istriang(a),print(a)))}

Hemos publicado esta sucesión en https://oeis.org/A226772

Concatenación n//n+1

También se producen números triangulares:

45, 78, 4950, 5253, 295296, 369370, 415416, 499500, 502503,

594595, 652653, 760761, 22542255, 49995000, 50025003, 88278828,

1033010331, 1487714878, 4999950000, 5000250003, 490150490151,

499999500000, 500002500003, 509949509950, 33471093347110,

49999995000000, 50000025000003, 69834706983471…

Page 83: Números y formas - Aprender y divertirse con hoja de …hojamat.es/publicaciones/numform.pdf · Este es uno de los capítulos de las Matemáticas en los que se aprecia ... también

83

Se destaca el subconjunto 45, 4950, 499500,….y es porque sus dobles

son 9999…*10000… y también los 5253, 502503, 50035003…¿En qué

se parecen entre sí?

Puedes reproducirlos con este código PARI

concatint(a,b)=eval(concat(Str(a),Str(b)))

istriang(x)=issquare(8*x+1)

{for(n=1,10^7,a=concatint(n,n+1);if(istriang(a),print(a)))}

Hemos publicado esta sucesión en https://oeis.org/A226788

Con n+1//n resultan verdaderos monstruos

21, 26519722651971, 33388573338856, 69954026995401,

80863378086336,…

A partir de este último no se han podido encontrar más para n<10^10,

o resultados menores que 10^20. Quizás con una herramienta o equipo

más potentes se pueda hallar alguno más fuera de esa acotación. Los

lectores quedáis invitados a intentarlo. Podéis usar este código PARI

concatint(a,b)=eval(concat(Str(a),Str(b)))

istriang(x)=issquare(8*x+1)

{for(n=1,10^7,a=concatint(n,n+1);if(istriang(a),print(a)))}

Si disponéis de MATHEMATICA también bastará adaptar este otro,

añadido por T.D. Noe a A226789:

TriangularQ[n_] := IntegerQ[Sqrt[1 + 8*n]]; t = {}; Do[s =

FromDigits[Join[IntegerDigits[n+1], IntegerDigits[n]]];

If[TriangularQ[s], AppendTo[t, s]], {n, 100000}]; t (* T. D. Noe, Jun

18 2013 *)

No vamos seguir las concatenaciones de este tipo. Las dejamos para

quien le apetezca encontrar más ejemplos curiosos. Sí podíamos

seguir jugando con las cifras, pero con otras estructuras. Seguimos

buscando triangulares.

Page 84: Números y formas - Aprender y divertirse con hoja de …hojamat.es/publicaciones/numform.pdf · Este es uno de los capítulos de las Matemáticas en los que se aprecia ... también

84

Otras concatenaciones para triangulares

No hemos intentado todavía concatenar un número con su reverso. Por

ejemplo, 59 con 95 forman el triangular 5995. Intentamos una

búsqueda por ahí. Antes de presentar resultados hay que advertir que

los terminados en 0, como 90, producen resultados ambiguos, en este

caso 990. Por eso restringiremos la búsqueda a números no múltiplos

de 10. En ese caso resultan estas soluciones:

55, 66, 5995, 8778, 617716, 828828, 35133153, 61477416,

1264114621,…, que forman una subsucesión de

http://oeis.org/A003098

Un resultado curioso es si concatenamos un número n por la izquierda

con 10*n, porque en ese caso resulta un número duplicado con un cero

en el centro. Hemos encontrado estos, que resultan muy vistosos:

41041, 66066, 165301653, 56661056661, 3719010371901,

276816602768166, 13776656013776656, 28265441028265441,

41631576041631576, 47337278047337278, 55666611055666611,

82189446082189446, 91836735091836735, 1185252600118525260,

1960592100196059210…

Evitamos seguir insistiendo en el tema. Con estos ejemplos nuestros

lectores pueden abordar otras búsquedas.

NÚMEROS ESPECIALES QUE SON UN

PRODUCTO ESPECIAL

Este apartado y los siguientes tienen el doble objetivo de presentar

unas curiosidades numéricas (algo intrascendentes) y analizar cómo

organizar búsquedas de cierto tipo intentando dar con el algoritmo más

Page 85: Números y formas - Aprender y divertirse con hoja de …hojamat.es/publicaciones/numform.pdf · Este es uno de los capítulos de las Matemáticas en los que se aprecia ... también

85

rápido posible, ya que llegan fácilmente al orden de 10^7.

Comenzamos con un ejemplo:

Números triangulares que son producto de triangulares

Muchos números triangulares son producto de otros dos también

triangulares. Por ejemplo, 45=15*3, 210=21*10, todos triangulares. Los

tienes publicados en https://oeis.org/A188630

36, 45, 210, 630, 780, 990, 1540, 2850, 3570, 4095, 4851, 8778,

11781, 15400, 17955, 19110, 21528, 25200,…

Esta búsqueda está resuelta, pero imagina que la deseamos

reproducir. No es fácil, porque para cada número natural deberíamos

buscar lo siguiente:

Ver si ese número N es triangular

En caso afirmativo, recorrer todos sus divisores d.

Para cada uno de ellos, investigar si tanto d como N/d son ambos

triangulares, y en caso afirmativo, parar la búsqueda para ese valor N

y proseguir con N+1.

Es fácil darse cuenta de que se perderá mucho tiempo recorriendo

números de uno en uno, que no son triangulares o bien que no poseen

muchos divisores triangulares (o ninguno). Con búsquedas de ese

tipo, llamemos “ingenuas”, nuestro ordenador se pasaba minutos y

minutos cuando llegaba a números grandes. Una solución es

encaminar la búsqueda para que, hasta donde sea posible, sólo se

recorran los números de cierto tipo. En el caso de triangulares,

cuadrados, oblongos o primos, es posible realizar ese filtro. Lo

concretamos:

Page 86: Números y formas - Aprender y divertirse con hoja de …hojamat.es/publicaciones/numform.pdf · Este es uno de los capítulos de las Matemáticas en los que se aprecia ... también

86

Generación de triangulares

Los números que usaremos, salvo los primos, se pueden engendrar a

partir de los precedentes. Comenzaremos por explicar distintas formas

de generar algunos tipos de números, y así ya las tenemos preparadas

para cuestiones posteriores.

En el caso de los triangulares manejamos dos variables: Número N e

incremento D. Comenzamos haciendo N=1 (primer triangular) e

incremento D=2, para que 1+2 genere el siguiente triangular, el 3.

Luego, en cada paso, N se convierte en N+D y D en D+1. Así funciona,

ya que los números triangulares se forman añadiendo una fila con un

elemento más.

Observa estos valores, generados con hoja de cálculo:

Recordamos: Los triangulares se generan tomando incrementos

con una unidad más en cada paso. Ya utilizaremos esto más

adelante.

Número N Triangular Incremento D

1 2

3 3

Los triangulares 6 4 Los incrementos crecen

se incrementan 10 5 en una unidad en cada paso

en D 15 6

21 7

28 8

36 9

Page 87: Números y formas - Aprender y divertirse con hoja de …hojamat.es/publicaciones/numform.pdf · Este es uno de los capítulos de las Matemáticas en los que se aprecia ... también

87

Generación de cuadrados

Aquí tenemos dos soluciones, ambas prácticas, según el contexto,

para recorrer sólo números cuadrados: la primera es trivial, declarar

una variable k y luego usar k2 en los cálculos. Está bien, pero a veces

es lenta y no admite con facilidad ciertas acotaciones. La segunda es

similar a la de los triangulares, pero incrementando D en D+2, en dos

unidades en lugar de en una.

Observa el esquema:

Para quienes conozcáis estas propiedades, esto parecerá trivial, pero

no está mal recordarlo, porque más adelante dará velocidad a nuestras

búsquedas.

Generación de oblongos

Un número es oblongo cuando tiene la forma N=k(k+1), es decir, doble

de un triangular. Es fácil ver que el siguiente oblongo será (k+1)(k+2).

Su diferencia (k+1)(k+2)-k(k+1) = 2(k+1), es decir, el doble del mayor

en el producto, luego el incremento adecuado será par y crecerá de 2

Número N cuadrado Incremento D

1 3

4 5

Los cuadrados 9 7 Los incrementos crecen

se incrementan 16 9 en dos unidades en cada paso

en D 25 11

36 13

49 15

64 17

Page 88: Números y formas - Aprender y divertirse con hoja de …hojamat.es/publicaciones/numform.pdf · Este es uno de los capítulos de las Matemáticas en los que se aprecia ... también

88

en 2. Esto nos permite generar oblongos: comenzamos con N=2 y

D=4, Así generamos los siguientes: N=2+2*2=6, N=6+2*3=12,

N=12+2*4=20,…También podemos declarar una variable y después

trabajar con k*(k+1). Así hemos procedido en nuestra primera

búsqueda con hojas de cálculo.

Así que, mientras los cuadrados se generan sumando impares,

los oblongos sumando pares (y los triangulares sumando todos)

Caracterización de estos números

Necesitaremos también en las búsquedas que emprenderemos una

forma de caracterizar estos números, cómo saber si un resultado es

cuadrado u oblongo, por ejemplo. Aunque es sencillo y conocido, lo

recordamos aquí:

Para saber si un número natural es cuadrado, la mejor prueba es que

la parte entera de su raíz cuadrada, elevada a su vez al cuadrado, nos

dé como resultado el número primitivo. En hoja de cálculo:

Public Function escuad(n) As Boolean

If n < 0 Then

escuad = False

Else

If n = Int(Sqr(n)) ^ 2 Then escuad = True Else escuad = False

End If

End Function

Número N oblongo Incremento D

2 4

6 6

12 8

20 10 Crecen de 2 en 2

30 12

42 14

56 16

72 18

Page 89: Números y formas - Aprender y divertirse con hoja de …hojamat.es/publicaciones/numform.pdf · Este es uno de los capítulos de las Matemáticas en los que se aprecia ... también

89

Si trabajas con las celdas, sin macros, el procedimiento es el mismo,

pero con distinto lenguaje. Si tienes, por ejemplo, en la celda C12, un

número del que deseas saber si es cuadrado, escribe en otra celda

esta fórmula: =SI((ENTERO(RAIZ(C12)))^2=C12;1;0), y te devolverá

un 1 si es cuadrado y un 0 si no lo es.

La caracterización de un número como triangular se basa en lo

anterior, ya que ser triangular N es equivalente a que 8*N+1 sea

cuadrado. Por tanto, podemos definir esta función:

Function estriangular(n) As Boolean

If escuad(8 * n + 1) Then estriangular = True Else estriangular =

False

End Function

En lenguaje de celdas sería algo más complejo:

=SI((ENTERO(RAIZ(C12*8+1)))^2=C12*8+1;1;0),

Por último, los oblongos, al ser doble de triangulares, se descubren

fácilmente:

Public Function esoblongo(n) As Boolean

If escuad(4 * n + 1) Then esoblongo = True Else esoblongo = False

End Function

Sin macros, =SI((ENTERO(RAIZ(C12*4+1)))^2=C12*4+1;1;0)

Algoritmos de búsqueda

Ya podemos pasar a nuestras búsquedas. Comenzaremos generando

la sucesión https://oeis.org/A188630, con la que comenzamos este

Page 90: Números y formas - Aprender y divertirse con hoja de …hojamat.es/publicaciones/numform.pdf · Este es uno de los capítulos de las Matemáticas en los que se aprecia ... también

90

estudio: Números triangulares que son producto de otros dos

triangulares. Nos organizaremos así:

Iniciamos triangulares: N=3, D=3 (Comenzamos por el 3)

1 Mientras no lleguemos al tope que nos hayamos marcado

Iniciamos otros triangulares para ver si son divisores: K=3, P=3

2 Mientras no lleguemos a N ni encontremos un producto de

triangulares

Para cada K vemos si:

1.-Es divisor de N

2.- Si el cociente N/k es triangular

Si cumple ambas condiciones, cerramos la búsqueda para N e

imprimimos.

Si no, generamos otro triangular convirtiendo K en K+P y P en

P+1

Fin de 2 Mientras

Generamos otro triangular convirtiendo N en N+D y D en D+1

Fin del 1 Mientras

Hemos comenzado los triangulares en el 3 para evitar trivialidades.

Para quienes no manejen mucho los algoritmos puede resultar

complicado, pero hay que repasar hasta entenderlo. Se puede traducir

al Visual Basic de Excel:

Sub productriang()

Dim i, j, k, p, c

i = 3: j = 3 Iniciamos la búsqueda en 3, para eliminar trivialidades

While i <= 10 ^ 4

k = 3: p = 3: c = 0 También iniciamos los divisores en 3

While c = 0 And p < i

If i / k = i \ k And estriangular(i / k) And i / k > 1 Then c = k

En la línea anterior buscamos que sea divisor, cociente triangular y no

trivial

If c <> 0 Then MsgBox (i) Si cumple todo, presentamos el resultado

k = k + p: p = p + 1 Generamos el siguiente divisor

Wend

Page 91: Números y formas - Aprender y divertirse con hoja de …hojamat.es/publicaciones/numform.pdf · Este es uno de los capítulos de las Matemáticas en los que se aprecia ... también

91

i = i + j: j = j + 1 Generamos el siguiente triangular

Wend

End Sub

Elimina comentarios, copia el resto como rutina para Visual Basic y al

ejecutar verás aparecer los valores 36, 45, 210,…

Si te apetece explorar, aquí tienes la versión para PARI

{i=3;j=3; Iniciamos la búsqueda en 3, para eliminar trivialidades

while(i<=10^4,k=3;p=3;c=0; También iniciamos los divisores en 3

while(k<i&&c==0,if(i/k==i\k&&ispolygonal(i/k,3)&&i/k>1,c=k);

En la línea anterior buscamos que sea divisor, cociente triangular

y no trivial

if(c>0,print1(i,", ")); Si cumple todo, imprimimos

k+=p;p+=1); Generamos el siguiente divisor

i+=j;j+=1) Generamos el siguiente triangular

}

Igualmente, si eliminas los comentarios y ejecutas este código PARI

verás reproducida la sucesión https://oeis.org/A188630

Números triangulares como producto de otros

En el apartado anterior planteamos el problema de buscar los números

triangulares que son a su vez producto de otros dos triangulares.

Presentamos una forma de generar cuadrados, oblongos y triangulares

de la forma más rápida posible, ya que estas búsquedas se hacen

lentas para números grandes, y construimos un algoritmo para generar

estos números, contenidos en la sucesión de OEIS

https://oeis.org/A188630

Page 92: Números y formas - Aprender y divertirse con hoja de …hojamat.es/publicaciones/numform.pdf · Este es uno de los capítulos de las Matemáticas en los que se aprecia ... también

92

Ahora generaremos triangulares con otros tipos de números, y

llegaremos a sucesiones que permanecían inéditas hasta ahora.

Triangulares producto de dos cuadrados

Aquí no hay que buscar mucho, ya que basta considerar que el número

que cumple esto es aquel que es cuadrado (producto de cuadrados) y

triangular a la vez. Esto está muy estudiado. Son estos:

1, 36, 1225, 41616, 1413721, 48024900, 1631432881,… y está

publicados en https://oeis.org/A001110

Vemos, pues, otros casos.

Triangulares que son producto de un triangular y un cuadrado

Ahora tenemos ocasión de aplicar lo expuesto anteriormente: cómo

generar de forma rápida cuadrados y triangulares y cómo saber si son

o no de ese tipo. Si esto te quedó claro, entenderás el algoritmo que

sigue. Primero en Visual Basic de Excel:

Sub productriang()

Dim i, j, k, p, c

i = 3: j = 3 Generamos el primer triangular

While i <= 10 ^ 4

k = 3: p = 3: c = 0 Aquí generamos posibles divisores

triangulares

While c = 0 And p < i Cuando c<>0 se para la búsqueda y

se comunica el resultado

If i / k = i \ k And escuad(i / k) And i / k > 1 Then c = k

La línea de arriba investiga si k es divisor y si i/k es

cuadrado

If c <> 0 Then MsgBox (i)

k = k + p: p = p + 1 Genera el siguiente posible divisor triangular

300

1176

3240

7260

14196

25200

29403

41616

64980

Page 93: Números y formas - Aprender y divertirse con hoja de …hojamat.es/publicaciones/numform.pdf · Este es uno de los capítulos de las Matemáticas en los que se aprecia ... también

93

Wend

i = i + j: j = j + 1 Genera el siguiente triangular para seguir buscando

Wend

End Sub

Si repasas el apartado anterior, es el mismo algoritmo propuesto en él,

pero cambiando estriangular(i/k) por escuad(i/k). Puedes repasar los

comentarios que se incluían. Con un ligero cambio en el código, hemos

situado los resultados en columna:

A partir de este valor se produce desbordamiento, por lo que acudimos

a PARI y al código

{i=3;j=3;

while(i<=10^6,k=3;p=3;c=0;

while(k<i&&c==0,if(i/k==i\k&&issquare(i/k)&&i/k>1,c=k);

if(c>0,print1(i,", "));

k+=p;p+=1);

i+=j;j+=1)

}

No insertamos comentarios porque es el mismo que presentamos en la

anteriormente, salvo el uso de la función issquare (“es cuadrado”)

Con él puedes obtener los primeros números triangulares que son

producto de un triangular y un cuadrado:

Page 94: Números y formas - Aprender y divertirse con hoja de …hojamat.es/publicaciones/numform.pdf · Este es uno de los capítulos de las Matemáticas en los que se aprecia ... también

94

300, 1176, 3240, 7260, 14196, 25200, 29403, 41616, 64980, 97020,

139656, 195000, 228150, 265356, 353220, 461280, 592416, 749700,

936396, 1043290, 1155960, 1412040, 1708476, 2049300, 2438736,

2881200, 3381300, 3499335, 3943836, 4573800,…

Esta sucesión no se había publicado, y lo hicimos en

http://oeis.org/A253650

Si los escribimos en columna podemos desarrollarlos como producto

del tipo deseado:

Antes de seguir adelante, hay que advertir que estas

descomposiciones en producto no tienen que ser únicas. Por ejemplo,

2881200 admite estos dos productos: 3*960400 y 300*9604, ambos

formados por un triangular y un cuadrado. Por eso en la tabla se puede

Triangular Cuadrado Triangular

300 100 3

1176 196 6

3240 324 10

7260 484 15

14196 676 21

25200 900 28

29403 9801 3

41616 1156 36

64980 1444 45

97020 1764 55

139656 2116 66

195000 2500 78

Page 95: Números y formas - Aprender y divertirse con hoja de …hojamat.es/publicaciones/numform.pdf · Este es uno de los capítulos de las Matemáticas en los que se aprecia ... también

95

tener la falsa idea de que el factor triangular es más pequeño que el

cuadrado. No es así, sino que hemos detenido la búsqueda al

encontrar un ejemplo.

Una curiosidad

Los números triangulares de esta sucesión, tendrán, como todos la

forma T(P)=P(P+1)/2. Pues bien, ni P ni P+1 pueden ser primos,

porque si se descompone en un producto de un triangular y un

cuadrado, tendríamos P(P+1)=k(k+1)m2 y si P o P+1 fueran primos,

tendrían que dividir a k o a k+1 o a m, y los tres son menores que P.

Estúdialo. Lo vemos en una tabla con los primeros casos:

Subconjunto interesante

Los números triangulares de orden k2-1, siendo k impar y mayor que 1,

pertenecen a esta sucesión. En efecto, si k es impar tendrá la forma

2m+1, luego el orden del triangular será

(2m+1)2-1=4m2+4m.

El triangular formado sobre él tendrá la forma ((2m+1)2*(4m2+4m)/2 y

se puede descomponer en un cuadrado y un triangular: 4(2m+1)2 *

m(m+1)/2.

Triangular P Factores(P) P+1 Factores(P+1)

300 24 [2,3][3,1] 25 [5,2]

1176 48 [2,4][3,1] 49 [7,2]

3240 80 [2,4][5,1] 81 [3,4]

7260 120 [2,3][3,1][5,1] 121 [11,2]

14196 168 [2,3][3,1][7,1] 169 [13,2]

25200 224 [2,5][7,1] 225 [3,2][5,2]

29403 242 [2,1][11,2] 243 [3,5]

41616 288 [2,5][3,2] 289 [17,2]

64980 360 [2,3][3,2][5,1] 361 [19,2]

97020 440 [2,3][5,1][11,1] 441 [3,2][7,2]

139656 528 [2,4][3,1][11,1] 529 [23,2]

195000 624 [2,4][3,1][13,1] 625 [5,4]

Page 96: Números y formas - Aprender y divertirse con hoja de …hojamat.es/publicaciones/numform.pdf · Este es uno de los capítulos de las Matemáticas en los que se aprecia ... también

96

Otra forma de expresarlo es que son triangulares construidos sobre 8

veces otro número triangular, ya que 4m2+4m=8*(m(m+1)/2). Estos

números los tienes en http://oeis.org/A185096

Casi todos los elementos de la sucesión tienen esta forma: 300, 1176,

3240, 7260, 14196, 25200,… y pertenecen a la sucesión

http://oeis.org/A083374 pero otros no, como 29403, 1043290 y

3499335.

Triangulares que son producto de un triangular y un primo

En esta búsqueda nos organizaremos de forma similar a la precedente

pero al llegar a investigar si (i/k) es cuadrado o triangular, deberemos

sustituirlo por la pregunta de si es primo. Esta es más difícil de

responder, pero disponemos de la función isprime en PARI y de

esprimo en nuestra hoja Conjeturas (situada en

http://www.hojamat.es/sindecimales/divisibilidad/herramientas/herrdiv.h

tm#global)

Puedes adaptar el algoritmo usado en la búsqueda anterior cambiando

esas funciones: Observa que ahora, con lo explicado antes, las

operaciones se entienden mejor, por lo que omitimos los comentarios y

usamos color rojo en las novedades. Para hoja de cálculo podría servir

esta rutina de Visual Basic:

Sub productriang()

Dim i, j, k, p, c

i = 3: j = 3

While i <= 10 ^ 3

k = 3: p = 3: c = 0

While c = 0 And p < i

Page 97: Números y formas - Aprender y divertirse con hoja de …hojamat.es/publicaciones/numform.pdf · Este es uno de los capítulos de las Matemáticas en los que se aprecia ... también

97

If i / k = i \ k And esprimo(i / k) And i / k > 1 Then c = k

If c <> 0 Then MsgBox (i)

k = k + p: p = p + 1

Wend

i = i + j: j = j + 1

Wend

End Sub

En PARI

{i=3;j=3;while(i<=10^6,k=3;p=3;c=0;while(k<i&&c==0,if(i/k==i\k&&is

prime(i/k)&&i/k>1,c=k);if(c>0,print1(i,", "));k+=p;p+=1);i+=j;j+=1)}

Los triangulares obtenidos son, en este caso, los siguientes:

6, 15, 21, 45, 66, 78, 105, 190, 210, 231, 435, 465, 630, 861, 903,

1035, 1326, 2415, 2556, 2628, 3003, 3570, 4005, 4950, 5460, 5565,

5995, 7140, 8646, 8778, 9870, 12246, 16471, 16836, 17205, 17391,

17766, 20100, 22155, 26565, 26796, 28680, 28920, 30381, 32131,

33411, 33930, 36856, 40755,… (los publicamos en

http://oeis.org/A253651)

Podíamos pensar que, al ser el segundo factor primo, el primero será

el máximo divisor triangular propio que tiene el número que se

descompone (ver http://hojaynumeros.blogspot.com.es/2013/02/de-los-

triangulares-alojados-los-primos.html). Esto es así en la gran mayoría

de casos. Así, 4005 se descompone como 45*89, siendo 45 es el

máximo divisor triangular propio de 4005 y 89 un factor primo. Sin

embargo, en el número 3570, su divisor triangular máximo es 595, que

daría lugar al producto 3570=595*6, y el 6 no es primo. El producto

válido sería 3570=210*17, que sí es primo.

Page 98: Números y formas - Aprender y divertirse con hoja de …hojamat.es/publicaciones/numform.pdf · Este es uno de los capítulos de las Matemáticas en los que se aprecia ... también

98

Triangulares producto de triangular y oblongo

Estos son los primeros:

6, 36, 120, 210, 300, 630, 1176, 2016, 3240, 3570, 4950, 7140, 7260,

10296, 14196, 19110, 23436, 25200, 32640, 39060, 41616, 52326,

61776, 64980, 79800, 97020, 116886, 139656, 145530, 165600,

195000, 228150, 242556, 265356, 304590, 306936, 349866, 353220,

404550, 426426, …

Si recuerdas la caracterización de los oblongos en el aparado anterior,

entenderás este código en PARI:

{i=3;j=3;

while(i<=10^6,k=2;p=4;c=0;

while(k<i&&c==0,if(i/k==i\k&&issquare(4*(i/k)+1)&&i/k>1,c=k);

if(c>0,write1("final.txt",i,", "));

k+=p;p+=2);

i+=j;j+=1)

}

Destacamos que el doble de cualquiera de estos números tiene la

forma N=P(P+1)Q(Q+1). Así, 2*2016=4032=7*8*8*9. Al contrario,

como uno de los factores es oblongo, el producto será par y se podrá

dividir entre dos y su mitad será producto de dos triangulares.

Todos los términos no nulos de la sucesión A083374 (6, 36, 120, 300,

630, 1176, 2016, 3240,…) pertenecen a esta, pues si tienen la forma

n2(n2-1)/2 se pueden descomponer en el oblongo n(n+1) y el triangular

n(n-1)/2, o bien el oblongo n(n-1) y el triangular n(n+1)/2.

Hemos publicado esta sucesión en http://oeis.org/A253652

Page 99: Números y formas - Aprender y divertirse con hoja de …hojamat.es/publicaciones/numform.pdf · Este es uno de los capítulos de las Matemáticas en los que se aprecia ... también

99

Triangulares producto de un cuadrado y un primo

Al llegar a este punto simplificamos la exposición, ya que tendrás una

buena idea de cómo se generan. Los triangulares que se forman

multiplicando un cuadrado y un primo son estos:

28, 45, 153, 171, 300, 325, 496, 2556, 2628, 3321, 4753, 4851, 7381,

8128, 13203, 19900, 25200, 25425, 29161, 29403, 56953, 64980,

65341, 101025, 166753, 195625, 209628, 320400, 354061, 388521,

389403, 468028, 662976, 664128, 749700, 750925, 780625, 781875,

936396,…

Los números perfectos lo cumplen

Entre los números encontrados figuran los perfectos 28, 496, 8128,…

https://oeis.org/A000396

La razón es que estos números tienen la forma 2k-1(2k-1)= 2k(2k-1)/2, y

son, por tanto, triangulares. Además, como (2k-1) es primo de

Mersenne en esta expresión, k ha de ser impar, y k-1, par, con lo que

2k-1 es un cuadrado y (2k-1) un primo, cumpliéndose así la condición.

Para no cansar más con este tema, sólo incluimos el código PARI con

las novedades en rojo:

PARI

{i=3;j=3;

while(i<=10^6,k=4;p=5;c=0;

while(k<i&&c==0,if(i/k==i\k&&isprime(i/k)&&i/k>1,c=k);

if(c>0,write1("final.txt",i,", "));

k+=p;p+=2);

i+=j;j+=1)

}

Page 100: Números y formas - Aprender y divertirse con hoja de …hojamat.es/publicaciones/numform.pdf · Este es uno de los capítulos de las Matemáticas en los que se aprecia ... también

100

Pues ya está bien. Ahora te toca a ti. ¿Sabrías prolongar las siguientes

sucesiones con las técnicas que hemos desarrollado en este capítulo?:

Triangulares producto de un cuadrado y un oblongo: 120, 300, 378,

528, 990, 1176, 2016,…

Y por último, los triangulares producto de un oblongo y un primo: 6, 10,

36, 66, 78, 210, 276, …

Con números cuadrados

Proseguimos el tema de buscar números especiales que son producto

de dos factores también especiales. Hemos estudiado algunas

variantes con triangulares, cuadrados, oblongos y primos. Ahora

descompondremos cuadrados como producto de otros dos de algún

tipo especial.

Un caso que no necesita estudio es el de cuadrados producto de

cuadrados, pues esta condición la cumplen todos salvo los cuadrados

de primos. Así que pasamos a otros productos. En todos ellos

exigiremos que los factores del producto sean distintos, para evitar

trivialidades, ya que si son iguales su producto sería cuadrado sin

necesidad de buscar más.

Los algoritmos se basarán todos en la generación de cuadrados

mayores que 1 que ya estudiamos, es decir, iniciar P=4 y K=5 y

después, en cada pasada del bucle, hacer P=P+K y K=K+2, ya que

sabemos que los incrementos de los cuadrados crecen de 2 en 2.

Producto de triangulares distintos

Hemos encontrado esta sucesión, que contiene muchos cuadrados de

términos de https://oeis.org/A175497:

Page 101: Números y formas - Aprender y divertirse con hoja de …hojamat.es/publicaciones/numform.pdf · Este es uno de los capítulos de las Matemáticas en los que se aprecia ... también

101

900, 7056, 32400, 44100, 88209, 108900, 298116, 705600, 1368900,

1498176, 2924100, 5336100, 8643600, 8820900, 9217296, 10432900,

15210000, 24147396, 37088100, 50893956, 50979600, 52490025,

55353600, 80568576, 114704100, 160123716, 200930625,

219632400, 265559616, 268304400, 296528400, 394657956,…

Pronto se llega a números grandes, por la fuerte condición que les

exigimos. Por ejemplo, 88209 es el cuadrado de 297, y se puede

descomponer en el producto de dos triangulares distintos:

88209=29403*3 y 29403 es el triangular número 242 (29403

=242*243/2) y 3 es el segundo (3=2*3/2).

Hemos comprobado los cálculos con dos métodos. Aquí tienes el de

PARI

{i=4;j=5; ‘Inicia cuadrados

while(i<=5*10^8,k=3;p=3;c=0; ‘Inicia factores triangulares

while(k<sqrt(i)&&c==0,if(i/k==i\k&&ispolygonal(i/k,3)&&i/k>1,c=k);

‘Sólo llegamos en la búsqueda hasta la raíz cuadrada, para que ambos

factores sean distintos

if(c>0,print(i);write1("final.txt",i,", ")); ‘Si c>0 se ha encontrado una

solución

k+=p;p+=1);

i+=j;j+=2)

}

Ambos factores triangulares han de tener algún factor en común para

que se forme un

cuadrado.

Sus partes libres de

cuadrados serán

900 3 [3,1] 300 [2,2][3,1][5,2]

7056 6 [2,1][3,1] 1176 [2,3][3,1][7,2]

32400 10 [2,1][5,1] 3240 [2,3][3,4][5,1]

44100 36 [2,2][3,2] 1225 [5,2][7,2]

88209 3 [3,1] 29403 [3,5][11,2]

108900 15 [3,1][5,1] 7260 [2,2][3,1][5,1][11,2]

298116 21 [3,1][7,1] 14196 [2,2][3,1][7,1][13,2]

705600 28 [2,2][7,1] 25200 [2,4][3,2][5,2][7,1]

1368900 6 [2,1][3,1] 228150 [2,1][3,3][5,2][13,2]

1498176 36 [2,2][3,2] 41616 [2,4][3,2][17,2]

2924100 45 [3,2][5,1] 64980 [2,2][3,2][5,1][19,2]

5336100 55 [5,1][11,1] 97020 [2,2][3,2][5,1][7,2][11,1]

8643600 3 [3,1] 2881200 [2,4][3,1][5,2][7,4]

8820900 300 [2,2][3,1][5,2] 29403 [3,5][11,2]

9217296 66 [2,1][3,1][11,1] 139656 [2,3][3,1][11,1][23,2]

10432900 10 [2,1][5,1] 1043290 [2,1][5,1][17,2][19,2]

15210000 78 [2,1][3,1][13,1] 195000 [2,3][3,1][5,4][13,1]

24147396 91 [7,1][13,1] 265356 [2,2][3,6][7,1][13,1]

Page 102: Números y formas - Aprender y divertirse con hoja de …hojamat.es/publicaciones/numform.pdf · Este es uno de los capítulos de las Matemáticas en los que se aprecia ... también

102

iguales, ya que es la única forma de que al final resulte un cuadrado.

Lo puedes comprobar con algunos ejemplos de la tabla. Por ejemplo,

705600 es el triangular número 1187, y se descompone en el triangular

28, con parte libre 7, y el triangular25200=60^2*7, que tiene también un

7 como parte libre.

Cuadrados producto de dos oblongos distintos

Este caso es equivalente al que exige que un cuadrado sea igual a

cuatro veces el producto de dos triangulares distintos. Así, si el

cuadrado 3600 es el producto de los dos oblongos 6=2*3 y 600=24*25,

también cumple, como es evidente, que su cuarta parte es el producto

de dos triangulares 3600/4=900=3*300.

Los primeros cuadrados que cumplen esto son:

144, 3600, 4900, 28224, 129600, 166464, 176400, 352836, 435600,

1192464, 2822400, 5475600, 5654884, 5992704, 11696400,

21344400, 34574400, 35283600, 36869184, 41731600, 60840000,

96589584, 148352400, 192099600, 203575824, 203918400,

209960100, 221414400, 322274304, 458816400,…

Aquí tienes la descomposición en producto de oblongos de los

primeros:

Page 103: Números y formas - Aprender y divertirse con hoja de …hojamat.es/publicaciones/numform.pdf · Este es uno de los capítulos de las Matemáticas en los que se aprecia ... también

103

Como todos son pares, ya tienen un factor común, y como en el caso

anterior, sus partes libres de cuadrados han de ser iguales.

Los puedes conseguir en PARI con

{i=4;j=5;while(i<=5*10^8,k=2;p=4;c=0;while(k<sqrt(i)&&c==0,if(i/k==i\k&

&issquare(4*i/k+1)&&i/k>1,c=k);if(c>0,print(i));k+=p;p+=2);i+=j;j+=2)}

Si usamos nuestra imaginación podemos recorrer muchas variantes,

pero nos quedaremos con esta última:

Cuadrados producto de triangular y primo

9, 196, 225, 900, 2601, 3249, 4225, 15376, 53361, 88209, 136161,

176400, 181476, 191844, 324900, 450241, 461041, 1032256,

2152089, 2873025, 3960100, 7027801, 8643600, 11826721,

12744900, 17791524, 19193161, 28515600, 43956900, 45360225,

61230625, 63282025, 96216481, 108680625,…

Aquí es como si el número primo aportara el factor que se necesita

para convertir un triangular en un cuadrado. Por tanto, el factor primo

es igual a la parte libre de cuadrados del otro factor triangular.

144 2 [2,1] 72 [2,3][3,2]

3600 6 [2,1][3,1] 600 [2,3][3,1][5,2]

4900 2 [2,1] 2450 [2,1][5,2][7,2]

28224 12 [2,2][3,1] 2352 [2,4][3,1][7,2]

129600 20 [2,2][5,1] 6480 [2,4][3,4][5,1]

166464 2 [2,1] 83232 [2,5][3,2][17,2]

176400 72 [2,3][3,2] 2450 [2,1][5,2][7,2]

352836 6 [2,1][3,1] 58806 [2,1][3,5][11,2]

435600 30 [2,1][3,1][5,1] 14520 [2,3][3,1][5,1][11,2]

1192464 42 [2,1][3,1][7,1] 28392 [2,3][3,1][7,1][13,2]

2822400 56 [2,3][7,1] 50400 [2,5][3,2][5,2][7,1]

5475600 12 [2,2][3,1] 456300 [2,2][3,3][5,2][13,2]

5654884 2 [2,1] 2827442 [2,1][29,2][41,2]

5992704 72 [2,3][3,2] 83232 [2,5][3,2][17,2]

11696400 90 [2,1][3,2][5,1] 129960 [2,3][3,2][5,1][19,2]

21344400 110 [2,1][5,1][11,1] 194040 [2,3][3,2][5,1][7,2][11,1]

Page 104: Números y formas - Aprender y divertirse con hoja de …hojamat.es/publicaciones/numform.pdf · Este es uno de los capítulos de las Matemáticas en los que se aprecia ... también

104

Por tener esta propiedad disponemos de dos códigos distintos para

encontrar esos números. Uno es el “largo”, que sigue lo explicado

previamente:

{i=4;j=5;

while(i<=5*10^8,k=3;p=3;c=0;

while(k<i&&c==0,if(i/k==i\k&&isprime(i/k)&&i/k>1,c=k);

if(c>0,print(i);write1("final.txt",i,", "));

k+=p;p+=1);

i+=j;j+=2)

}

Podemos usar este otro, mucho más corto, pero que no nos ordena los

números en orden creciente:

{i=1;j=2;while(i<=10^5,m=core(i);if(isprime(m),print1(i*m,", "));i+=j;j+=1)}

Como hemos indicado, se podría seguir con otros casos. Intenta

reproducir este:

Cuadrados producto de un triangular y un oblongo

36, 900, 3600, 7056, 15876, 41616, 44100, 54756, 69696, 108900,…

Número Factor triangular Factor primo Parte libre de cuadrados del triangular

9 3 3 3

196 28 7 7

225 45 5 5

900 300 3 3

2601 153 17 17

3249 171 19 19

4225 325 13 13

15376 496 31 31

53361 4851 11 11

88209 29403 3 3

136161 3321 41 41

176400 25200 7 7

181476 2556 71 71

191844 2628 73 73

Page 105: Números y formas - Aprender y divertirse con hoja de …hojamat.es/publicaciones/numform.pdf · Este es uno de los capítulos de las Matemáticas en los que se aprecia ... también

105

EQUILIBRADOS ENTRE SEMEJANTES

Existen números equilibrados que son media entre el anterior y el

posterior de la misma clase. Así, un número primo es equilibrado si es

promedio de sus dos primos contiguos. Por ejemplo, 257 es media de

su anterior 251 y el posterior 263, que por cierto también es primo

equilibrado. Los tres primos componentes de la terna formarán, pues,

una progresión aritmética.

Los primos equilibrados los tienes en http://oeis.org/A006562

5, 53, 157, 173, 211, 257, 263, 373, 563, 593, 607, 653, 733, 947,

977, …

Si dispones de las funciones ESPRIMO, PRIMANT y PRIMPROX (las

puedes encontrar en

http://www.hojamat.es/sindecimales/divisibilidad/herramientas/herrdiv.h

tm#global ), es fácil encontrarlos. Por ejemplo, con esta función:

Public Function equiprimo(n)

If esprimo(n) And n = (primprox(n) + primant(n)) / 2 Then

equiprimo = True Else equiprimo = False

End Function

Con ella es fácil reproducir la lista:

Las diferencias, salvo en el 5, son múltiplos de 6. La razón es

que a partir del 5 todos los primos son del tipo 6n+1 o 6n+5. En

las ternas que se forman tienen que ser todos del mismo tipo,

ya que si el primero es 6n+1 y el segundo 6m+5, el tercero

tendría el tipo 6m+5+(6k+4)=6h+3, no primo. Igualmente, si el

primero es tipo 6n+5 y el segundo 6m+1, el tercero sería

6m+1+(6h+2). Lo puedes ver con Z6: Si el primero tuviera resto 1 y el

Page 106: Números y formas - Aprender y divertirse con hoja de …hojamat.es/publicaciones/numform.pdf · Este es uno de los capítulos de las Matemáticas en los que se aprecia ... también

106

último resto 5, el promedio presentaría resto 3 y no sería primo. Igual

con los otros casos. Una consecuencia curiosa de esto es la sucesión

publicada en http://oeis.org/A101597, que cuenta el número de

compuestos comprendidos entre el primo equilibrado y sus contiguos, y

es claro que todos los elementos tienen el valor 5, 11, 17,…es decir, un

múltiplo de 6 menos 1.

Se ha conjeturado que existen infinitos primos equilibrados.

Otros números equilibrados

Con cuadrados

Ningún cuadrado como tal puede ser equilibrado, ya que (n+1)2-

n2=2n+1 y n2-(n-1)2=2n-1. Igual le ocurre a los triangulares, ya que, por

definición, la diferencia entre el triangular de orden n y su anterior es

precisamente n, y con su posterior, n+1. No busques equilibrados entre

números poligonales o procedentes de valores numéricos de

polinomios.

Así que tendremos que ir visitando otros tipos de números hasta dar

con aquellos que presenten elementos equilibrados.

Libres de cuadrados

Este tipo de números sí admite equilibrados. Los tienes en

http://oeis.org/A245289

2, 6, 14, 17, 19, 22, 26, 30, 34, 38, 42, 53, 55, 58, 66, 70, 78, 86, 89,

91, 94, 102, 106, 110, 114, 130, 138, 142, 158, 161, 163, 166, 170,

178, 182, 186, 194, 197, 199, 202, 210, 214, 218, 222, 230, 233, 235,

238,…

Los hemos reproducido con hoja de cálculo, incorporando sus factores

primos, y nos ha llamado la atención que en la terna de libres de

cuadrados consecutivos figuran muchos primos, lo que hace que el

M.C.D. de los integrantes de la terna sea frecuentemente un 1.

Page 107: Números y formas - Aprender y divertirse con hoja de …hojamat.es/publicaciones/numform.pdf · Este es uno de los capítulos de las Matemáticas en los que se aprecia ... también

107

Hemos buscado factores comunes en muchas ternas, hasta 10^8, y

sólo hemos encontrado el 2. No parece que tengan en común los

factores 3, 5 o 7. Si aparece este caso, será para números muy

grandes. Con PARI hemos obtenido listados de ternas con M.C.D.

igual a 2, pero no para valores mayores. No tenemos respuesta para la

cuestión de si terminarán apareciendo.

Semiprimos equilibrados

También se pueden encontrar ternas de semiprimos consecutivos que

formen progresión aritmética, con lo que el central de la terna sería un

semiprimo equilibrado. Son estos:

34, 86, 94, 122, 142, 185, 194, 202, 214, 218, 262, 289, 302, 314, 321,

358, 371, 394, 407, 413, 415, 422, 446, 471, 489, 493, 497, 517, 535,

562, 581, 586, 626, 634, 669, 687, 698, 734, 785, 791, 815, 838, 842,

Terna de libres de cuadrados equilibrados Descomposición en factores primos

29 30 31 [29,1] [2,1][3,1][5,1] [31,1]

33 34 35 [3,1][11,1] [2,1][17,1] [5,1][7,1]

37 38 39 [37,1] [2,1][19,1] [3,1][13,1]

41 42 43 [41,1] [2,1][3,1][7,1] [43,1]

51 53 55 [3,1][17,1] [53,1] [5,1][11,1]

53 55 57 [53,1] [5,1][11,1] [3,1][19,1]

57 58 59 [3,1][19,1] [2,1][29,1] [59,1]

65 66 67 [5,1][13,1] [2,1][3,1][11,1] [67,1]

69 70 71 [3,1][23,1] [2,1][5,1][7,1] [71,1]

77 78 79 [7,1][11,1] [2,1][3,1][13,1] [79,1]

85 86 87 [5,1][17,1] [2,1][43,1] [3,1][29,1]

87 89 91 [3,1][29,1] [89,1] [7,1][13,1]

89 91 93 [89,1] [7,1][13,1] [3,1][31,1]

93 94 95 [3,1][31,1] [2,1][47,1] [5,1][19,1]

Page 108: Números y formas - Aprender y divertirse con hoja de …hojamat.es/publicaciones/numform.pdf · Este es uno de los capítulos de las Matemáticas en los que se aprecia ... también

108

922, 982, 989, 1042, 1057, 1079, 1135, 1138,…

http://oeis.org/A213025

Podemos investigar aquí también qué factores comunes tienen estas

ternas de semiprimos. Hemos encontrado ternas con el factor 2 en

común:

En ellas los otros factores que acompañan al 2 son ternas de primos

equilibrados.

Esfénicos equilibrados

Existen esfénicos (productos de tres primos distintos) que son

equilibrados, es decir, que forman ternas en progresión aritmética con

el anterior y el posterior esfénico. Forman esta sucesión:

186, 370, 406, 418, 518, 582, 602, 710, 786, 814, 826, 830, 942, 978,

994, 1010, 1034, 1070, 1162, 1310, 1374, 1394, 1570, 1630, 1686,

1758, 1886, 1978, 2014, 2114, 2158, 2270, 2274, 2278, 2294, 2438,

2510, 2534, 2570, 2630, 2666, 2690, 2774, 2778, 2782, 2806, …

Entre ellos figura el año 2014, que ya se comentó en su día que

formaba una terna de esfénicos con el 2013 y el 2015.

Los hemos publicado en http://oeis.org/A258276

Siguiendo con la idea de estudiar el MCD de los tres elementos de la

terna, aquí encontramos una gran variedad de números primos como

resultado, entre ellos 705, 710 y 715 con factor común 5, o 3311, 3322

y 3333 con el 11. Al tener tres factores es más fácil obtener estos

resultados.

Page 109: Números y formas - Aprender y divertirse con hoja de …hojamat.es/publicaciones/numform.pdf · Este es uno de los capítulos de las Matemáticas en los que se aprecia ... también

109

Podríamos estudiar la misma cuestión con números formados por el

producto de cuatro primos distintos, y también encontraríamos

equilibrados:

1518, 1554, 2190, 2590, 3354, 4710, 4970, 5810, 7566, 8170, 10506,

11110, 11346, 12194, 12610, 13706, 14098, 15690, 16874, 17574,

18538, 18734, 19830, …

No hemos querido seguir para no cansar a los lectores. Si estudias el

código PARI que hemos usado puedes proseguir el estudio en esa

dirección, cambiando el 4 por 5, 6 o 7.

is4prim(n)=if(n>0,omega(n)==4&&bigomega(n)==4,0)

next4prim(n)={local(k=n+1);while(!is4prim(k),k+=1);k}

prec4prim(n)={local(k=n-1);while(!is4prim(k)&&k>0,k-=1);k}

{for(i=1,10^4,if(is4prim(i)&&2*i== next4prim(i)+

prec4prim(i),print(i)))}

DAMOS VUELTAS A LOS TRIANGULARES

CUADRADOS

Generación de la sucesión

Dos entradas del blog de John D. Cook

(http://www.johndcook.com/blog/2015/08/20/when-is-a-triangle-a-

square/ y siguiente) me han animado a volver a tomar el tipo de

cuestión al que llamé “dar vueltas” a un tema o concepto. Lo haré

sobre estos números:

0, 1, 36, 1225, 41616, 1413721, 48024900, 1631432881,…

(http://oeis.org/A001110)

Evidentemente, estos números tienen en común el ser triangulares y

cuadrados a la vez. Puedes leer un desarrollo sencillo y claro en este

documento:

Page 110: Números y formas - Aprender y divertirse con hoja de …hojamat.es/publicaciones/numform.pdf · Este es uno de los capítulos de las Matemáticas en los que se aprecia ... también

110

http://www2.caminos.upm.es/Departamentos/matematicas/revistapm/re

vista_impresa/vol_IV_num_1/jue_mat_num_triang.pdf

Me he dado cuenta de que es un concepto sencillo pero que da lugar a

bastantes reflexiones, algoritmos y repasos de teoría.

Nosotros seguiremos en parte este documento para iniciar el tema.

Búsqueda de números triangulares cuadrados

Un número triangular tiene por fórmula n(n+1)/2 y un cuadrado m2.

Aquellos números que participen de las dos características tendrán que

cumplir la igualdad

𝑛(𝑛 + 1)

2= 𝑚2

De esta igualdad deducimos esta otra mucho más práctica:

𝑛2 + 𝑛 +1

4− 2𝑚2 − 1/4 = (𝑛 +

1

2)2 − 2𝑚2 − 1/4 = 0

O bien

(2𝑛 + 1)2 − 8𝑚2 = 1

Con cambio de variable se convierte en una ecuación de Pell:

𝑥2 − 8𝑦2 = 1

Esta ecuación la tenemos muy estudiada

http://hojamat.es/parra/pell.pdf (documento de Rafael Parra)

http://hojaynumeros.blogspot.com.es/2010/02/ecuacion-de-pell.html

Disponemos además de una hoja de cálculo para ayudar a resolverla:

http://www.hojamat.es/sindecimales/aritmetica/herramientas/herrarit.ht

m#pell

Usamos esta herramienta para el coeficiente 8 y el segundo miembro 1

y nos dan las primeras soluciones:

Page 111: Números y formas - Aprender y divertirse con hoja de …hojamat.es/publicaciones/numform.pdf · Este es uno de los capítulos de las Matemáticas en los que se aprecia ... también

111

(1,0) (3,1) (17,6) (99,35) (577,204) (3363,1189) (19601,6930) (114243,

40391),…

Por recurrencia:

Según la teoría de la ecuación de Pell, las soluciones aparecen con las

recurrencias (en este caso) xn=3*xn-1+8*yn-1 yn=3*yn-1+1*xn-1. Por

ejemplo, 99=3*17+8*6, 35=3*6+1*17.

Ahora sólo nos queda elevar al cuadrado las soluciones de y (que

equivalen a la variable m de la primera igualdad que planteamos) y nos

resultarán los triangulares cuadrados:

02=0, 12=1, 62=36, 352=1225, 2042=41616, 11892= 1413721,…

Primer algoritmo

El estudio que acabamos de desarrollar nos da una pista para la

generación de términos triangulares cuadrados: Iniciamos dos

variables X=1, Y=0, y en cada paso del algoritmo convertimos X en

3X+8Y y la Y en 3Y+X. Terminado el cálculo presentamos el valor de

Y2 como siguiente triangular cuadrado. En el Basic de las hojas de

cálculo quedaría así:

Page 112: Números y formas - Aprender y divertirse con hoja de …hojamat.es/publicaciones/numform.pdf · Este es uno de los capítulos de las Matemáticas en los que se aprecia ... también

112

Sub triangcuad()

Dim x, y, x1, y1, i, t, fila

x = 1: y = 0 ‘Valores de inicio

fila = 3 ‘Fila inicial

Cells(fila, 4).Value = 0 ‘El primer valor es un cero

For i = 1 To 8 ‘Calculamos sólo ocho

x1 = 3 * x + 8 * y ‘Iteración para x

y1 = 3 * y + x ‘Iteración para y

x = x1

y = y1

t = y * y ‘Número triangular cuadrado

fila = fila + 1

Cells(fila, 4).Value = t ‘Se presenta el resultado

Next i

End Sub

Obtendríamos:

El que el último se nos ofrezca en coma flotante nos da idea de las

limitaciones de la hoja para cálculos con enteros de muchas cifras. Si

acudimos a PARI no nos encontraremos con esas limitaciones. Prueba

este código:

{x=1;y=0;print(0);while(x<10^10,x1=3*x+8*y;y1=3*y+x;x=x1;y=y1;t=

y^2;print(t))}

Page 113: Números y formas - Aprender y divertirse con hoja de …hojamat.es/publicaciones/numform.pdf · Este es uno de los capítulos de las Matemáticas en los que se aprecia ... también

113

En pocos segundos te presenta los triangulares cuadrados menores

que 10^10.

Relación de recurrencia con una sola variable

Por la naturaleza de su definición podemos esperar que estos números

sigan una relación de recurrencia de segundo orden. Para encontrar su

expresión, que será del tipo

An=aAn-1+bAn-2+c usaremos los valores iniciales 0, 1, 36, 1225, 41616

para plantear:

36=a*1+b*0+c

1225=a*36+b*1+c

41616=a*1225+b*36+c

Resolvemos

1189=35a+b

40391=1189a+35b

1224=36a y a=34, b=-1 y c=2

Según los cálculos anteriores, la relación de recurrencia será

An=34An-1-An-2+2

Es la misma que propone John D. Cook en su blog.

En este blog no olvidamos la hoja de cálculo. Intenta una resolución

como la de la imagen usando cálculo matricial:

Page 114: Números y formas - Aprender y divertirse con hoja de …hojamat.es/publicaciones/numform.pdf · Este es uno de los capítulos de las Matemáticas en los que se aprecia ... también

114

A partir de esta escritura matricial del sistema de ecuaciones, creamos

debajo la matriz inversa de los coeficientes con MINVERSA, y a su

derecha su producto por los términos independientes con MMULT:

Conseguimos así la misma solución 34, -1, 2

Segundo algoritmo

La relación de recurrencia nos permite un segundo algoritmo para

encontrar los triangulares cuadrados. El que describimos a

continuación presenta los nueve primeros (después existen problemas

de coma flotante)

Sub triancuad1()

Dim m, n, p, k, fila

m = 0: n = 1 ‘Valores iniciales

fila = 3

Cells(1, 3).Value = m ‘presenta los dos primeros términos

Cells(2, 3).Value = n

For k = 1 To 7

p = 34 * n - m + 2 ‘relación de recurrencia

Cells(fila, 3).Value = p: fila = fila + 1 ‘presenta los siguientes términos

m = n: n = p ‘cada término se convierte en el anterior

Next k

End Sub

Page 115: Números y formas - Aprender y divertirse con hoja de …hojamat.es/publicaciones/numform.pdf · Este es uno de los capítulos de las Matemáticas en los que se aprecia ... también

115

Los términos rellenarán una columna de hoja de cálculo:

Siguiendo nuestra costumbre, lo traducimos a PARI para conseguir

más términos:

{x=0;y=1;print(0);print(1);for (k=1, 20, z=34*y-x+2;print(z);x=y;y=z)}

No resistimos la tentación, al igual que propone J.C. Cook, de intentar

una versión recursiva en forma de función. Funciona muy bien en hoja

de cálculo:

Public Function ftriangcuad(n)

If n < 2 Then

ftriangcuad = n

Else

ftriangcuad = 34 * ftriangcuad(n - 1) - ftriangcuad(n - 2) + 2

End If

End Function

Page 116: Números y formas - Aprender y divertirse con hoja de …hojamat.es/publicaciones/numform.pdf · Este es uno de los capítulos de las Matemáticas en los que se aprecia ... también

116

No necesita explicación. La tabla siguiente se forma con gran rapidez

de cálculo:

Fórmula directa

Si lees el capítulo sobre sucesiones recurrentes en nuestra publicación

Sucesiones (http://www.hojamat.es/publicaciones/sucesiones.pdf)

entenderás que a partir de la fórmula de recurrencia es posible

encontrar la expresión directa de cada término (fórmula del término

general). Sólo insertamos la captura de pantalla de nuestra hoja de

cálculo Recurrencias

(http://www.hojamat.es/sindecimales/aritmetica/herramientas/herrarit.ht

m#recurre2)

en la parte homogénea de la recurrencia:

Page 117: Números y formas - Aprender y divertirse con hoja de …hojamat.es/publicaciones/numform.pdf · Este es uno de los capítulos de las Matemáticas en los que se aprecia ... también

117

Con un ligero retoque y la interpretación de los decimales llegamos a la

propuesta por John D. Cook:

𝐴(𝑛) =(17 + 12√2)

𝑛+ (17 − 12√2)

𝑛− 2

32

El uso de la raíz cuadrada de 2 le quita utilidad en nuestro trabajo, por

lo que intentaremos prescindir de ella. Para ver la influencia del

formato de coma flotante, la implementamos en hoja de cálculo, y el

resultado es similar al de los algoritmos anteriores:

Función generatriz

Para quien no lo sepa, diremos que la función generatriz de una

sucesión, si se desarrolla como una serie de potencias, poseerá como

coeficientes de esas potencias de x los términos de la sucesión.

En el caso de los números triangulares cuadrados la función generatriz

es

𝐹(𝑥) =𝑥(1 + 𝑥)

(1 − 𝑥)(1 − 34𝑥 + 𝑥2)

(ver http://oeis.org/A001110)

Con esta sencilla orden de PARI podemos comprobar su desarrollo.

print(taylor(x*(1+x)/((1-x)*(1-34*x+x^2)),x,20))

Page 118: Números y formas - Aprender y divertirse con hoja de …hojamat.es/publicaciones/numform.pdf · Este es uno de los capítulos de las Matemáticas en los que se aprecia ... también

118

Tercer algoritmo

Finalizamos con la presentación de un algoritmo de los que llamamos

“ingenuos”, que no usan la teoría para simplificar los cálculos, pero sí

la fuerza bruta de la velocidad de proceso. En este caso obligaremos a

los números naturales a ir creciendo hasta alcanzar un cuadrado, y

después a la inversa, que los cuadrados avancen hasta alcanzar un

triangular. Cuando se llegue a una igualdad se imprime el resultado. A

pesar de su simplicidad, no resulta lento. Es éste:

Sub triancuad2()

Dim i, j, m, n, k, fila

m = 3: n = 4: i = 2: j = 3 ‘Se inician las variables

k = 10 ^ 7

fila = 3

Cells(fila, 3).Value = 1: fila = fila + 1

While m < k ‘Busca soluciones menores que k

While m <= n ‘Los triangulares crecen

If m = n Then Cells(fila, 3).Value = m: fila = fila + 1 ‘Hay igualdad

i = i + 1: m = m + i

Wend

While n <= m ‘Los cuadrados crecen

If m = n Then Cells(fila, 3).Value = m: fila = fila + 1‘Hay igualdad

j = j + 2: n = n + j

Wend

Wend

End Sub

Dejamos a los lectores el estudio de por qué funciona este algoritmo

para descubrir los triangulares cuadrados. Como los anteriores, llega a

los mismos resultados, en este caso hasta 10^7:

Page 119: Números y formas - Aprender y divertirse con hoja de …hojamat.es/publicaciones/numform.pdf · Este es uno de los capítulos de las Matemáticas en los que se aprecia ... también

119

Curiosidades.

Otra recurrencia

Según un comentario incluido en http://oeis.org/A001110, podemos

tener en cuenta otra recurrencia a partir de n=3:

𝑎𝑛+1 =(𝑎𝑛 − 1)2

𝑎𝑛−1

En efecto, 1225=(36-1)2/1, 41616=(1225-1)2/36, … O con hoja de

cálculo:

Intenta averiguar cómo crear esta tabla siguiendo la recurrencia. La

podemos expresar también como que la media geométrica entre el

anterior y el siguiente a un término coincide con el cuadrado de ese

término al que se le ha restado una unidad.

Una propiedad similar es que la media geométrica entre un término y el

siguiente es también un número triangular. Lo tienes en esta tabla.

Escribe los triangulares cuadrados e intenta después reproducirla:

Page 120: Números y formas - Aprender y divertirse con hoja de …hojamat.es/publicaciones/numform.pdf · Este es uno de los capítulos de las Matemáticas en los que se aprecia ... también

120

En http://oeis.org/A029549 tienes estudiadas esas medias geométricas

y puedes descubrir que estos números son oblongos y también su

conexión con ciertas ternas pitagóricas. A partir de esta sucesión se

abren tantos caminos que es mejor parar aquí.

Por otra parte, por ser cuadrados, los términos son suma de dos

triangulares consecutivos, luego los triangulares cuadrados son

“triangulares suma de dos triangulares consecutivos”

Raíz cuadrada

Ya que tratamos con cuadrados, sería interesante estudiar sus raíces

cuadradas, que son estas:

0, 1, 6, 35, 204, 1189, 6930, 40391, 235416, 1372105, 7997214,

46611179, 271669860,…

(http://oeis.org/A001109)

Estos números no nos son desconocidos, pues son soluciones de la

incógnita Y en la ecuación de Pell que usamos para encontrar sus

cuadrados

𝑥2 − 8𝑦2 = 1

Insertamos de nuevo la tabla que obtuvimos:

Page 121: Números y formas - Aprender y divertirse con hoja de …hojamat.es/publicaciones/numform.pdf · Este es uno de los capítulos de las Matemáticas en los que se aprecia ... también

121

Más adelante veremos alores relacionados con la variable X.

Recurrencia entre las raíces

Al igual que sus cuadrados, estos números se pueden generar

mediante una recurrencia de segundo grado. Para descubrirla

operamos como en el apartado anterior.

Dn=aDn-1+bDn-2+c usaremos los valores iniciales 0, 1, 6, 35, 204 para

plantear:

6=a*1+b*0+c

35=a*6+b*1+c

204=a*35+b*6+c

Resolvemos

29=5a+b

169=29a+5b

169-5*29=169-145=24=4a a=6, b=-1, c=0

Luego Dn=6Dn-1-Dn-2, que es la recursión que figura en A001109

Esta recurrencia la podemos comprobar con nuestra hoja de cálculo

dedicada a ellas

http://www.hojamat.es/sindecimales/aritmetica/herramientas/herrarit.ht

m#recurre2

Escribimos los coeficientes 6 y -1

Y obtenemos la sucesión

Page 122: Números y formas - Aprender y divertirse con hoja de …hojamat.es/publicaciones/numform.pdf · Este es uno de los capítulos de las Matemáticas en los que se aprecia ... también

122

Orden como triangulares

Al igual que hemos estudiado las raíces cuadradas de los triangulares

cuadrados, también podemos fijar la atención en su orden como

triangulares.

Para ello planteamos k(k+1)/2=A(n), siendo A(n) un término de la

sucesión de triangulares cuadrados. Es fácil ver que la solución será

𝑘 = √8𝐴(𝑛) + 1 − 1

2

Se generará esta otra sucesión:

1, 8, 49, 288, 1681, 9800, 57121, 332928, 1940449, 11309768,

65918161, 384199200,… (http://oeis.org/A001108)

También estos números están relacionados con la ecuación de Pell del

anterior apartado.

𝑥2 − 8𝑦2 = 1

Page 123: Números y formas - Aprender y divertirse con hoja de …hojamat.es/publicaciones/numform.pdf · Este es uno de los capítulos de las Matemáticas en los que se aprecia ... también

123

Basta recordar que llamamos x a 2n+1. Deshaciendo el cambio en la

tabla:

(3-1)/2=1, (17-1)/2=8, (99-1)/2=49, (577-1)/2=288,… y así resultarán

todos.

Según OEIS, su fórmula recursiva es idéntica a la de los anteriores,

pero con término independiente igual a 2:

Dn=6Dn-1-Dn-2+2

Para no cansar a los lectores nos limitamos a comprobarla.

8=6*1-0+2, 49=6*8-1+2, 288=6*49-8+2,…

Diferencias entre términos

Si restamos cada dos términos consecutivos, el resultado coincide con

las raíces cuadradas de los términos de orden impar. Es una propiedad

muy curiosa, pero no he encontrado ninguna demostración elemental

de la misma.

En la tabla, si elevamos las diferencias al cuadrado nos resultan los

términos de orden impar. Son los de color rojo enlazados con flechas.

Si, además, encontráramos su orden como triangulares, sería un

cuadrado (compruébalo), y ellos mismos, además de triangulares

serían hexagonales. Bastante curioso, como ves.

Page 124: Números y formas - Aprender y divertirse con hoja de …hojamat.es/publicaciones/numform.pdf · Este es uno de los capítulos de las Matemáticas en los que se aprecia ... también

124

Recurrencia directa

Lekraj Beedassy, en http://oeis.org/A001110, propone la siguiente

recurrencia no lineal que sólo depende del término anterior:

𝑎𝑛+1 = 1 + 17𝑎𝑛 + 6√𝑎𝑛 + 8𝑎𝑛2

Esta propiedad permite engendrar de nuevo los números triangulares

cuadrados en una hoja de cálculo directamente, sin macros, y con gran

rapidez, con una fórmula similar a =1+17*I5+6*RAIZ(I5+8*I5^2), donde

puedes sustituir I5 por el término anterior de la sucesión.

Page 125: Números y formas - Aprender y divertirse con hoja de …hojamat.es/publicaciones/numform.pdf · Este es uno de los capítulos de las Matemáticas en los que se aprecia ... también

125

POLIGONALES

¿ERES UN POLIGONAL?

Si reuniéramos en una sola lista los números triangulares,

cuadrangulares, pentagonales, etc. ¿llenarían todo el conjunto de los

números naturales?

Podemos llamar orden n del número poligonal al número de unidades

incluidas en uno de sus lados, y tipo k, al número de lados.

Evidentemente, si consideramos los poligonales de orden 2, cualquier

número N se puede representar como un polígono de N lados, luego la

respuesta es afirmativa.

El problema es más interesante si sólo estudiamos números

poligonales de al menos orden 3.

Existen números que son triangulares, como el 10, pentagonales,

como el 22, o incluso algunos, como el 28, que son triangulares y

hexagonales simultáneamente, como puedes observar en la imagen:

¿Existirán números que sólo puedan considerarse como

poligonales de grado uno y no admitan otras representaciones

poligonales?

Page 126: Números y formas - Aprender y divertirse con hoja de …hojamat.es/publicaciones/numform.pdf · Este es uno de los capítulos de las Matemáticas en los que se aprecia ... también

126

Dado un número cualquiera, como 2011, sería interesante averiguar

qué representaciones admite como número poligonal. Se puede

abordar el problema desde varios puntos de vista. Veamos el más

sencillo:

Generación mediante triangulares

Todo número poligonal de orden n y tipo k se puede generar mediante

esta fórmula:

siendo Tn-1 el número triangular de orden n-1.

No es difícil justificar esta fórmula La simple visión de la siguiente

imagen te permite comprenderla. Las unidades azules representan a n,

y las de los otros tres colores a los números triangulares que terminan

de engendrar el pentagonal:

Así que para saber si un número es n-gonal bastará restarle el valor

de n y después averiguar si la diferencia contiene a Tn-1 un

número entero de veces.

En una hoja de cálculo se pueden organizar tres columnas: La primera

con los números naturales n, la segunda con sus sumas acumuladas,

que serían los números triangulares T, y en la tercera el cociente (N-

n)/T. Si este cociente es entero, hemos descubierto que el número

probado es n-gonal.

Page 127: Números y formas - Aprender y divertirse con hoja de …hojamat.es/publicaciones/numform.pdf · Este es uno de los capítulos de las Matemáticas en los que se aprecia ... también

127

En la imagen tienes el proceso para descubrir que el número 28 es 28-

gonal, hexagonal y triangular, como ya sabíamos. También podemos

comprobar que hay números, como el 2011, que sólo admiten formar

un polígono de grado 1 y tipo 2011. Sin embargo, el 2016 admite seis

representaciones, con tipos 2016, 673, 136, 24, 6 y 3.

Generación mediante fórmula

De la generación de números poligonales a partir de triangulares se

puede deducir la popular fórmula

Si la consideramos como ecuación de segundo grado en n, se puede

exigir que su discriminante sea cuadrado perfecto, es decir:

Esto nos da otro procedimiento: Recorremos valores de k y

observamos cuáles producen cuadrados perfectos y después si el valor

de n y k son enteros. En la imagen puedes observar cómo se organiza

la búsqueda en hoja de cálculo

Page 128: Números y formas - Aprender y divertirse con hoja de …hojamat.es/publicaciones/numform.pdf · Este es uno de los capítulos de las Matemáticas en los que se aprecia ... también

128

Según la anterior fórmula, los números primos p sólo pueden ser p-

gonales, es decir, de orden 2. Son como collares, con los elementos

situados todos en el perímetro. Hay otros números compuestos que

comparten esta misma propiedad. Están recogidos en

http://oeis.org/A176949:

4, 8, 14, 20, 26, 32, 38, 44, 50, 56, 62, 68, 74, 77, 80, 86, 98, 104, 110,

116, 119,…

Ninguno puede ser k-gonal para k<N

Con este sencillo código en el Basic de Excel puedes encontrar de

cuántas formas pueden expresarse los 28 primeros números como

poligonales (excluimos 0, 1 y 2)

Fila=15 For i = 3 to 28 a = 1 b = 0 For k = 2 To i – 1 c= (i - k) / a If c=int(c) Then b = b + 1 a = a + k Next k ActiveWorkbook.Sheets(1).Cells(fila, 6).Value = i ActiveWorkbook.Sheets(1).Cells(fila, 7).Value = b fila = fila + 1 Next i

Obtendríamos esta tabla

Page 129: Números y formas - Aprender y divertirse con hoja de …hojamat.es/publicaciones/numform.pdf · Este es uno de los capítulos de las Matemáticas en los que se aprecia ... también

129

Número Posibles

3 1

4 1

5 1

6 2

7 1

8 1

9 2

10 2

11 1

12 2

13 1

14 1

15 3

16 2

17 1

18 2

19 1

20 1

21 3

22 2

23 1

24 2

25 2

26 1

27 2

28 3

Observamos que en los primos sólo existe una posibilidad desechando

las trivialidades y que también ocurre lo mismo en 4, 8, 14… como se

afirmó más arriba.

Para quienes vayan conociendo PARI, tenemos este código

equivalente, aunque menos legible:

Page 130: Números y formas - Aprender y divertirse con hoja de …hojamat.es/publicaciones/numform.pdf · Este es uno de los capítulos de las Matemáticas en los que se aprecia ... también

130

isinteger(n)=(n==truncate(n)) {for(i=3,28,a=1;b=0;for(k=2,i-1,if(isinteger((i-k)/a),b=b+1);a=a+k);print(i," ",b))}

CUBOS Y GNOMONES

En alguna página web he vuelto a encontrar esta propiedad:

1 = 13

3+5 = 23

7+9+11 = 33

13+15+17+19 = 43

Independientemente de su elegancia, es una invitación a profundizar

en otras relacionadas con ella y a justificar rigurosamente su

existencia.

(1) La propiedad presentada está relacionada con otra bien conocida:

1=12

1+3=22

1+3+5=32

1+3+5+7=42

1+3+5+7+9=52

Saca consecuencias:

¿Se puede afirmar que todo cubo perfecto es diferencia de dos

cuadrados?

En caso afirmativo ¿Qué tipo de números son los que pueden formar

esa diferencia?

¿Podrías demostrarlo con todo rigor?

Page 131: Números y formas - Aprender y divertirse con hoja de …hojamat.es/publicaciones/numform.pdf · Este es uno de los capítulos de las Matemáticas en los que se aprecia ... también

131

(2) La propiedad considerada nos permite encontrar una expresión

algebraica para la suma de varios cubos consecutivos, por ejemplo

K3 + (k+1) 3 + (k+2) 3+ … + (k+r) 3

¿Sabrías encontrarla? Se pueden dar varias distintas. Si deseas

comprobar la que propongas usa la hoja de cálculo

Es esclarecedor observar la cuestión propuesta desde el punto de vista

geométrico. Si representamos la suma 7+9+11 como un embaldosado

compuesto de tres gnomones, n3 se adivina apilando baldosas:

En este caso se visualiza fácilmente

Prueba a convertir en cubo de esta forma otras sumas parecidas, como

13+15+17+19.

También es atractiva la idea de formar primero el cubo y después

convertirlo en suma de impares. Observa la figura:

Page 132: Números y formas - Aprender y divertirse con hoja de …hojamat.es/publicaciones/numform.pdf · Este es uno de los capítulos de las Matemáticas en los que se aprecia ... también

132

Otra vuelta

Hasta ahora hemos sumado números impares y potencias. La

demostración algebraica de las fórmulas de este tipo puede estar

sujeta a errores. Nadie puede decir que no se ha equivocado en un

desarrollo algebraico de dos hojas. Un método para comprobar

nuestros cálculos es el uso de una fórmula de interpolación.

En este caso un método de interpolación adecuado es el de Newton.

Se puede adaptar con cierta facilidad al caso de valores enteros

equidistantes. En la siguiente dirección puedes encontrar su

implementación en Excel y Calc:

http://www.hojamat.es/sindecimales/aritmetica/herramientas/herrarit.ht

m

Si aplicamos esta herramienta al caso de la suma de cubos tomando,

por ejemplo, como primer cubo el 27, nos resultaría esta expresión:

Suma=27 + 64(x-3)+61/2(x-3)(x-4)+5(x-3)(x-4)(x-5)+1/4(x-3)(x-4)(x-

5)(x-6)

Puedes comprobar su validez dando a x los valores 3, 4, 5, 6,7,… para

comprobar que se obtienen las sumas de cubos 27, 91, 216, 432,

775,…

Otro problema es el simplificar esta expresión, que también sería una

operación sujeta a errores. Si lo haces, observarás que efectivamente

la fórmula equivale a T2k+r

- T2

k-1.

Page 133: Números y formas - Aprender y divertirse con hoja de …hojamat.es/publicaciones/numform.pdf · Este es uno de los capítulos de las Matemáticas en los que se aprecia ... también

133

Nota: No resisto incluir la interpolación que se logra con la calculadora

en red WIRIS, porque es un gran auxiliar en este tipo de desarrollos:

¿Por qué entonces usar la hoja de cálculo? Yo tengo mi propia

respuesta, y es que resulta más divertido pedalear que echar gasolina.

Cuando ya tenía programadas las tres entradas sobre Cubos y

gnómones encontré esta curiosidad en el siempre interesante blog de

Claudio http://simplementenumeros.blogspot.com/

Me planteé buscar cocientes similares pero con suma de cubos. Para

ello, según lo que hemos visto en esas entradas, bastaría buscar

diferencias de cuadrados de números triangulares tales que las

diferencias entre sus índices fueran iguales dos a dos, como ocurre,

por ejemplo con T102 - T5

2 = 452-102 y T182 - T13

2

=1532-782, (10-5=5

y 18-13=5) y que, además, las dos diferencias fueran divisibles, como

ocurre en este ejemplo, en el que 1532-782=17325 es múltiplo de 452-

102=1925.

Después bastaría traducir las diferencias entre triangulares en sumas

de cubos, con lo que obtendríamos cocientes aparentemente

complicados con resultado simple. La exigencia de que las diferencias

entre índices sean iguales se debe a un deseo de simetría pero no es

imprescindible.

Con una tabla de doble entrada de cuadrados de triangulares o con

código Basic se encuentran fácilmente.

Presentamos cuatro de esos resultados, pero se pueden obtener

muchos más.

Page 134: Números y formas - Aprender y divertirse con hoja de …hojamat.es/publicaciones/numform.pdf · Este es uno de los capítulos de las Matemáticas en los que se aprecia ... también

134

Carácter de múltiplos de 5

Es fácil ver que aunque la expresión propuesta tiene denominador 6,

su resultado será entero, porque ese ha sido su origen, y porque los

factores k(k+1)(2k+1) garantizan un factor 2 y un 3. Estúdialo, que no

es difícil de descubrir.

¿De dónde sacamos el factor 5?

Lo podemos ver mediante congruencias módulo 5. El valor de k puede

presentar respecto al 5 los restos 0, 1, 2, 3 o 4.

Resto 0: En ese caso k contiene el factor 5

Resto 1: El factor 12k2+12k+1 será múltiplo de 5

Resto 2: Contamos con el factor 2k+1

Resto 3: El factor 12k2+12k+1 sería congruente con

12*9+12*3+1=108+36+1=145, múltiplo de 5

Resto 4: Nos proporciona el factor deseado el valor de k+1

En todos los casos la suma de cuadrados será un múltiplo de 5.

Hemos terminado con éxito. Nuestras sospechas tenían fundamento y

la sucesión 25, 365, 2030, 7230, 19855, 45955, 94220, 176460…

representa simplemente los distintos valores de un polinomio de quinto

grado definido sobre los números naturales.

Page 135: Números y formas - Aprender y divertirse con hoja de …hojamat.es/publicaciones/numform.pdf · Este es uno de los capítulos de las Matemáticas en los que se aprecia ... también

135

POLIGONALES CENTRADOS

Los números poligonales ordinarios se engendran acumulando

distintos polígonos a partir de un vértice, como podemos ver en la

figura

Los poligonales centrados son similares, pero los polígonos se

acumulan alrededor de un centro, con sus lados paralelos

Todos los poligonales de esta clase se pueden pues generar mediante

sumas de números naturales que representen los contornos de los

polígonos. En el caso de los triángulos serían 1+3+6+9+12+…

Efectuando sumas parciales obtendríamos la sucesión 1, 4, 10, 19,

31,.., a la que nombraremos como “números triangulares centrados”.

En el caso de cuadrados se formaría la suma 1+4+8+12+16+…, y las

sumas formarían la sucesión 1, 5, 13, 25, 41,…, que serían los

números “cuadrados centrados”.

Con el mismo método resultarían los “pentagonales centrados”, a partir

de la suma 1+5+10+15+20…, que serían 1, 6, 16, 31, 51,…

En general, los polígonos de orden n y lado k, al sumarse formarían:

1+n+2n+3n+4n+...+kn=1+n*(1+2+3+4+5+...+k)=1+n*k*(k+1)/2

Page 136: Números y formas - Aprender y divertirse con hoja de …hojamat.es/publicaciones/numform.pdf · Este es uno de los capítulos de las Matemáticas en los que se aprecia ... también

136

Si contamos el 1 como primer elemento, la suma del paréntesis tendría

un elemento menos, y daría la expresión, si llamamos POLC al

poligonal centrado:

POLC(n,k)=1+n*k*(k-1)/2=(n*k2-n*k+2)/2

Es decir:

En ella n representa el tipo de poligonal y k el lado.

Así, POLC(5,4)=(5*16-5*4+2)/2=62/2=31, tal como vimos en el listado

de pentagonales.

POLC(3,5)=(3*25-3*5+2)/2=62/2=31, que sería el quinto triangular que

obtuvimos más arriba.

Calculadora CALCUPOL

Estos cálculos se pueden evitar con nuestra calculadora de números

figurados, “Calcupol”. Esta calculadora Se descarga desde

http://www.hojamat.es/sindecimales/aritmetica/herramientas/herrarit.ht

m#figurados

Con la tecla CEN (de centrado) y la secuencia de teclas 3 CEN 5 =

obtendríamos el triangular centrado de lado 5, que ya sabemos vale

31.

Page 137: Números y formas - Aprender y divertirse con hoja de …hojamat.es/publicaciones/numform.pdf · Este es uno de los capítulos de las Matemáticas en los que se aprecia ... también

137

Usaremos esta calculadora varias veces en este tema.

Relación con los poligonales ordinarios

Si recordamos la fórmula de los números poligonales

k((n-2)*k-(n-4))/2

y la comparamos con la de los centrados

(n*k2-n*k+2)/2

resulta:

(n*k2-n*k+2)/2-(k2*(n-2)-k*(n-4)/2 =

=( n*k2-n*k+2-n*k2+2k2+n*k-4k)/2 =

= (2+2k2-4k)/2=(k-1)2

Así que si conocemos un poligonal ordinario, bastará sumarle el

cuadrado del lado después de restarle una unidad. Lo vemos con

Calcupol. El número poligonal de orden 7 (heptagonal) de lado 10 tiene

un valor de 235 (secuencia de teclas 7 POL 10 =) y si le añadimos (10-

1)2, obtenemos 235+81=316, que es el poligonal centrado del mismo

orden y lado. Puedes comprobarlo con la secuencia 7 CEN 10 =

Relación con los triangulares ordinarios

La expresión

𝑃𝑂𝐿𝐶(𝑛, 𝑘) =𝑛𝑘2 − 𝑛𝑘 + 2

2

Page 138: Números y formas - Aprender y divertirse con hoja de …hojamat.es/publicaciones/numform.pdf · Este es uno de los capítulos de las Matemáticas en los que se aprecia ... también

138

Se puede escribir así:

𝑃𝑂𝐿𝐶(𝑛, 𝑘) = 𝑛𝑘(𝑘 − 1)

2+ 1 = 𝑛𝑇(𝑘 − 1) + 1

Esto nos indica que un poligonal centrado se forma añadiendo a 1 n

números triangulares de lado n-1. En el caso, por ejemplo, del

pentagonal de lado 4, se podrá descomponer en cinco triángulos de

lado 3 y una unidad. La siguiente imagen, adaptación de otra de la

Wikipedia, nos muestra claramente los triángulos:

Números triangulares centrados

Comenzamos el estudio particularizado para cada orden, siendo n=3 el

caso de menos lados. Ya se comentó más arriba que los triangulares

centrados se forman mediante triángulos concéntricos como los de la

imagen:

Se considera la unidad como primer triángulo.

Page 139: Números y formas - Aprender y divertirse con hoja de …hojamat.es/publicaciones/numform.pdf · Este es uno de los capítulos de las Matemáticas en los que se aprecia ... también

139

Ya se vio que se forman mediante la suma 1+3+6+9+12+...+3k, que da

lugar a la expresión 1+3*k(k-1)/2, con lo que los primeros triangulares

centrados serán; 1, 1+3, 1+3+6, 1+3+6+9,….es decir: 1, 4, 10, 19,…

Completamos la sucesión con más términos:

1, 4, 10, 19, 31, 46, 64, 85, 109, 136, 166, 199, 235, 274, 316, 361,

409, 460, 514, 571, 631, 694, 760, 829, 901,… Están publicados en

http://oeis.org/A005448

Con nuestra calculadora Calcupol puedes recorrerlos uno a uno.

Fijas en la parte derecha “Centrado” y orden 3. Borras la pantalla con

CA y escribes 1. Después, cada vez que pulses PROX, aparecerán los

siguientes términos: 4, 10, 19, 31,...

Es interesante deducir la expresión del término general, que ya

conocemos, mediante nuestro interpolador lineal para números

naturales

(lo puedes descargar desde

http://www.hojamat.es/sindecimales/aritmetica/herramientas/herrarit.ht

m#newton)

Si la aplicamos a los números 1, 4, 10, 19, 31, 46, 64 descubrimos que

sus diferencias de tercer orden son nulas, y que esto los convierte en

valores de un polinomio de segundo grado.

Page 140: Números y formas - Aprender y divertirse con hoja de …hojamat.es/publicaciones/numform.pdf · Este es uno de los capítulos de las Matemáticas en los que se aprecia ... también

140

La herramienta de interpolación nos proporciona también los

coeficientes en las filas inferiores.

Si conoces la interpolación de Newton entenderás que el polinomio

buscado es

1/1+3/1(x-1)+3/2(x-1)(x-2)=1+3x(x-1)/2

Esto comprueba lo indicado más arriba.

Propiedades

(1) A partir del 10, todos los triangulares centrados son suma de

tres triangulares consecutivos.

TC(n)=T(n)+T(n-1)+T(n-2)

Es una cuestión de Álgebra:

T(x-2)+T(x-1)+T(x)=(x-2)(x-1)/2+(x-1)x/2+x(x+1)/2=(x2-3x+2+x2-

x+x2+x)/2=(3x2-3x+2)/2=1+3x(x-1)/2= TC(x)

Por inducción

Se cumple para TC(4)=10=1+3+6=T(1)+T(2)+T(3)

Si se cumple para x, será TC(x)=T(x-2)+T(x-1)+T(x). Si añadimos 3x,

se convertirá en TC(x+1), y bastará demostrar que T(x-2) se convierte

en T(x+1)

En efecto:

Page 141: Números y formas - Aprender y divertirse con hoja de …hojamat.es/publicaciones/numform.pdf · Este es uno de los capítulos de las Matemáticas en los que se aprecia ... también

141

(x-2)(x-1)/2+3x=(x2-3x+2+6x)/2=(x2+3x+2)/2=T(x+1)

Piénsalo bien, sólo hay que convertir T(x-2) en T(x+1)

(2) Relación con números combinatorios

TC(n+1) = C(n+3, 3)-C(n, 3)

Otra cuestión de Álgebra:

C(n+3, 3)-C(n, 3)=((n+3)(n+2)(n+1)-n(n-1)(n-2))/6

(n3+6n2+11n+6-n3+3n2-2n)/6=(9n2+9n+6)/6=1+3n(n+1)/2=TC(n+1)

Puedes ir restando números combinatorios situados debajo de la línea

continua, con un salto de tres lugares, e irán resultando los triangulares

centrados:

20-1=19; 35-4=31; 56-10=46; 84-20=64; 120-35=85…

(3) Triangulares centrados primos

Entre los triangulares de este tipo existen algunos que son primos.

Sólo es una curiosidad. Los primeros son:

19, 31, 109, 199, 409, … https://oeis.org/A125602

Page 142: Números y formas - Aprender y divertirse con hoja de …hojamat.es/publicaciones/numform.pdf · Este es uno de los capítulos de las Matemáticas en los que se aprecia ... también

142

(4) Recurrencia

Todas las sucesiones de números figurados admiten recurrencias, por

ser fórmulas polinómicas. Los triangulares centrados también poseen

una generación por recurrencia, además de la contenida en la

definición, de añadir 3n al término anterior:

Elegimos esta:

TC(n) = 3*TC(n-1) - 3*TC(n-2) + TC(n-3), TC(1)=1, TC(2)=4, TC(3)=10.

Se cumple para el 19=3*10-3*4+1=30-12+1=19

Otro término lo cumplirá por simples cálculos algebraicos.

3*TC(n-1) - 3*TC(n-2) + TC(n-3)=3*(1+3(n-1)(n-2)/2)-3*(1+3(n-2)(n-

3)/2)+1+3(n-3)(n-4)/2

Si no te apetece simplificar acude a cualquier CAS. Nosotros hemos

usado Wolfram Alpha

Hemos obtenido la expresión

que coincide con 1+3n(n-1)/2, expresión de TC(n)

Existen otras recurrencias, que puedes intentar demostrar:

a(n) = a(n-1) + 3*n-3. - (Vincenzo Librandi)

a(n) = 2*a(n-1) - a(n-2) + 3. - (Ant King)

Page 143: Números y formas - Aprender y divertirse con hoja de …hojamat.es/publicaciones/numform.pdf · Este es uno de los capítulos de las Matemáticas en los que se aprecia ... también

143

Cuadrados centrados

Vimos en los primeros párrafos que la suma 1+4+8+12+16+…, que se

forma añadiendo 4 unidades a cada sumando, forma, con sus sumas

parciales, la sucesión 1, 5, 13, 25, 41,…, que serían los números

“cuadrados centrados”.

Los primeros términos son:

1, 5, 13, 25, 41, 61, 85, 113, 145, 181, 221, 265, 313, 365, 421, 481,

545, 613, 685, 761, 841, 925, 1013, 1105, 1201, 1301, 1405, 1513,

1625, 1741, 1861, 1985, 2113, 2245, 2381,...y están publicados en

http://oeis.org/A001844

Con nuestra calculadora Calcupol puedes recorrerlos uno a uno. La

puedes descargar desde

http://www.hojamat.es/sindecimales/aritmetica/herramientas/herrarit.ht

m#figurados

Fijas en la parte derecha “Centrado” y orden 4. Borras la pantalla con

CA y escribes 1

Como en el caso de los triangulares, con cada pulsación de la tecla

PROX irás obteniendo los siguientes cuadrados centrados: 5, 13, 25,...

Si en la expresión de los poligonales centrados

sustituimos n por 4 nos resultará la expresión de los cuadrados

centrados:

Page 144: Números y formas - Aprender y divertirse con hoja de …hojamat.es/publicaciones/numform.pdf · Este es uno de los capítulos de las Matemáticas en los que se aprecia ... también

144

Por ejemplo CC(4)=2*16-2*4-1=32-8+1=25

Esta fórmula presenta una interpretación sencilla, pues equivale a la

suma de dos cuadrados consecutivos. Así, 25=16+9, o 13=9+4. Si

recordamos que los cuadrados son sumas de impares, con esta

propiedad podemos engendrar los cuadrados centrados como una

suma creciente y decreciente de impares. Lo vemos con el 61:

61=1+3+5+7+9+11+9+7+5+3+1

Con un poco de Álgebra es fácil ver que es válida también esta otra

expresión:

Así, el término 5 equivaldrá a (81+1)/2=41, como puedes comprobar en

el listado. También 41 es suma de cuadrados: 41=26+16

Lo podemos expresar también como que el doble de un cuadrangular

menos una unidad es un cuadrado perfecto. Esto convierte a un

cuadrado centrado N en la hipotenusa de un triangulo rectángulo

con un cateto igual a N-1. En efecto, N²-(N-1)² =2N-1 es un cuadrado.

Por ejemplo:

41² - 40² = 1681 – 1600 = 81 = 9²

Pentagonales centrados

Al igual que en los casos anteriores, partimos de la sucesión formada

por el 1 y los múltiplos de 5, ya que en un pentagonal centrado se van

añadiendo polígonos de cinco lados aumentando una unidad en cada

caso: 1+5+10+15+20. Las sumas parciales formarán los pentagonales

centrados (PC(n)):

1, 6, 16, 31, 51,…

Los primeros pentagonales centrados son:

Page 145: Números y formas - Aprender y divertirse con hoja de …hojamat.es/publicaciones/numform.pdf · Este es uno de los capítulos de las Matemáticas en los que se aprecia ... también

145

1, 6, 16, 31, 51, 76, 106, 141, 181, 226, 276, 331, 391, 456, 526, 601,

681, 766, 856, 951, 1051, 1156, 1266, 1381, 1501, 1626, 1756, 1891,

2031, 2176, 2326, 2481, 2641, 2806, 2976,…

Están publicados en http://oeis.org/A005891

Para conseguir su expresión podemos acudir a la interpolación

polinómica. Como ya la hemos usado en casos anteriores, sólo

insertaremos una captura de pantalla:

Vemos que dará lugar a un polinomio de segundo grado. Leemos los

coeficientes:

P(x)=1+5*(x-1)+5/2*(x-1)(x-2)=(5n^2+5n+2)/2

Así que

Basta observar la fórmula para darse cuenta de que todos estos

números son congruentes con la unidad módulo 5. Así 16=5*3+1,

76=5*15+1,...Ya sabemos que sus diferencias son múltiplos de 5.

También es sencillo comprobar que los coeficientes del 5 en la anterior

expresión son todos números triangulares, ya que PC(n)=5n(n+1)/2+1.

Page 146: Números y formas - Aprender y divertirse con hoja de …hojamat.es/publicaciones/numform.pdf · Este es uno de los capítulos de las Matemáticas en los que se aprecia ... también

146

Hexagonales centrados

La definición de estos números coincide con la de los anteriores, pero

añadiendo a cada uno de ellos un hexágono nuevo (o múltiplo de 6)

Dejamos como ejercicio comprobar que su expresión es

HC(n)=(n+1)3-n3

Con ella podemos desarrollar la sucesión de hexagonales centrados:

1, 7, 19, 37, 61, 91, 127, 169, 217, 271, 331, 397, 469, 547, 631, 721,

817, 919, 1027, 1141, 1261, 1387, 1519, 1657, 1801, 1951, 2107,

2269, 2437, 2611, 2791, 2977, 3169, 3367, 3571, 3781, 3997

(http://oeis.org/A003215)

La expresión obtenida equivale claramente a una diferencia de cubos

consecutivos. En efecto, 7=2³-1³, 19=3³-2³=27-8, 37=4³-3³=64-27

Propiedad combinatoria

Sumas iguales a cero

Benoit Cloitre propone en la página OEIS citada que los números de la

sucesión se corresponden con el número de tripletes ordenados de

enteros (a,b,c),con -n <= a,b,c <= n, tales que a+b+c=0. Esta propiedad

está expresada si el primer índice de la sucesión es cero, por lo que

debemos aplicarla a n-1.

Por ejemplo, en el caso de n=3, HC(3)=19 coincidirá con el número de

sumas de tres sumandos comprendidos entre -2 y 2 cuya suma sea 0.

Podemos comprobar esta propiedad con nuestra hoja Cartesius

http://www.hojamat.es/sindecimales/combinatoria/herramientas/herrco

mb.htm#cartesius

El planteo sería muy simple:

xtotal=3

Xt=-2..2

Page 147: Números y formas - Aprender y divertirse con hoja de …hojamat.es/publicaciones/numform.pdf · Este es uno de los capítulos de las Matemáticas en los que se aprecia ... también

147

Suma=0

Aunque no hayas usado nunca esta hoja Cartesius, entenderás que se

fija un número de sumandos igual a 3, comprendidos entre -2 y 2 y

cuya suma sea 0.

Introducimos este planteo en la hoja

Pulsamos el botón Iniciar y obtenemos las 19 sumas esperadas:

Esta propiedad se puede demostrar por inducción. Ya hemos

comprobado para n=3. Para n=2 basta con que recorras el listado de

sumas y te quedes con las que tienen máximo 1. Las contamos y

resultan 7, y es trivial que para el caso n=1 sólo obtenemos un caso.

Con esto se comprueba para los casos 1, 2 y 3.

Page 148: Números y formas - Aprender y divertirse con hoja de …hojamat.es/publicaciones/numform.pdf · Este es uno de los capítulos de las Matemáticas en los que se aprecia ... también

148

Para el caso n, podemos pasar al caso n+1, con lo que hay que añadir

los elementos -(n+1) y n+1. Las nuevas sumas pueden ser de tres

clases:

Si contienen -(n+1) y n+1, el tercer sumando será 0, y reordenando nos

resultan 6 sumas nuevas.

Si sólo contiene el sumando -(n+1), deberá estar acompañado por

todos los sumandos positivos entre 1 y n que sumen n+1. Existen n

sumas ordenadas de ese tipo, y el sumando -(n+1) se puede situar en

3 posiciones, luego aparecerán 3n sumas nuevas.

El tercer caso también abarcará 3n sumas. Reunimos los tres casos y

nos resulta 3n+3n+6=6(n+1), luego efectivamente, se añadirá un

múltiplo de 6 al término anterior, lo que lo convierte en el siguiente

hexagonal centrado.

Una propiedad aritmética

Las medias parciales de los k primeros términos coinciden con k².

Está basada en un inicio para n=0, por lo que usaremos n en lugar de

n+1 en la demostración.

Esta propiedad se verifica en los primeros términos:

(1+7)/2 = 4 = 2²

(1+7+19)/3 = 9 = 3²

Si lo suponemos cierto para n, deberemos demostrar que la siguiente

media coincide con (n+1)². Usaremos la expresión general aplicada al

término n.

M(n+1)=S(n+1)/(n+1)=(S(n)+3n²+3n+1)/(n+1)=(n*n²+3n²+3n+1)/(n+1) =

(n+1)³/(n+1) = (n+1)².

Es evidente que hemos demostrado de paso que las sumas parciales

coinciden con n³

Aquí dejamos los poligonales centrados.

Con esta muestra podrás investigar más sobre el tema.

Page 149: Números y formas - Aprender y divertirse con hoja de …hojamat.es/publicaciones/numform.pdf · Este es uno de los capítulos de las Matemáticas en los que se aprecia ... también

149

NÚMEROS PIRAMIDALES

POLIGONALES Y PIRAMIDALES

Repaso de los números poligonales

Los números piramidales son una extensión natural de los poligonales,

por lo que puede ser adecuado comenzar con un repaso de estos. Lo

más importante que hay que recordar ahora es su formación

recurrente. Por ejemplo, los triangulares se forman añadiendo un lado

nuevo a los ya formados en el anterior triangular, como queda claro en

la imagen:

Es decir, que

t1 = 1 = 1

t2 = 1+2 = 3

t3 = 1+2+3 = 6

t4 = 1+2+3+4 = 10

En general, Tn+1=Tn+n, lo que convierte a los triangulares en sumas de

números consecutivos. Por eso Tn=1+2+3+…+n=n(n+1)/2.

Hemos preparado una hoja de cálculo con Calcupol, una calculadora

especializada en números figurados, que puedes descargar desde

http://www.hojamat.es/sindecimales/aritmetica/herramientas/herrarit.ht

m#figurados

Page 150: Números y formas - Aprender y divertirse con hoja de …hojamat.es/publicaciones/numform.pdf · Este es uno de los capítulos de las Matemáticas en los que se aprecia ... también

150

En ella, con la tecla POL puedes encontrar el k-ésimo número

triangular. Por ejemplo, con la secuencia de teclas 3 POL 12 =

encontrarás el triangular número 12, que resulta ser 78, como se ve en

la imagen:

El presentar la calculadora en este momento se justifica porque la

vamos a usar en toda una serie de entradas. Otra utilidad que tiene es

la de identificar si un número es de un tipo dado o no. Observa la celda

Tipos. Si fijas el tipo en Triangular (usa la lista desplegable) podrás

averiguar si el número que escribas en pantalla es o no triangular, con

la tecla ES, o bien encontrar el próximo o el anterior con PROX y ANT.

Ya las iremos viendo. Fija el tipo en triangular, escribe 75 y pulsa la

tecla ES. Te responderá que no es de ese tipo y en pantalla aparecerá

un cero. Si hubieras escrito 78, te devolvería 12, que es su número de

orden, o lado.

De igual forma se definen los números cuadrados, pero ahora, a cada

elemento le añadimos dos lados, formando lo que se llama un gnomon,

de fórmula 2n+1:

En la figura se observa la generación de cada número cuadrado:

C1 = 1 = 1

C2 = 1+3 = 4

Page 151: Números y formas - Aprender y divertirse con hoja de …hojamat.es/publicaciones/numform.pdf · Este es uno de los capítulos de las Matemáticas en los que se aprecia ... también

151

C3 = 1+3+5 = 9

C4 = 1+3+5+7 = 16

Los primeros números cuadrados son: 1, 4, 9, 16, 25,… como bien

sabemos, y, según se acaba de ver, son suma de impares

consecutivos. Fija la calculadora en el tipo Cuadrado. Escribe un 1 en

pantalla y ve pulsando reiteradamente la tecla PROX. Obtendrás esa

secuancia 1, 4, 9, 16, 25,… En la imagen se había llegado al 36:

El resto de poligonales se define de la misma forma que los cuadrados

y los triangulares, como números que forman pentágonos, hexágonos,

o de más lados. Basta ir añadiendo n-2 lados nuevos, 3 para los

pentagonales, 4 para los hexagonales, y así con los demás.

Escribe en la calculadora que el tipo es Poligonal y el orden 5 y podrás

analizar los pentagonales. Con la tecla PROX (o la ANT) puedes

recorrerlos. Comprueba que los primeros pentagonales son 1, 5, 12,

22, 35, 51, 70, 92,… En la imagen se ha llegado, con la tecla PROX, al

siguiente a 92, que es el 117

Page 152: Números y formas - Aprender y divertirse con hoja de …hojamat.es/publicaciones/numform.pdf · Este es uno de los capítulos de las Matemáticas en los que se aprecia ... también

152

Con este repaso ya estamos en condiciones de comenzar el estudio de

los números piramidales.

Números piramidales

Al igual que los poligonales se generan añadiendo a cada uno de ellos

lados nuevos, los piramidales se forman mediante números

poligonales nuevos que van haciendo el papel de bases de una

pirámide.

Tomemos, por ejemplo, los números triangulares, 1, 3, 6, 10,…

Imaginemos que comenzamos por 1 (siempre se comienza con él), que

hará el papel de vértice, y después le adosamos como base el

siguiente triangular, 3, y después el siguiente, 6, y así hasta que

obtengamos el orden deseado. Lo puedes ver en la imagen.

Para ver otras imágenes similares para los casos de cuadrados,

pentagonales o hexagonales entra en Mathword:

http://mathworld.wolfram.com/PyramidalNumber.html

Page 153: Números y formas - Aprender y divertirse con hoja de …hojamat.es/publicaciones/numform.pdf · Este es uno de los capítulos de las Matemáticas en los que se aprecia ... también

153

A los números poligonales de orden 3 (triangulares) les llamaremos

tetraédricos, a los de orden 4, piramidales cuadrados, y al resto,

pentagonales, hexagonales, y así hasta el orden que deseemos.

Usamos la palabra orden para no crear confusión con la calculadora

que ofrecemos. Llamaremos lado al número de poligonales que se

acumulan.

Con nuestra calculadora calcupol podemos seguir cualquiera de estas

sucesiones. Por ejemplo, para ver los piramidales hexagonales, fijamos

el tipo en Piramidal y el orden en 6. Escribimos un 1 en pantalla y

vamos pulsando la tecla PROX. Aparecerán los piramidales 1, 7, 22,

50,…En la imagen hemos llegado hasta 372:

Puedes comprobar los resultados obtenidos en la dirección

http://oeis.org/A002412

Si tienes un piramidal en pantalla, como puede ser el hexagonal 715,

de lado 10, con la secuencia de teclas – ANT = puedes restarle el

anterior, de lado 9, y te dará 190, que es precisamente el poligonal de

tipo 6 y lado 10. Para comprobarlo usa la secuencia de teclas 6 POL

10, y te resultará 190.

Ya estamos en condiciones de sintetizar la generación de los números

piramidales:

El número piramidal de orden k y lado n equivale a la suma del

piramidal de idéntico orden y un lado menos y el poligonal de mismo

orden y lado.

Page 154: Números y formas - Aprender y divertirse con hoja de …hojamat.es/publicaciones/numform.pdf · Este es uno de los capítulos de las Matemáticas en los que se aprecia ... también

154

Si nombramos los piramidales como PIR y los poligonales como POL,

se podría expresar así:

PIR(N,K)=PIR(N-1,K)+POL(N,K)

Por ejemplo (lo puedes ir calculando con Calcupol): El octavo piramidal

hexagonal es 372, y el poligonal hexagonal de lado nueve es 153. Si

los sumamos obtenemos el noveno piramidal hexagonal, ya que

372+153=525, que es el piramidal esperado.

Fórmula

Existe una expresión general para calcular PIR(N,K). De todas las

versiones publicadas nos quedamos con la siguiente:

𝑃𝐼𝑅(𝑛, 𝑘) =3𝑛2 + 𝑛3(𝑘 − 2) − 𝑛(𝑘 − 5)

6

Es un polinomio de tercer grado, al igual que los poligonales se

expresan con uno de segundo (ver

http://www.hojamat.es/sindecimales/aritmetica/teoria/teorarit.pdf)

Tienes una demostración en http://oeis.org/wiki/Pyramidal_numbers

Lo comprobamos con 372, pirámide hexagonal de lado 8:

PIR(8,6)=(3*64+512*4-8*1)/6=2232/6=372

Con un poco de Álgebra, se puede extraer de esta fórmula el factor

n(n+1)/2, que es, precisamente, el número triangular del mismo lado

que el piramidal que estamos calculando. La fórmula quedaría

entonces así:

𝑃𝐼𝑅(𝑛, 𝑘) = 𝑇𝑛

𝑛(𝑟 − 2) − (𝑟 − 5)

3

Page 155: Números y formas - Aprender y divertirse con hoja de …hojamat.es/publicaciones/numform.pdf · Este es uno de los capítulos de las Matemáticas en los que se aprecia ... también

155

Lo comprobamos: El vigésimo piramidal octogonal, según Calcupol, es

8190. El triangular del mismo lado 20, 210. Aplicamos la fórmula:

8190=210*(20*6-3)/3=210*117/3=24570/3=8190

TETRAEDROS.

A los números piramidales triangulares se les conoce también como

tetraédricos, o simplemente tetraedros (abreviado, TET), en recuerdo

del primer poliedro regular. Todos ellos se forman a partir del 1

adosando los distintos números triangulares, 3, 6, 10, 15, 21, 28… por

lo que también podemos decir que los tetraédricos equivalen a las

sumas parciales de los triangulares.

TET(1)=1

TET(2)=1+3=4

TET(3)=4+6=1+3+6=10

TET(4)=10+10)1+3+6+10=20

La lista de los primeros será, pues:

1, 4, 10, 20, 35, 56, 84, 120, 165, 220, 286, 364, 455, 560, 680, 816,

969, 1140, 1330, 1540, 1771, 2024, 2300, 2600, 2925,…

http://oeis.org/A000292

La puedes reproducir con la calculadora Calcupol que presentamos

anteriormente.

Page 156: Números y formas - Aprender y divertirse con hoja de …hojamat.es/publicaciones/numform.pdf · Este es uno de los capítulos de las Matemáticas en los que se aprecia ... también

156

http://www.hojamat.es/sindecimales/aritmetica/herramientas/herrarit.ht

m#figurados

Basta fijar el tipo en Piramidal, el orden en 3, y escribir en pantalla 1.

Con esto, cada vez que pulses en la Tecla PROX se formará un nuevo

piramidal tetragonal. En la imagen llegamos hasta 969:

La formula para estos números se simplifica mucho. Recordamos la

expresión general para todos los piramidales:

𝑃𝐼𝑅(𝑛, 𝑘) =3𝑛2 + 𝑛3(𝑘 − 2) − 𝑛(𝑘 − 5)

6

Hacemos k=3 y queda:

𝑇𝐸𝑇(𝑛) = 𝑃𝐼𝑅(𝑛, 3) =3𝑛2 + 𝑛3 + 2𝑛

6=

𝑛(𝑛 + 1)(𝑛 + 2)

6

Esta expresión nos suena familiar, y es que equivale al número

combinatorio n+2 sobre 3:

𝑇𝐸𝑇(𝑛) = (𝑛 + 2

3)

Por ejemplo, el tetragonal de orden 7 es 84, y equivale a

(7*8*9)/6=504/6=84

Puedes usar la expresión de hoja de cálculo COMBINAT, para calcular

el número combinatorio:

Page 157: Números y formas - Aprender y divertirse con hoja de …hojamat.es/publicaciones/numform.pdf · Este es uno de los capítulos de las Matemáticas en los que se aprecia ... también

157

Y ya, por repasar más detalles, con nuestra calculadora combinatoria

puedes usar las teclas 9 C 3:

La tienes en la dirección

http://www.hojamat.es/sindecimales/combinatoria/herramientas/herrco

mb.htm#calcucomb

Por ser números combinatorios de orden 3, los tetraedros se situarán

en la cuarta fila del triángulo de Pascal:

(Imagen adaptada de otra contenida en Wikipedia.es)

Por cierto, y como era de esperar, los números triangulares se

presentan en la anterior paralela.

Page 158: Números y formas - Aprender y divertirse con hoja de …hojamat.es/publicaciones/numform.pdf · Este es uno de los capítulos de las Matemáticas en los que se aprecia ... también

158

Curiosidades

Al igual que hicimos con cuestiones similares, desarrollaremos a

continuación algunas propiedades, muchas de ellas tomadas de

http://oeis.org/A000292

Suma de diferencias

Si, como es costumbre en oeis.org, comenzamos la sucesión por el 0,

resulta que cada número tetragonal de orden n es suma de todas las

diferencias b-a que se pueden formar entre los números 1, 2, 3, … n

entre sí, si b>=a (Amarnath Murthy, May 29 2003). Lo vemos mejor con

un esquema de esas diferencias. La imagen contiene el desarrollo para

el tetragonal 20:

Las cabeceras de filas y columnas están formadas por los números del

1 al 5. En el centro figuran las diferencias entre ellos sin contar las

negativas, y a la derecha figuran sus sumas por filas. El número 20

resulta como suma de todas las diferencias del interior. Como ese

número, por definición, es suma de triangulares, se formará a partir de

10+6+3+1, que son triangulares porque cada uno es suma de enteros

consecutivos 1, 2,… k.

Suma de productos

El mismo autor, Amarnath Murthy, nos propone otra igualdad

interesante, y es proceder a multiplicar todos los sumandos posibles p

y q cuya suma es n+1, y todos los productos también sumarán un

número tetragonal. En este caso es más una curiosidad algebraica que

aritmética, pues se justifica así:

Suma de productos p*q con p+q=n+1:

1(n+1-1)+2(n+1-2)+3(n+1-3)+…+n(n+1-n)=n(n+1)(n+1)/2-1^2-2^2-

…n^2

Page 159: Números y formas - Aprender y divertirse con hoja de …hojamat.es/publicaciones/numform.pdf · Este es uno de los capítulos de las Matemáticas en los que se aprecia ... también

159

Pero la suma de cuadrados es n(n+1)(2n+1)/6

Restamos y queda n(n+1)(n+1)/2- n(n+1)(2n+1)/6 =n(n+1)(3n+3-

2n+1)/6=n(n+1)(n+2)/6

Como es la expresión del tetragonal de orden n, ya lo tenemos

demostrado. Un esquema con hoja de cálculo aclara bastante el

proceso. En este caso no se considera el 0 como inicio de la sucesión:

Suma de cuadrados

Un número tetragonal, si tiene lado par n, coincide con la suma de los

cuadrados de todos los números pares comprendidos entre 1 y n/2.

Por ejemplo:

El tetragonal 56=TET(6) equivale a la suma

2^2+4^2+6^2=4+16+36=56. Basta aplicar la suma de cuadrados

consecutivos, ((n(n + 1)(2n + 1)) / 6), al caso n/2 y después multiplicar

por el factor común 2^2=4. En cuanto desarrollemos obtenemos

TET(n)= n(n+1)(n+2)/6:

Suma cuadrados pares es 4(n/2)(n/2+1)(n+1)/6=n(n+1)(n+2)/6

Page 160: Números y formas - Aprender y divertirse con hoja de …hojamat.es/publicaciones/numform.pdf · Este es uno de los capítulos de las Matemáticas en los que se aprecia ... también

160

Si es impar, bastará sumar el mismo número de cuadrados impares.

Como 35=1^2+3^2+5^2=1+9+25

Basta ver que la suma de ambos daría la de todos los cuadrados, en

este caso, 56+35=91, que coincide con (6*7*(2*6+1))/6=7*13=91, luego

es válida la posibilidad de sumar cuadrados impares.

Fórmula de recurrencia

Al tener expresión algebraica sencilla, los números tetragonales

permiten fácilmente una expresión recurrente. En concreto es:

TET(n) = n + 2*TET(n-1) - TET(n-2)

La demostración es inmediata: n + 2*TET(n-1) - TET(n-2)=n+2*(n-

1)*n*(n+1)/6-(n-2)(n-1)n/6 y simplificando llegamos a

n(n+1)(n+2)/6=TET(n)

Si disponemos estos números en columna, podemos aplicar a cada

dos de ellos esta fórmula de recurrencia:

Las líneas nos indican que 35 depende de los dos anteriores, 10 y 20,

y de su número de orden 5, mediante la opeación 35 = 5+2*20-10 = 35

Equivalencia con un triangular

Una cuestión interesante es estudiar la posibilidad de reducir un

número tetraédrico a un triangular puro, como si “aplanáramos” la

pirámide hasta convertirla en un número triangular con un lado distinto.

Basta aplicar el criterio para que m sea triangular, y es que 8m+1 sea

Page 161: Números y formas - Aprender y divertirse con hoja de …hojamat.es/publicaciones/numform.pdf · Este es uno de los capítulos de las Matemáticas en los que se aprecia ... también

161

cuadrado. Pues bien. Aplicando ese criterio, sólo se han encontrado

cinco números tetraédricos que sean también triangulares: 1, 10, 120,

1540 y 7140. Si aplicamos el criterio al cuarto, nos queda:

8*1540+1 = 12321 = 111^2

Por cierto, la equivalencia con un cuadrado aún es más escasa: sólo

son cuadrados los tetraédricos 1, 4 y 19600.

CUADRADOS

Nos dedicaremos ahora al estudio de los números piramidales

cuadrangulares, o “pirámides cuadradas”, que se forman al apilar

números cuadrados consecutivos.´Puedes hacerte una idea con las

imágenes y definiciones contenidas en

https://es.wikipedia.org/wiki/N%C3%BAmero_piramidal_cuadrado)

Los primeros números piramidales cuadrados son:

0, 1, 5, 14, 30, 55, 91, 140, 204, 285, 385, 506, 650, 819, 1015, 1240,

1496, 1785, 2109, 2470, 2870, 3311, 3795, 4324, 4900, 5525, 6201,

6930, 7714, 8555, 9455,…

(http://oeis.org/A000330)

Según su definición, cada piramidal cuadrado será equivalente a la

suma 1+4+9+16+…, pero se conoce, y es muy popular, la fórmula de

la suma de los n primeros cuadrados, que es igual a n*(n+1)*(2*n+1)/6,

luego, si llamamos PCUAD(n) al enésimo piramidal cuadrado

tendremos:

𝑃𝐶𝑈𝐴𝐷(𝑛) =𝑛(𝑛 + 1)(2𝑛 + 1)

6

Si descomponemos 2n+1 en (n+2)+(n-1) resulta

𝑃𝐶𝑈𝐴𝐷(𝑛) = (𝑛 + 2

3) + (

𝑛 + 13

)

Page 162: Números y formas - Aprender y divertirse con hoja de …hojamat.es/publicaciones/numform.pdf · Este es uno de los capítulos de las Matemáticas en los que se aprecia ... también

162

Al igual que un cuadrado se descompone en dos triángulos

consecutivos, aquí serían dos tetraedros: Todo número piramidal

cuadrado es suma de dos piramidales triangulares consecutivos.

Lo podemos comprobar con nuestra calculadora especializada

calcupol, que puedes descargar desde

http://www.hojamat.es/sindecimales/aritmetica/herramientas/herrarit.ht

m#figurados

Elegimos n=9 y con la calculadora vamos obteniendo:

Secuencia de teclas: 4 PIR 9 =, y nos da el piramidal cuadrado 285

Con la secuencia 3 PIR 9 = obtenemos la pirámide triangular 165, y

con 3 PIR 8 =, la anterior, 120, y se verifica que 165+120=285.

Si recordamos que los piramidales triangulares (tetraedros) son suma

de triangulares y los piramidales cuadrados suma de cuadrados, en

hoja de cálculo queda

Aquí vemos que los números triangulares engendran las pirámides

triangulares, que si sumamos por filas, cada dos triangulares forman un

cuadrado, y su suma el piramidal pedido. Al final, dos pirámides

Page 163: Números y formas - Aprender y divertirse con hoja de …hojamat.es/publicaciones/numform.pdf · Este es uno de los capítulos de las Matemáticas en los que se aprecia ... también

163

triangulares consecutivas suman la cuadrangular correspondiente.

Merece la pena estudiar con detalle el esquema de cálculo.

Otra forma de generar piramidales cuadrados a partir de los

triangulares es el cálculo mediante esta fórmula:

𝑃𝐶𝑈𝐴𝐷(𝑛) =1

4(

2𝑛 + 23

)

Basta desarrollar:

((2n+2)(2n+1)2n)/(6*4)=n(n+1)(2n+1)/6, que es la fórmula usual, según

vimos más arriba.

En este tipo de propiedades se basa esta otra generación de

piramidales cuadrados (ver http://oeis.org/A000330). Formamos un

triángulo como el del esquema, que está construido sobre n=5. Se

entiende fácilmente:

1

1 2

1 2 3

1 2 3 4

1 2 3 4 5

1 2 3 4

1 2 3

1 2

1

Hemos situado un 1 en una base de 2*5-1 elementos, un 2, en la

siguiente de 2*5-3, después un 3 2*5-5 veces, y así hasta llegar a un 5.

Lo que propone este esquema es que

𝑃𝐶𝑈𝐴𝐷(𝑛) = ∑ 𝑘(2𝑛 − 2𝑘 + 1)

𝑛

1

Se puede demostrar por inducción recorriendo el esquema:

PCUAD(1)=1, PCUAD(2)=1+1+1+2=5=1+4,

PCUAD(3)=1+1+1+1+1+2+2+2+3=14=1+4+9

Page 164: Números y formas - Aprender y divertirse con hoja de …hojamat.es/publicaciones/numform.pdf · Este es uno de los capítulos de las Matemáticas en los que se aprecia ... también

164

Se generan así 1, 5 y 14, los primeros piramidales. Para demostarlo

para n+1 suponiéndolo correcto para n, bastará considerar, por

definición, que los sumandos nuevos al ampliar el esquema de n a n+1

son:

1+(1+2)+(2+3)+(3+4)+(4+5)+…(n+n+1)=1+3+5+7+…2n+1=(n+1)^2,

luego si se incrementa en un cuadrado, dará lugar al siguiente

piramidal cuadrado, lo que completa la demostración.

Se puede dar otra interpretación a esta fórmula, y es que representa la

suma de la función MÍNIMO a todos los pares formados por el conjunto

1..n consigo mismo. Para entenderlo lo construimos para n=5

En el esquema se han situado los mínimos de cada par fila-columna en

la celda correspondiente, y hemos reproducido el esquema de más

arriba: el 1 se repite 9 veces, el 2, 7 veces, el 3, 5…y así hasta el

último 5 que se repite una vez. Coincide, por tanto, con la fórmula

explicada. Por eso, en la última fila aparecen los piramidales

cuadrados (Enrique Pérez Herrero, Jan 15 2013)

Esta generación la usaremos más adelante.

CURIOSIDADES

Cuadrados que se ven en una cuadrícula.

Un acertijo muy popular consiste en saber ver todos los cuadrados

contenidos en una cuadrícula también cuadrada.

Page 165: Números y formas - Aprender y divertirse con hoja de …hojamat.es/publicaciones/numform.pdf · Este es uno de los capítulos de las Matemáticas en los que se aprecia ... también

165

Para n=1 se ve 1 cuadrado, para n=2, 5 cuadrados, para n=3, 14,

luego son números piramidales cuadrangulares. Lo vemos: En la

cuadrícula de la imagen hay 36 cuadrados de una unidad, y de 2

unidades ha de haber 25, ya que se puede duplicar o copiar de cinco

formas distintas por filas o por columnas, y así, el de tres unidades se

copia 3*3=9 veces, hasta que llegamos al total del que sólo existe una

copia, luego S=36+25+16+9+4+1=91, que es la sexta pirámide

cuadrangular.

Permutaciones con el tercer elemento mayor o igual

Los números piramidales cuadrados coinciden también con las

variaciones de n elementos tomados de 3 en 3 con repetición,

considerando tan solo aquellas en las que el tercer elemento es mayor

o igual que los anteriores. En efecto, basta descomponer las

variaciones binarias en conjuntos según el máximo valor de sus

elementos. Lo aclaramos con un ejemplo. Imagina las variaciones

binarias de 6 elementos, 36 en total:

Page 166: Números y formas - Aprender y divertirse con hoja de …hojamat.es/publicaciones/numform.pdf · Este es uno de los capítulos de las Matemáticas en los que se aprecia ... también

166

A cada una le hemos adosado el máximo valor que presenta. Tenemos

9 con el máximo 5, a las que solo podemos adosar como tercer

elemento otro 5. Después vemos 7 con máximo 4, que admiten una

ampliación con 2 elementos, el 4 y el 5. Para el 3 como máximo se

presentan 5 arreglos y se pueden ampliar con 3, 4 y 5. Resumiendo,

los posibles arreglos de tres elementos en los que el tercero sea mayor

o igual que los otros vendrán dados por este cálculo:

9*1+7*2+5*3+3*4+1*5=9+14+15+12+5=55=1+4+9+16+25. Nos ha

resultado PCUAD(5)

No hay nada sorprendente en esto, ya que hemos reproducido la

fórmula que demostramos más arriba:

𝑃𝐶𝑈𝐴𝐷(𝑛) = ∑ 𝑘(2𝑛 − 2𝑘 + 1)

𝑛

1

Con nuestra hoja Cartesius puedes comprobarlo

http://www.hojamat.es/sindecimales/combinatoria/herramientas/herrco

mb.htm#cartesius

Basta escribir las condiciones (en el ejemplo para n=5)

1 1 1

1 2 2

1 3 3

1 4 4

1 5 5

2 1 2

2 2 2

2 3 3

2 4 4

2 5 5

3 1 3

3 2 3

3 3 3

3 4 4

3 5 5

4 1 4

4 2 4

4 3 4

4 4 4

4 5 5

5 1 5

5 2 5

5 3 5

5 4 5

5 5 5

Page 167: Números y formas - Aprender y divertirse con hoja de …hojamat.es/publicaciones/numform.pdf · Este es uno de los capítulos de las Matemáticas en los que se aprecia ... también

167

Obtendrás 55 para n=5. Puedes ir cambiando xt=1..5 por otro intervalo

y obtendrás el número piramidal correspondiente.

Suma de productos de cuadrados

Terminamos con otra propiedad curiosa, y es que el piramidal

PCUAD(n) es la raíz cuadrada de la suma de todos los cuadrados

(i*j)^2 en los que tanto i como j recorren los valores 1,2..n. Lo puedes

estudiar en este esquema:

En él hemos construido todos los cuadrados y hemos sumado los

valores inferiores a cada n en la fila de abajo. Se destaca en color el

caso n=5 para que lo sigas mejor. Al extraer la raíz cuadrada en la

parte inferior resultan los primeros piramidales cuadrados.

No es difícil razonarlo. Recorre la primera columna de las sumas, por

ejemplo la que tiene fondo de color. Sus sumandos 1+4+9+16+25

forman el piramidal cuadrado 55. La segunda equivale a la primera

multiplicada por 4, luego su suma será 55*4, y las siguientes 55*9,

55*16 y 55*25. Si sacamos factor común en las sumas resultará

55(1+4+9+16+25)=55*55, luego su raíz cuadrada será el piramidal

pedido.

De igual forma se puede razonar que si tomamos productos triples

(i*j*k)^3, su raíz cúbica será también igual al piramidal cuadrado

correspondiente.

Page 168: Números y formas - Aprender y divertirse con hoja de …hojamat.es/publicaciones/numform.pdf · Este es uno de los capítulos de las Matemáticas en los que se aprecia ... también

168

Para terminar, una curiosidad: el único piramidal cuadrado que a su

vez es un cuadrado es el 4900.

PENTÁGONOS

Hoy desarrollaremos los piramidales pentagonales, que se forman

añadiendo a la unidad (el vértice de la pirámide) distintos números

pentagonales como si fueran cortes poligonales de la pirámide. Para

nosotros es preferible ver los piramidales como suma de pentagonales

sucesivos. Así, si estos forman la sucesión 1, 5, 12, 22, 35, 51, 70, 92,

117, 145,… http://oeis.org/A000326, los piramidales correspondientes

coincidirán con sus sumas parciales: 1, 6, 18, 40, 75, 126, 196,…

Los primeros piramidales pentagonales son:

1, 6, 18, 40, 75, 126, 196, 288, 405, 550, 726, 936, 1183, 1470, 1800,

2176, 2601, 3078, 3610, 4200, 4851, 5566, 6348, 7200, 8125, 9126,

10206, 11368, 12615, 13950, 15376, 16896, 18513, 20230, 22050,

23976, 26011, 28158, 30420, 32800, 35301, 37926, 40678,…

http://oeis.org/A002411

Puedes usar nuestra calculadora calcupol para recorrerlos.

Descárgala, si lo deseas, desde

http://www.hojamat.es/sindecimales/aritmetica/herramientas/herrarit.ht

m#figurados

Para recorrer la sucesión basta fijar el tipo en Piramidal y el orden en

5. Después se escribe un 1 en pantalla y cada pulsación de la tecla

PROX nos devolverá un térmiuno nuevo de la sucesión.

En la imagen hemos llegado a 405:

Page 169: Números y formas - Aprender y divertirse con hoja de …hojamat.es/publicaciones/numform.pdf · Este es uno de los capítulos de las Matemáticas en los que se aprecia ... también

169

Con la tecla ANT puedes retroceder, y entre ambas recorrer el rango

de términos que desees.

Fórmula

Los números piramidales pentagonales (PPENT) siguen una expresión

polinómica muy sencilla:

𝑃𝑃𝐸𝑁𝑇(𝑛) =𝑛2(𝑛 + 1)

2=

𝑛3 + 𝑛2

2

Esta fórmula se obtiene particularizando para 5 la general de los

piramidales:

𝑃𝐼𝑅(𝑛, 𝑘) =3𝑛2 + 𝑛3(𝑘 − 2) − 𝑛(𝑘 − 5)

6

𝑃𝑃𝐸𝑁𝑇(𝑛) =3𝑛2 + 𝑛3(5 − 2) − 𝑛(5 − 5)

6=

3𝑛3 + 3𝑛2

6=

𝑛3 + 𝑛2

2

Por tanto, podemos afirmar que los piramidales pentagonales son los

promedios entre el cuadrado y el cubo de un número. Por ejemplo:

405 es el piramidal pentagonal número 9, y se cumple que

405=(81+729)/2=810/2=405

Esto nos permite crear una tabla a partir de la sucesión de números

naturales:

Page 170: Números y formas - Aprender y divertirse con hoja de …hojamat.es/publicaciones/numform.pdf · Este es uno de los capítulos de las Matemáticas en los que se aprecia ... también

170

También la fórmula obtenida descubre que una pirámide pentagonal de

lado k equivale a k veces el triangular del mismo lado. Por ejemplo, la

pirámide de lado 6, 126, es seis veces mayor que 21, que es el

triangular número 6.

Otra interpretación

El número piramidal pentagonal PPENT(n) equivale a la suma de los

n+1 múltiplos menores de n. En efecto, esos múltiplos serán n*0, n*1,

n*2,…n*n, y formarán progresión aritmética de diferencia n, luego su

suma será (n*0+n*n)*(n+1)/2 = (n^3+n^2)/2, que es la expresión

descubierta más arriba. Aquí tienes el esquema para PPENT(7)=196

Si extraemos factor común el 7, nos queda la suma 0+1+2+3+…que es

un número triangular, tal como vimos unos párrafos más arriba. Esto

nos descubre otra interpretación, y es que un número piramidal

pentagonal equivale a un “prisma” triangular de la misma altura.

Page 171: Números y formas - Aprender y divertirse con hoja de …hojamat.es/publicaciones/numform.pdf · Este es uno de los capítulos de las Matemáticas en los que se aprecia ... también

171

Expresado como sumatorio:

𝑃𝑃𝐸𝑁𝑇(𝑛) = 𝑛 ∑ 𝑖

𝑛

𝑖=0

Recurrencia

Estos números presentan tantas formas de generación que el

procedimiento recurrente no es muy necesario. No obstante, existen

varias fórmulas de recurrencia. La más simple es

ppent(n) = 3*ppent(n-1) - 3*ppent(n-2) + ppent(n-3) + 3.

Es una recurrencia de tercer orden no homogénea. Se puede

demostrar con un simple desarrollo algebraico, si recordamos que

ppent(n)=(n^3+n^2)/2. Desarrollamos:

Ppent(n)=3((n-1)^3+(n-1)^2)/2-3((n-2)^3+(n-2)^2)/2+((n-3)^3+(n-

3)^2)/2+3

Si nos da pereza simplificar, podemos acudir a Wiris, wxMaxima u otro

similar. Usamos el segundo y obtenemos:

Como el resultado es (n^3+n^2)/2, hemos demostrado la recurrencia.

No es muy útil.

Cuestiones combinatorias

Es muy ilustrativo el estudio de algunas relaciones entre temas propios

de números enteros con otros combinatorios. Muchas sucesiones

poseen sentidos bastante simples si las estudiamos desde ese punto

de vista. Los piramidales pentagonales también las admiten.

Page 172: Números y formas - Aprender y divertirse con hoja de …hojamat.es/publicaciones/numform.pdf · Este es uno de los capítulos de las Matemáticas en los que se aprecia ... también

172

Bolas y cajas

Imagina que disponemos de n bolas que deseamos guardar en n cajas,

pero con la caprichosa condición de que solo usaremos 2 de ellas,

dejando vacías las demás. En ese caso, el número de formas de

guardar esas bolas es PPENT(n-1)

Así que decidimos usar solamente dos cajas. En la imagen nos hemos

decidido por la 3 y la 6:

Nos comprometemos a guardar las n bolas en esas dos cajas. Es

evidente que tenemos n-1 posibilidades, si no deseamos que una

quede vacía: 1+(n-1), 2+(n-2), 3+(n-3), …(n-1)+1

Por otra parte, la elección de las cajas, que en nuestro ejemplo eran la

3 y la 6, se puede efectuar de C(n,2) formas, combinaciones de n cajas

tomadas de 2 en 2, es decir n(n-1)/2

Multiplicamos las formas de elegir dos cajas por las de rellenarlas, y

tenemos:

P= n(n-1)/2*(n-1)=(n^3-2n^2+n)/2

Esta expresión coincide con PPENT(n-1)=((n-1)^3+(n-1)^2)/2=(n^3-

2n^2+n)/2

Cadenas de caracteres

En esta cuestión imaginamos que formamos todas las palabras

posibles de tres caracteres en un alfabeto de n caracteres, y

consideramos iguales entre sí aquellas palabras que contienen los

mismos caracteres, pero invertidos.

Por ejemplo, supongamos las cinco letras A, B, C, D, E agrupadas en

palabras de tres AAA, ABC, CBD,… y consideramos idénticas las

inversas entre sí, como CBD, que la considaremos equivalente a DBC.

Page 173: Números y formas - Aprender y divertirse con hoja de …hojamat.es/publicaciones/numform.pdf · Este es uno de los capítulos de las Matemáticas en los que se aprecia ... también

173

Con estas condiciones, el número de palabras con un alfabeto de n

caracteres será PPENT(n).

Tampoco es difícil de razonar: El número total de palabras será n*n*n,

pero estas se dividen en dos grupos:

Simétricas (capicúas o palindrómicas) que se contarán una vez. Su

número es n*n=n^2 (el tercer elemento está obligado)

No simétricas, cuyo número se ha dividir entre 2 para eliminar las

palabras simétricas entre sí. Como su número es (n*n*n-n*n), al dividir

entre 2 quedará (n*n*n-n*n)/2=n^2(n-1)/2

Sumamos ambos casos:

P=n^2+n^2(n-1)/2 = n^2(2+n-1)/2 = n^2(n+1)/2 =

(n^3+n^2)/2=PPENT(n)

Así hemos demostrado la equivalencia. Lo vemos con el caso n=5:

Con un alfabeto de 5 caracteres se forman 5*5*5=125 palabras, de las

que 5*5=25 son capicúas, y 125-25=100, no capicúas. Estas segundas

hay que contarlas una vez, luego dividimos entre 2, quedando 50.

Sumamos ambos casos y obtenemos 50+25=75, que es el quinto

número piramidal pentagonal.

HEXÁGONOS Y OCTÓGONOS

Quienes sigáis esta serie sobre números piramidales adivinaréis que

los de tipo hexagonal son suma de los primeros números poligonales

hexagonales, 1, 6, 15, 28, 45, 66, 91, 120, 153, 190, 231, 276, 325,…

Ve acumulando sumas y obtendrás los piramidales hexagonales

(PHEX):

1, 7, 22, 50, 95, 161, 252, 372, 525, 715, 946, 1222, 1547, 1925, 2360,

2856, 3417, 4047, 4750, 5530, 6391, 7337, 8372,…

http://oeis.org/A002412

Page 174: Números y formas - Aprender y divertirse con hoja de …hojamat.es/publicaciones/numform.pdf · Este es uno de los capítulos de las Matemáticas en los que se aprecia ... también

174

Con nuestra calculadora Calcupol puedes también recorrerlos.

Descárgala desde

http://www.hojamat.es/sindecimales/aritmetica/herramientas/herrarit.ht

m#figurados

Fija el tipo en Piramidal de orden 6, escribe un 1 en pantalla y pulsa

reiteradamente la tecla PROX. Así los obtendrás uno a uno. En la

imagen hemos llegado hasta el 525.

Como todos los números piramidales, estos poseen una expresión

polinómica que los genera. En este caso es

PHEX(n) = n(n + 1)(4n - 1)/6

Esta fórmula se obtiene particularizando para 6 la general de los

piramidales:

𝑃𝐼𝑅(𝑛, 𝑘) =3𝑛2 + 𝑛3(𝑘 − 2) − 𝑛(𝑘 − 5)

6

𝑃𝐻𝐸𝑋(𝑛) =3𝑛2 + 4𝑛3 − 𝑛

6=

𝑛(𝑛 + 1)(4𝑛 − 1)

6

Lo podemos comprobar, por ejemplo, para n=6, y nos queda:

PHEX(6)=6*7*23/6=161, como era de esperar.

Recurrencias

Estos números presentan varias formas de generación por recurrencia.

La más práctica es la siguiente:

a(n) = 3*a(n-1) - 3*a(n-2) + a(n-3) + 4.

Page 175: Números y formas - Aprender y divertirse con hoja de …hojamat.es/publicaciones/numform.pdf · Este es uno de los capítulos de las Matemáticas en los que se aprecia ... también

175

Podemos organizarlo según esta tabla, que iniciamos con 1, 7, 22. En

cada fila añadimos un elemento nuevo calculado mediante la

recurrencia:

Como suma

De diferencias de cuadrados

La definición de los números figurados mediante acumulación de otros

más simples hace que sean frecuentes las generaciones mediante

sumas de elementos. En el caso de los piramidales hexagonales basta

acumular diferencias de cuadrados entre un número natural N y todos

los menores que él. Lo puedes ver en esta tabla:

Por ejemplo, 50 es igual a 4^2-0^2+4^2-1^2+4^2-2^2+4^2-3^2

Se puede desarrollar algebraicamente, si recuerdas la fórmula para la

suma de los primeros cuadrados:

S = n2-02+n2-12+n2-22+n2-32+…n2-(n-1)2 = n3-n(n-1)(2n-1)/6 = n(n+1)(4n-

1)/6

Hemos desembocado en la fórmula de los piramidales hexagonales, lo

que demuestra la propiedad.

Page 176: Números y formas - Aprender y divertirse con hoja de …hojamat.es/publicaciones/numform.pdf · Este es uno de los capítulos de las Matemáticas en los que se aprecia ... también

176

De números naturales impares

Todos los números piramidales hexagonales son suma de triangulares

impares. Así, el tercero a(3) = t(1)+t(3)+t(5) = 1+6+15 = 22

Algebraicamente:

1*2/2+3*4/2+5*6/2+…(2n-1)n, expresada como sumatorio queda:

∑(2𝑛 − 1)𝑛 = 2 ∑ 𝑛2 − ∑ 𝑛

𝑛

1

= 2𝑛(𝑛 + 1)(2𝑛 + 1)

6

𝑛

1

𝑛

1

−𝑛(𝑛 + 1)

2

Si desarrollas llegarás a la fórmula del piramidal hexagonal:

PHEX(n)=n(n+1)(4n-1)/6

Relación con números combinatorios

El denominador 6 de la fórmula de los piramidales hexagonales sugiere

su relación con números combinatorios de índice inferior igual a 3, y,

en efecto, existen esas relaciones:

𝑃𝐻𝐸𝑋(𝑛) = (𝑛 + 2

3) + 3 (

𝑛 + 13

)

Lo desarrollamos:

C(n+2,3)+3C(n+1,3)=(n+2)(n+1)n/6+3(n+1)n(n-1)/6=n(n+1)/6*(n+2+3n-

3)=n(n+1)(4n-1)/6

Es un simple identidad algebraica.

Otra relación:

𝑃𝐻𝐸𝑋(𝑛) = (4𝑛 − 1)(

𝑛 + 12

)

3

Es también una identidad algebraica sencilla.

Page 177: Números y formas - Aprender y divertirse con hoja de …hojamat.es/publicaciones/numform.pdf · Este es uno de los capítulos de las Matemáticas en los que se aprecia ... también

177

Fin de la serie

No podemos extender en demasía la exposición de números

piramidales. Terminamos con una breve referencia a los octogonales.

Como todos los anteriores, los piramidales octogonales resultan de la

suma de los poligonales del mismo número de lados. Por ejemplo, la

pirámide octogonal de índice 5 resultará de la suma:

1+8+21+40+65=135

Los primeros octogonales son:

1, 9, 30, 70, 135, 231, 364, 540, 765, 1045, 1386, 1794, 2275, 2835,

3480, 4216, 5049, 5985, 7030, 8190, 9471, 10879,…

http://oeis.org/A002414

Su fórmula: a(n)=n(n+1)(2n-1)/2

Por ejemplo, a(7)=7*8*13/2=28*13=364

NÚMEROS PIRAMIDALES DE CUATRO

DIMENSIONES

Números figurados e interpolación polinómica

Los desarrollos sobre números piramidales estudiados hasta ahora

contenían siempre una fórmula polinómica para expresar cada término

de la sucesión. Por ello parece conveniente presentar un método

general para la obtención de esas fórmulas. Disponemos para ello de

la teoría de la interpolación polinómica, en la que elegiremos el método

de Newton, y una hoja de cálculo que nos facilitará las cosas.

Page 178: Números y formas - Aprender y divertirse con hoja de …hojamat.es/publicaciones/numform.pdf · Este es uno de los capítulos de las Matemáticas en los que se aprecia ... también

178

Teoría

Los conceptos y métodos de la interpolación polinómica los puedes

encontrar en cualquier texto del primer ciclo de estudios universitarios.

Todos se basan en el cálculo con diferencias sucesivas. En nuestro

caso nos limitaremos a la suma de funciones enteras definidas

sobre los primeros números naturales, lo que simplifica mucho los

cálculos.

Por ejemplo, imaginemos que deseamos sumar los primeros números

oblongos: 1*2, 2*3, 3*4, 4*5, 5*6,…o bien, 2, 6, 12, 20, 30,…Queremos

obtener una expresión para 2+6+12+20+30+…

En las técnicas de interpolación que usaremos, la primera operación es

la obtención de las diferencias sucesivas, es decir, diferencias entre los

elementos, diferencias entre las diferencias, y así sucesivamente hasta

(en el caso polinómico) que sean todas iguales. Lo intentamos con el

ejemplo. En primer lugar encontramos las sumas parciales de

oblongos: 2, 2+6=8, 2+6+12=20,…que serían 2, 8, 20, 40, 70, 112,

168, 240,…

Lo puedes organizar con una hoja de cálculo:

La siguiente imagen, tomada de nuestra hoja newton.xls, puedes

entender muy bien el concepto de diferencias sucesivas. En primer

lugar hemos escrito nuestra sucesión: 2, 8, 20, 40, 70, 112,

168,…Después hemos ido restando cada elemento con el siguiente: 6,

12, 20, 30, 42, 56,…(resultan ser los oblongos). A continuación hemos

restado esas diferencias: 6, 8, 10, 12, 14,…entre sí, y al final en otra

resta, hemos llegado a 2, 2, 2,…Esa es la señal de que esos números

siguen una expresión polinómica, el que las diferencias se hagan

iguales y las siguientes nulas.

Page 179: Números y formas - Aprender y divertirse con hoja de …hojamat.es/publicaciones/numform.pdf · Este es uno de los capítulos de las Matemáticas en los que se aprecia ... también

179

Como aquí las diferencias constantes se alcanzan en tres pasos, la

fórmula que buscamos será un polinomio de tercer grado. Lo

repasamos en teoría:

Fórmula de interpolación de Newton

Cuando en un proceso se llega a diferencias constantes, sabemos que

es posible expresar la sucesión dada mediante un polinomio

dependiente del número de orden. La fórmula hallada por Newton es

algo compleja en el caso general, en el que hay que usar diferencias

divididas, pero se simplifica bastante en el caso de un polinomio

aplicado al conjunto 1, 2, 3, 4,…Sería esta:

𝑃(𝑥) = 𝑎0 + 𝑑1(𝑥 − 1) +𝑑2(𝑥−1)(𝑥−2)

2!+

𝑑3(𝑥−1)(𝑥−2)(𝑥−3)

3!+…

En ella a0 es el primer término, d1 la primera diferencia, d2 la primera de

las segundas diferencias, y así sucesivamente. En nuestro caso sería:

P(x)=2+6(x-1)+6(x-1)(x-2)/2!+2(x-1)(x-2)(x-3)/3!

Reduciendo a común denominador y simplificando:

𝑃(𝑥) =2𝑥3 + 6𝑥2 + 4𝑥

6=

𝑥3 + 3𝑥2 + 2𝑥

3

Lo hemos comprobado con la hoja de cálculo. Observa la coincidencia

de los valores en rojo:

Page 180: Números y formas - Aprender y divertirse con hoja de …hojamat.es/publicaciones/numform.pdf · Este es uno de los capítulos de las Matemáticas en los que se aprecia ... también

180

Como ves, lo que es más complicado es la simplificación, pero el

método es simple: encontrar diferencias sucesivas hasta que se

estabilicen, y aplicar la fórmula de interpolación simplificada para los

puntos de apoyo 1, 2, 3, 4,…

Todo esto puedes reproducirlo con nuestra hoja de cálculo newton,

que puedes descargar desde

http://www.hojamat.es/sindecimales/aritmetica/herramientas/herrarit.ht

m#newton

Recuerda que sólo te vale para sumas definidas sobre los primeros

naturales. Basta escribir los valores de la sucesión en la segunda fila

y leer los coeficientes (en forma de fracción) más abajo:

Es fácil de interpretar (de derecha a izquierda): 2/1 es el coeficiente de

1, 6/1 el de (x-1), 6/2 para (x-1)(x-2) y, finalmente, 2/6 para (x-1)(x-2)(x-

3). Después vendría la simplificación, pero si quieres ahorrártela, la

hoja dispone del cálculo para un valor concreto. Por ejemplo, ¿cuánto

suman los diez primeros oblongos? Para ello dispones de las celdas

adecuadas

Page 181: Números y formas - Aprender y divertirse con hoja de …hojamat.es/publicaciones/numform.pdf · Este es uno de los capítulos de las Matemáticas en los que se aprecia ... también

181

La respuesta es 440. Hay que advertir que puede producir pequeños

errores para índices grandes, por lo que se aconseja comprobar.

Si deseas evitarte la simplificación puedes acudir a un CAS. Nosotros

hemos usado wxMaxima para obtener el resultado previsto:

Otro paso sería el intentar, si es posible, buscar una interpretación al

resultado obtenido. En nuestro caso es fácil ver que, para x mayor o

igual a 3, se reduce a

𝑃(𝑥) = 2 (𝑛3

)

Este proceso se puede repetir para números poligonales, piramidales o

poligonales centrados (y otros similares que nos inventemos), porque

todos se basan en sus definiciones en la acumulación de sumas, lo

que garantiza que la fórmula buscada es de tipo poligonal.

Otro ejemplo

Sabemos que los números piramidales triangulares se construyen

sumando los triangulares, y estos, a su vez, acumulando los naturales.

Con un sencillo esquema los identificamos:

Page 182: Números y formas - Aprender y divertirse con hoja de …hojamat.es/publicaciones/numform.pdf · Este es uno de los capítulos de las Matemáticas en los que se aprecia ... también

182

Luego los primeros piramidales son 1, 4, 10, 20,….Los volcamos en la

hoja newton para descubrir sus diferencias y los coeficientes de

interpolación:

Las diferencias son 1, 3, 3 y 1, y con ellas se construyen los

coeficientes 1/1, 3/1, 3/2 y 1/6. Rellenamos la fórmula de interpolación

y queda:

P(x)=1+3(x-1)+3/2(x-1)(x-2)+1/6(x-1)(x-2)(x-3)

Simplificamos con wxMaxima:

Coincide con la que se obtuvo para los números tetragonales (o

piramidales triangulares)

http://hojaynumeros.blogspot.com.es/2017/04/numeros-piramidales-2-

tetraedros.html

𝑇𝐸𝑇(𝑛) = 𝑃𝐼𝑅(𝑛, 3) =3𝑛2 + 𝑛3 + 2𝑛

6=

𝑛(𝑛 + 1)(𝑛 + 2)

6

Con lo aprendid estamos preparados para saltar a la cuarta o quinta

dimensión, y sumar números piramidales consecutivos.

Page 183: Números y formas - Aprender y divertirse con hoja de …hojamat.es/publicaciones/numform.pdf · Este es uno de los capítulos de las Matemáticas en los que se aprecia ... también

183

NÚMEROS PIRAMIDALES DE CUATRO

DIMENSIONES

Si has seguido los desarrollos anteriores sobre números piramidales

(escribe “piramidales” en la casilla de búsqueda) sabrás que estos

números se generan mediante sumas parciales en una sucesión de

dimensión inferior.

Así, los tetraedros se generan sumando números triangulares. Estos, a

su vez, mediante números lineales. Las pirámides cuadradas se

obtienen sumando cuadrados, y los de tipo poligonal sumando los de

dos dimensiones del mismo número de lados.

Si sumamos números piramidales, obtendremos piramidales de cuatro

dimensiones. Por ejemplo, los piramidales cuadrados son 1, 5, 14, 30,

55, 91, 140, 204

(ver http://hojaynumeros.blogspot.com.es/2017/05/numeros-

piramidales-3-cuadrados.html)

Formamos con ellos sumas parciales: 1, 1+5, 1+5+14, 1+5+14+30,… y

obtenemos:

1, 6, 20, 50, 105, 196, 336, 540,…

Estos será, pues, los piramidales cuadrados de cuatro dimensiones.

http://oeis.org/A002415

A partir de ahora iremos recorriendo los piramidales de cuatro

dimensiones según su número de lados. En todos ellos comenzaremos

con sumas parciales en piramidales de tres dimensiones, obtendremos

su fórmula polinomial y terminaremos con curiosidades y

equivalencias. Comenzamos con triangulares:

Page 184: Números y formas - Aprender y divertirse con hoja de …hojamat.es/publicaciones/numform.pdf · Este es uno de los capítulos de las Matemáticas en los que se aprecia ... también

184

Tetraedros de cuatro dimensiones

Estudiamos anteriormentelos tetraedros de tres dimensiones.

Vimos que la sucesión de los mismos comienza con

1, 4, 10, 20, 35, 56, 84, 120, 165, 220,…

Si procedemos a formar sumas parciales, obtendremos los tetraedros

de cuatro dimensiones:

1, 1+4, 1+4+10, 1+4+10+20, 1+4+10+20+35,…. Es decir, 1, 5, 15, 35,

70, 126, 210,…

Los primeros términos de la sucesión de números de este tipo son:

1, 5, 15, 35, 70, 126, 210, 330, 495, 715, 1001, 1365, 1820, 2380,

3060, 3876, 4845, 5985, 7315, 8855,…, y están recogidos en

http://oeis.org/A000332 con otra definición, coincidente, como veremos,

salvo algunos ceros.

Los nombraremos como PIR3_4(n), donde 3 es el número de lados, 4

la dimensión y n el número de orden. Así 210=PIR3_4(7).

Obtención de la fórmula polinomial

Ya estudiamos este procedimiento en una anteriormente

http://hojaynumeros.blogspot.com.es/2017/09/numeros-figurados-e-

interpolacion.html

Consiste en usar la fórmula de interpolación de Newton aplicada a los

primeros números. Remitimos a la entrada enlazada para seguir el

procedimiento. En primer lugar escribimos los primeros términos 1, 5,

15, 35, 70,… y obtenemos sus diferencias sucesivas:

Page 185: Números y formas - Aprender y divertirse con hoja de …hojamat.es/publicaciones/numform.pdf · Este es uno de los capítulos de las Matemáticas en los que se aprecia ... también

185

Observamos que son nulas las diferencias de orden 5, luego el

polinomio que buscamos es de cuarto grado. Sus coeficientes los

leemos más abajo:

Por tanto, el polinomio buscado será

1+4(x-1)+3(x-1)(x-2)+2(x-1)(x-2)(x-3)/3+(x-1)(x-2)(x-3)(x-4)/24

Podemos acudir a wxMaxima para simplificar:

O bien con la web https://www.wolframalpha.com, obtenemos el mismo

polinomio:

Con el comando factor de wxMaxima factorizamos:

Esta es la fórmula más práctica para obtener los tetraedros de cuatro

dimensiones, que, es fácil verlo, coincide con:

Page 186: Números y formas - Aprender y divertirse con hoja de …hojamat.es/publicaciones/numform.pdf · Este es uno de los capítulos de las Matemáticas en los que se aprecia ... también

186

𝑃𝐼𝑅3_4(𝑛) = (𝑛 + 3

4)

En toda esta serie nos aparecen números combinatorios, y es porque

se pueden localizar los piramidales en el triángulo de Pascal

En la imagen están destacados los triangulares, los tetraedros de tres

dimensiones y los de cuatro, que son suma de los anteriores.

Interpretaciones

Los tetraedros de cuatro dimensiones coinciden con las intersecciones

de las diagonales de un polígono convexo, siempre que no concurran

más de dos diagonales en el mismo punto. Por eso debemos elegir

polígonos no regulares, como el de la figura

En él está representado el número 35, como las intersecciones (en

color verde) en un heptágono. El 35 es el cuarto tetraedro de cuatro

dimensiones, y le corresponde el polígono de tres lados más. Esto es

por el n+3 que figura en la fórmula general.

He encontrado una explicación muy sencilla debida a Ignacio Larrosa

@ilarrosac: Si el polígono es irregular, cada cuatro vértices formarán

un cuadrilátero convexo distinto, y sus diagonales producen un único

Page 187: Números y formas - Aprender y divertirse con hoja de …hojamat.es/publicaciones/numform.pdf · Este es uno de los capítulos de las Matemáticas en los que se aprecia ... también

187

punto de intersección. Por tanto, el número de ellos coincidirá con el de

combinaciones de n+3 lados tomados de 4 en 4. Muy elegante.

También se debe a Ignacio Larrosa la siguiente interpretación:

El número piramidal triangular de cuatro dimensiones de orden n

coincide con todos los triángulos equiláteros que se pueden dibujar

uniendo tres puntos de una matriz que rellena otro triángulo equilátero

de lado n+1.

En la figura puedes ver uno de esos triángulos.

La idea de Ignacio Larrosa consiste en que cada triángulo tiene sus

vértices en los lados de otro mayor que sigue la orientación de la

matriz triangular, y que basta contar estos últimos y también todos los

triángulos de cualquier orientación que se inscriben en ellos.

En la imagen hemos destacado en azul el equilátero orientado que

contiene al elegido en primer lugar. Nos dedicamos a contar:

Número de triángulos orientados de lado k

Es un problema muy estudiado. En la imagen, el número de triángulos

similares al dibujado es 1+2=3. Si tuviera una celda menos de lado su

número sería 1+2+3=6, y así hasta los de lado 1, cuyo número sería

Page 188: Números y formas - Aprender y divertirse con hoja de …hojamat.es/publicaciones/numform.pdf · Este es uno de los capítulos de las Matemáticas en los que se aprecia ... también

188

1+2+3+4+5+6=21, todos números triangulares. En general, el número

de triángulos orientados de lado k en una matriz de lado n sería T(n-

k+1), siendo T el triangular de ese orden.

Número de triángulos contenidos en un orientado

Basta deslizar el primer vértice (por ejemplo el que cae a la izquierda)

a lo largo de su lado, y los demás se situarán en un punto fijado sin

ambigüedad. En el ejemplo se podrían inscribir cuatro.

Es fácil ver que, en general, se pueden inscribir k-1 triángulos.

Con estos dos datos se pueden contar todos los equiláteros posibles

mediante una suma de productos. Para n=6, caso del ejemplo sería:

S=1*5+3*4+6*3+10*2+15*1=5+12+18+20+15=70, que es el piramidal

de orden 5.

Para un estudio general puedes consultar

http://people.missouristate.edu/lesreid/sol03_01.html

Recurrencia

Es fácil ver, según la fórmula general, que

PIR3_4(N)=PIR3_4(N-1)*(N+3)/(N-1)

En efecto:

𝑃𝐼𝑅3_4(𝑛) =𝑛(𝑛 + 1)(𝑛 + 2)(𝑛 + 3)

24

Page 189: Números y formas - Aprender y divertirse con hoja de …hojamat.es/publicaciones/numform.pdf · Este es uno de los capítulos de las Matemáticas en los que se aprecia ... también

189

𝑃𝐼𝑅3_4(𝑛 − 1) =(𝑛 − 1)𝑛(𝑛 + 1)(𝑛 + 2)

24

Luego basta multiplicar por n+3 y dividir entre n-1 para pasar de uno a

otro.

Por ejemplo: 70*9/5=14*9=126

PIRÁMIDES CUADRANGULARES EN CUATRO

DIMENSIONES

Pirámides cuadrangulares

De la misma forma, si tomamos la sucesión de números piramidales

cuadrados de tres dimensiones

(ver http://hojaynumeros.blogspot.com.es/2017/05/numeros-

piramidales-3-cuadrados.html), podemos ir obteniendo sus sumas

parciales.

Estos son los números piramidales cuadrados:

1, 5, 14, 30, 55, 91, 140, 204, 285, 385, 506, 650, 819, 1015, 1240,

1496, 1785, 2109, 2470, 2870, 3311, 3795, 4324, 4900, 5525, 6201,

6930, 7714, 8555, 9455,…

Formamos sus sumas parciales:

1, 6, 20, 50, 105, 196, 336, 540, 825, 1210, 1716, 2366, 3185, 4200,

5440, 6936, 8721, 10830, 13300, 16170, 19481, 23276, 27600,…

Se obtienen así: 1=1, 1+5=6, 1+5+14=20, 1+5+14+30=50,…

Esta será la sucesión de números piramidales cuadrados de cuatro

dimensiones. Los nombraremos como PIR4_4(n)

Los tienes publicados en http://oeis.org/A002415

Obtención de la fórmula polinomial

Page 190: Números y formas - Aprender y divertirse con hoja de …hojamat.es/publicaciones/numform.pdf · Este es uno de los capítulos de las Matemáticas en los que se aprecia ... también

190

Ya estudiamos este procedimiento anteriormente en

http://hojaynumeros.blogspot.com.es/2017/09/numeros-figurados-e-

interpolacion.html

Consiste en usar la fórmula de interpolación de Newton aplicada a los

primeros números naturales. Remitimos a la entrada enlazada para

seguir el procedimiento. En primer lugar escribimos los primeros

términos 1, 6, 20, 50, 105, 196, 336,… y obtenemos sus diferencias

sucesivas de forma automática:

Como las quintas diferencias son nulas, el polinomio interpolador será

de cuarto grado. Los coeficientes los tienes en la parte baja en forma

de fracción. Así quedaría:

1+5(x-1)+9/2(x-1)(x-2)+7/6(x-1)(x-2)(x-3)+2/24(x-1)(x-2)(x-3)(x-4)

Lo podemos simplificar con wxMaxima:

O en la web de Wolfram Alpha, obteniendo el mismo resultado:

Page 191: Números y formas - Aprender y divertirse con hoja de …hojamat.es/publicaciones/numform.pdf · Este es uno de los capítulos de las Matemáticas en los que se aprecia ... también

191

Hay que tener en cuenta que esta expresión es válida si se comienza

la sucesión en 1. Podrás encontrar otras distintas cuando el inicio

contenga ceros.

La comprobamos, por ejemplo para n=5 y n=6:

PIR4_4(5)=5*6^2*7/12=105

PIR4_4(6)=6*7^2*8/12=4*49=196

Expresión con números combinatorios:

Todos los números figurados se pueden expresar mediante números

combinatorios de una forma más o menos compleja. En este caso

disponemos de dos expresiones

𝑃𝐼𝑅44(𝑛) = 2 (𝑛 + 3

4) − (

𝑛 + 23

)

(n+3)(n+2)(n+1)n/12-(n+2)(n+1)n/6=(n+2)(n+1)n((n+3)/12-

1/6)=n(n+2)(n+1)^2/12, que coincide con la fórmula obtenida más

arriba. Podemos comprobarlo también con la función COMBINAT de

las hojas de cálculo:

Para n=6 tendríamos =2*COMBINAT(9;4)-COMBINAT(8;3)=196

Coincide con el resultado obtenido anteriormente.

Puedes probar también con esta otra:

𝑃𝐼𝑅44(𝑛) = (𝑛 + 3

4) + (

𝑛 + 24

)

Así, PIR4_4(7)=COMBINAT(10;4)+COMBINAT(9;4)=336, que es su

valor correcto.

No es difícil comprobar la equivalencia de ambas expresiones

combinatorias.

En la siguiente imagen del triángulo de Pascal hemos rodeado de

círculos estos números combinatorios que sirven de sumandos:

Page 192: Números y formas - Aprender y divertirse con hoja de …hojamat.es/publicaciones/numform.pdf · Este es uno de los capítulos de las Matemáticas en los que se aprecia ... también

192

Podemos sumar cada uno con el siguiente y resultarán piramidales

cuadrados de 4 dimensiones:

1+5=6; 5+15=20; 15+35=50; 35+70=105;

Interpretación geométrica

Al igual que ocurría con las pirámides triangulares y los triángulos,

estos números pueden representar el número de cuadrados que se

pueden dibujar en una rejilla cuadrada de n vértices, si sus lados no

son paralelos a los de la rejilla. En la imagen hemos representado

cuatro de ellos.

Podemos razonar de un modo similar al que usamos con triángulos.

En primer lugar contaremos los cuadrados que se pueden dibujar si

sus lados han de ser paralelos a las líneas de la rejilla. Por ejemplo, en

la imagen se pueden dibujar 36 cuadrados de lado 1, 25 de lado 2, 16

de 3, y así hasta el cuadrado total que sería uno solo. Por tanto, el

número de cuadrados de lados paralelos sería 1+4+9+16+25+36=91.

Resulta ser equivalente a un número piramidal cuadrado de índice 6.

En efecto, puedes repasar la definición y fórmulas en

Page 193: Números y formas - Aprender y divertirse con hoja de …hojamat.es/publicaciones/numform.pdf · Este es uno de los capítulos de las Matemáticas en los que se aprecia ... también

193

http://hojaynumeros.blogspot.com.es/2017/05/numeros-piramidales-3-

cuadrados.html

Dentro de cada cuadrado de lado k en posición paralela se pueden

dibujar k-1 cuadrados de los que nos interesan

En el caso del lado seis, podemos acumular los cuadrados según el

número de lados, y obtendríamos:

36*0+25*1+16*2+9*3+4*4+1*5=105=PIR4_4(5)

Con cinco lados obtendríamos un resultado similar:

25*0+16*1+9*2+4*3+1*4=50=PIR4_4(4)

La demostración general supone mucho cálculo algebraico que nos da

pereza abordar.

OTROS NÚMEROS PIRAMIDALES DE

CUATRO DIMENSIONES

Generación directa

Partimos de los piramidales pentagonales de tres dimensiones:

1, 6, 18, 40, 75, 126, 196, 288, 405, 550, 726, 936, 1183, 1470, 1800,

2176, 2601, 3078, 3610, 4200, 4851, 5566, 6348, 7200, 8125,…

http://hojaynumeros.blogspot.com.es/2017/06/numeros-piramidales-4-

pentagonos.html

Page 194: Números y formas - Aprender y divertirse con hoja de …hojamat.es/publicaciones/numform.pdf · Este es uno de los capítulos de las Matemáticas en los que se aprecia ... también

194

Como en casos anteriores, construimos las sumas parciales: 1, 1+6=7,

1+6+18=25, 1+6+18+40=65,…y obtenemos:

1, 7, 25, 65, 140, 266, 462, 750, 1155, 1705, 2431, 3367, 4550, 6020,

7820, 9996, 12597, 15675, 19285, 23485, 28336, 33902,…

Estos serán, pues los piramidales pentagonales de cuatro

dimensiones. Los puedes estudiar en

http://oeis.org/A001296

Expresión algebraica

Lo que sigue lo hemos aplicado en bastantes números figurados, por lo

que simplificaremos las explicaciones. En primer lugar, abrimos

nuestro interpolador para números naturales

http://www.hojamat.es/sindecimales/aritmetica/herramientas/herrarit.ht

m#newton

Escribimos en él los primeros términos de la sucesión que hemos

formado.

Leemos los coeficientes de (x-1), (x-1)(x-2), (x-1)(x-2)(x-3),…y

formamos el polinomio interpolador, que según la anulación de

diferencias quintas, será de cuarto grado:

Page 195: Números y formas - Aprender y divertirse con hoja de …hojamat.es/publicaciones/numform.pdf · Este es uno de los capítulos de las Matemáticas en los que se aprecia ... también

195

1+6*(x-1)+6*(x-1)*(x-2)+10/6*(x-1)*(x-2)*(x-3)+3/24*(x-1)*(x-2)*(x-3)*(x-

4)

Lo simplificamos en la página de WolframAlpha y obtenemos

Expresión que coincide con una de las contenidas en

https://oeis.org/A001296

Son números de Stirling

Los pentagonales que estamos estudiando son un caso particular de

los números de Stirling de segunda clase para los índices (n+2,n),

como puedes ver en los elementos subrayados de nuestra tabla

(La tienes alojada en

http://www.hojamat.es/sindecimales/combinatoria/herramientas/herrco

mb.htm#nume)

Si deseas conocer mejor estos números puedes acudir a

https://es.wikipedia.org/wiki/N%C3%BAmeros_de_Stirling_de_segunda

_especie

Page 196: Números y formas - Aprender y divertirse con hoja de …hojamat.es/publicaciones/numform.pdf · Este es uno de los capítulos de las Matemáticas en los que se aprecia ... también

196

Relación con números triangulares

En https://oeis.org/A001296 figura una propiedad interesante, debida a

Jon Perry, y es que cada número piramidal pentagonal de cuatro

dimensiones es suma de números triangulares multiplicado cada uno

por su número de orden, es decir:

𝑃𝐼𝑅5_4(𝑛) = ∑ 𝑗 × 𝑇(𝑗)

𝑛

𝑗=1

Esto se cumple para los primeros términos. Si tomamos los

triangulares 1, 3, 6, 10, 15,…y formamos las sumas 1*1=1, 1*1+2*3=7,

1*1+2*3+3*6=25, 1*1+2*3+3*6+4*10=65,…y comprobamos que se

cumple para los primeros términos. Acudimos a la inducción completa.

Bastará demostrar que PIR5_4(n+1)-PIR5_4(n) = (n+1)*T(n+1). En

efecto, es cuestión de Álgebra:

PIR5_4(n+1)-PIR5_4(n)=((n+1)(n+2)(n+3)(3(n+1)+1)-

n(n+1)(n+2)(3n+1))/24

Simplificamos y queda:

PIR5_4(n+1)-

PIR5_4(n)=(n+1)(n+2)(12n+12)/24=(n+1)(n+1)(n+2)/2=(n+1)T(n+1)

Luego la propiedad es extensible a todos los términos.

En la misma página de OEIS, J. M. Bergot interpreta esta propiedad

como la suma de todos los productos de dos elementos del conjunto

{1, 2, …, n} tomados de todas las formas posibles, sin tener en cuenta

el orden). Por ejemplo, 1*1 + 1*2 + 1*3 + 2*2 + 2*3 + 3*3 = 25.

Es sencillo de comprender, ya que si desarrollamos la suma de cada

triangular por su número de orden, basta recordar que cada triangular

es suma de números consecutivos, luego la suma queda:

1*T(1)+2*T(2)+3*T(3)+4*T(4) =

1*1+2(1+2)+3*(1+2+3)+4*(1+2+3+4)+…

Page 197: Números y formas - Aprender y divertirse con hoja de …hojamat.es/publicaciones/numform.pdf · Este es uno de los capítulos de las Matemáticas en los que se aprecia ... también

197

Se ve que en la suma están contenidos todos los productos de cada

número natural por sus precedentes, tal como afirma J. M. Bergot.

Es un buen ejercicio de hoja de cálculo construir una tabla que refleje

este resultado. Hay que manejar bien las referencias absolutas y

relativas. Un buen ejercicio de aprendizaje.

En la tabla que hemos construido se destaca el conjunto de sumandos

que forman el piramidal pentagonal 140:

Piramidales hexagonales

Según nuestra política de no cansar con los temas, a los piramidales

hexagonales les dedicaremos menos espacio y con ellos terminaremos

el recorrido por los que tienen cuatro dimensiones.

Las operaciones que hay que realizar son:

(1) Acumulación de piramidales hexagonales de tres dimensiones

Partimos de los piramidales hexagonales de tres dimensiones:

1, 7, 22, 50, 95, 161, 252, 372, 525, 715, 946, 1222, 1547, 1925, 2360,

2856, 3417, 4047, 4750, 5530, 6391, 7337, 8372,…

(Ver http://hojaynumeros.blogspot.com.es/2017/07/numeros-

piramidales-5-hexagonos.html)

Los acumulamos en sumas parciales:

Page 198: Números y formas - Aprender y divertirse con hoja de …hojamat.es/publicaciones/numform.pdf · Este es uno de los capítulos de las Matemáticas en los que se aprecia ... también

198

1, 1+7=8, 1+7+22=30, 1+7+22+50=80,…

Nos resultarán así los números piramidales hexagonales de cuatro

dimensiones:

1, 8, 30, 80, 175, 336, 588, 960, 1485, 2200, 3146, 4368, 5915, 7840,

10200, 13056, 16473, 20520, 25270, 30800,…

Los tienes publicados en http://oeis.org/A002417

(2) Obtención de la fórmula algebraica

También para estos números usamos nuestro interpolador

http://www.hojamat.es/sindecimales/aritmetica/herramientas/herrarit.ht

m#newton

Como ya lo hemos desarrollado para otros tipos de números, sólo

insertaremos el volcado de pantalla:

Como en otras ocasiones, observamos la anulación de las diferencias

quintas y copiamos los coeficientes del polinomio en la parte inferior.

1+7*(x-1)+15/2*(x-1)*(x-2)+13/6*(x-1)*(x-2)*(x-3)+4/24*(x-1)*(x-2)*(x-

3)*(x-4)

Simplificamos la expresión en la página de WolframAlpha y nos

devuelve cuatro variantes:

Page 199: Números y formas - Aprender y divertirse con hoja de …hojamat.es/publicaciones/numform.pdf · Este es uno de los capítulos de las Matemáticas en los que se aprecia ... también

199

Aunque la cuarta es la más sintética a efectos de cálculo, nos

quedamos con la primera, porque se puede interpretar en términos de

número combinatorio:

𝑃𝐼𝑅64(𝑛) = 𝑛 (𝑛 + 2

3)

Una curiosa propiedad

Según un comentario de Floor van Lamoen en la página OEIS donde

vienen publicados, se cumple que PIR6_4(n) es la suma de todos los

números que no pueden expresarse de la forma t*(n+1) + u*(n+2) con

t, u enteros no negativos. Desconocemos la demostración de este

hecho, pero nos va a servir para repasar unos conceptos que

desarrollamos en una entrada de este blog

http://hojaynumeros.blogspot.com.es/2010/02/frobenius-y-los-

mcnuggets.html

Según el vocabulario introducido en esa entrada, lo que afirma Floor

van Lamoen es que los números que estamos estudiando son suma

de aquellos que no son representables respecto al conjunto

{n+1,n+2}. En la entrada citada llamábamos número de Frobenius al

máximo número no representable para un conjunto. En el caso de dos

números coprimos a y b, ese número equivale a a*b-a-b. Esto nos

Page 200: Números y formas - Aprender y divertirse con hoja de …hojamat.es/publicaciones/numform.pdf · Este es uno de los capítulos de las Matemáticas en los que se aprecia ... también

200

garantiza que la suma de no representables es finita, y que, por tanto,

puede coincidir con un número de nuestra lista.

En este caso particular, el número de Frobenius para {n+1,n+2} será

(n+1)*(n+2)-(n+1)-(n+2)=n2+n-1

En el caso de n=7, ese número será igual a 7^2+7-1=55. Quiere decir

que los números no representables respecto al conjunto {8,9} serán no

mayores que 55. En efecto, los hemos buscado con hoja de cálculo y

resultan ser

1, 2, 3, 4, 5, 6, 7, 10, 11, 12, 13, 14, 15, 19, 20, 21, 22, 23, 28, 29, 30,

31, 37, 38, 39, 46, 47, 55

Su suma es 588, que coincide con PIR6_4(7), como puedes comprobar

en la lista de arriba.

Podemos repetir esta comprobación para cualquier otro valor. Para ello

necesitamos una función que nos devuelva VERDADERO si un

número n es representable respecto al conjunto {a,b}. Puede ser esta:

Function sumamult(n, a, b) As Boolean ‘Investiga si n=t*a+u*t

Dim i, p, q

Dim es As Boolean

es = False

i = 0 ‘Recorrerá los posibles múltiplos de a

While i <= n And Not es

p = i / a: q = (n - i) / b ‘Encuentra los posibles valores de t y u

If p = Int(p) And q = Int(q) Then ‘Si ambos t y u son enteros, es

representable

es = True

End If

i = i + 1

Wend

sumamult = es

End Function

Page 201: Números y formas - Aprender y divertirse con hoja de …hojamat.es/publicaciones/numform.pdf · Este es uno de los capítulos de las Matemáticas en los que se aprecia ... también

201

Con esta función no es difícil sumar los no representables para un

número dado:

Function suma_no_repr(n)

Dim i, s

s = 0

For i = 1 To n * n + n – 1 ‘Busca hasta el número de Frobenius

If Not sumamult(i, n + 1, n + 2) Then s = s + i ‘Si no es representable,

lo suma

Next i

suma_no_repr = s

End Function

Efectivamente, con ella se reproduce la lista de los piramidales

hexagonales de cuatro dimensiones:

Esto ocurre a menudo en las pequeñas investigaciones matemáticas,

que aparecen conexiones inesperadas, como nos ha ocurrido aquí. Era

difícil imaginar una conexión de números piramidales con el número de

Frobenius.

Page 202: Números y formas - Aprender y divertirse con hoja de …hojamat.es/publicaciones/numform.pdf · Este es uno de los capítulos de las Matemáticas en los que se aprecia ... también

202

Otros números piramidales de cuatro dimensiones

Dejamos aquí el estudio pormenorizado de este tipo de números. Si

deseas avanzar en el número de lados puedes usar este polinomio que

los genera, en el que n es la longitud (en número de puntos) de cada

lado y k el número de lados de la base:

𝑃𝐼𝑅𝐾4(𝑛) =𝑛(𝑛 + 1)(𝑛 + 2)((𝑘 − 2)𝑛 + (6 − 𝑛))

24

Si vas particularizando este polinomio para los valores de k 3, 4, 5 y 6

resultarán las expresiones que hemos estudiado ya. Por ejemplo, para

los que acabamos de estudiar:

K=5: n(n+1)(n+2)(3n+1)/24, tal como hemos obtenido en párrafos

anteriores

K=6: n(n+1)(n+2)(4n)/24, que al simplificar coincide con la que se

presentó más arriba.

También podemos expresar la fórmula general así:

𝑃𝐼𝑅𝐾4(𝑛) = (𝑛 + 2

3)

(𝑘 − 2)𝑛 + (6 − 𝑛)

4

Veamos algunos ejemplos:

K=7

Resultan los piramidales heptagonales de cuatro dimensiones:

1, 9, 35, 95, 210, 406, 714, 1170, 1815,… http://oeis.org/A002418

K=8

1, 10, 40, 110, 245, 476, 840, 1380, 2145,… http://oeis.org/A002419

K=9

1, 11, 45, 125, 280, 546, 966, 1590, 2475,… http://oeis.org/A051740

De esta forma podemos ir aumentando el número de lados, pero cada

vez serán menos interesantes.

Page 203: Números y formas - Aprender y divertirse con hoja de …hojamat.es/publicaciones/numform.pdf · Este es uno de los capítulos de las Matemáticas en los que se aprecia ... también

203

NÚMEROS PIRAMIDALES CENTRADOS

En este capítulo generaremos números piramidales a partir de los

poligonales centrados.

Al igual que los números poligonales centrados se formaban

acumulando contornos de polígonos o de múltiplos de un número,

podríamos también acumular estos números poligonales centrados

mediante sus sumas parciales. Estas sumas se pueden representar

como niveles dentro de una pirámide centrada. Lo vemos en la imagen,

que representa las pirámides centradas de tres lados que

estudiaremos a continuación:

Son pirámides que contienen en el interior de sus bases los polígonos

anteriores.

Pirámides triangulares centradas

Siguiendo un proceso similar al de casos anteriores, partimos de los

números triangulares centrados

Page 204: Números y formas - Aprender y divertirse con hoja de …hojamat.es/publicaciones/numform.pdf · Este es uno de los capítulos de las Matemáticas en los que se aprecia ... también

204

1, 4, 10, 19, 31, 46, 64, 85, 109, 136, 166, 199, 235, 274, 316, 361,

409, 460, 514, 571, 631, 694, 760, 829, 901,…, ya estudiados en las

entradas referidas y en http://oeis.org/A005448

Los acumulamos mediante sumas parciales:

1, 1+4=5, 1+4+10=15, 1+4+10+19=34,… y así hasta completar. De

esta forma se generarán los piramidales triangulares centrados

1, 5, 15, 34, 65, 111, 175, 260, 369, 505, 671, 870, 1105, 1379, 1695,

2056, 2465, 2925, 3439, 4010, 4641, 5335, 6095, 6924, 7825, 8801,

9855, 10990, 12209, 13515, 14911, 16400, 17985, 19669,…

http://oeis.org/A006003

Los tres primeros se corresponden con la imagen del primer párrafo.

En este caso también podemos usar el interpolador de Newton para los

primeros números naturales, que puedes descargar desde

http://www.hojamat.es/sindecimales/aritmetica/herramientas/herrarit.ht

m#newton

Rellenamos los valores de la función con los primeros términos 1, 5,

15, 34, 65, 111, 175 y leemos los coeficientes en la parte inferior:

Page 205: Números y formas - Aprender y divertirse con hoja de …hojamat.es/publicaciones/numform.pdf · Este es uno de los capítulos de las Matemáticas en los que se aprecia ... también

205

El polinomio interpolador será

1+4(x-1)+3(x-1)(x-2)+(x-1)(x-2)(x-3)/2

Simplificamos mediante la web de Wolfram|Alpha y obtenemos

𝑃𝐼𝑅𝐶3(𝑛) =𝑛(𝑛2 + 1)

2

Este es un caso particular de la fórmula contenida en el libro Figurate

Numbers, de Elena Deza y Michel Deza, que sólo incluimos como

comprobación, ya que nos interesa la generación de cada caso

particular. Es esta:

𝑃𝐼𝑅𝐶𝑚(𝑛) =𝑚𝑛3 + (6 − 𝑚)𝑛

6

Particularizando para m=3 resulta la que hemos obtenido.

Por ejemplo, PIRC3(9)=9*82/2=369, que es el noveno término de

nuestra lista de más arriba.

La expresión que hemos obtenido coincide con la que figura en

http://oeis.org/A006003

Uso de Calcupol

Pasamos a usar nuestra calculadora Calcupol,

(http://www.hojamat.es/sindecimales/aritmetica/herramientas/herrarit.ht

Page 206: Números y formas - Aprender y divertirse con hoja de …hojamat.es/publicaciones/numform.pdf · Este es uno de los capítulos de las Matemáticas en los que se aprecia ... también

206

m#figurados) cuyas prestaciones hemos ampliado con la inserción de

una nueva tecla que gestiona estas pirámides centradas

Su funcionamiento es similar a las dedicadas a números poligonales y

otros piramidales: escribimos el orden m (que en este caso valdría 3),

después pulsamos la tecla PIRC con el ratón, escribimos el valor de n,

por ejemplo 7, y terminamos con la tecla =. Nos resultaría, en este

caso del 7, el valor de 175, que coincide con el séptimo término de la

sucesión que estamos estudiando:

Propiedades

Desarrollamos a continuación algunas propiedades de estos números.

Comenzamos con una de Felice Russo incluida en

http://oeis.org/A006003

Se expresa así:

Si escribimos los números naturales en grupos de longitud progresiva

desde el 1, es decir, formamos: 1; 2,3; 4,5,6; 7,8,9,10;… y sumamos

cada grupo, obtenemos los piramidales que estamos estudiando:

1=1

2+3=5

4+5+6=15

7+8+9+10=34

11+12+13+14+15=65

Page 207: Números y formas - Aprender y divertirse con hoja de …hojamat.es/publicaciones/numform.pdf · Este es uno de los capítulos de las Matemáticas en los que se aprecia ... también

207

No es difícil justificarlo. Basta observar que la suma de enteros

consecutivos desde 1 hasta k es el número triangular k(k+1)/2, luego

los grupos formados tendrán como suma la diferencia entre el

triangular correspondiente al último término y el del anterior grupo. Así,

7+8+9+10=T(10)-T(6)=10*11/2-6*7/2=55-21=34, como era de esperar.

Sólo hay que advertir que los índices de esos triangulares son a su vez

triangulares también, 10=4*5/2 y 6=3*4/2, y además consecutivos. Esto

nos permite plantearlo con la variable x:

S(x)=T(x(x+1)/2)-T(x(x-1)/2) = (x(x+1)/2)*(x(x+1)/2+1)/2-(x(x-1)/2)*(x(x-

1)/2+1)/2

Acudimos de nuevo a Wolfram|Alpha para

simplificar y comprobamos:

Nos resulta la fórmula esperada de estos

piramidales centrados, luego la propiedad es cierta.

Propiedad combinatoria

Los piramidales centrados que estamos estudiando son suma de tres

números combinatorios a partir del 15:

𝑎(𝑘) = (𝑛3

) + (𝑛 + 1

3) + (

𝑛 + 23

)

Basta desarrollar: ((n)(n-1)(n-2)+(n+1)(n)(n-1)+(n+2)(n+1)(n))/6=

Page 208: Números y formas - Aprender y divertirse con hoja de …hojamat.es/publicaciones/numform.pdf · Este es uno de los capítulos de las Matemáticas en los que se aprecia ... también

208

=n/6*(n2-3n+2+n2-1+n2+3n+2)

=n/6*(3n2+3)=(n3+n)/2

Significa que si recorremos la cuarta diagonal del triángulo aritmético y

sumamos de tres en tres, resultarán los números que estamos

estudiando:

En una imagen tomada de la Wikipedia hemos señalado los números

que debemos sumar:

Así, 1+4+10=15, 4+10+20=34, 10+20+35=65,…

Relación con los triangulares

𝑃𝐼𝑅𝐶3(𝑛) = 𝑇(𝑛) + 𝑛𝑇(𝑛 − 1)

(Bruno Berselli, Jun 07 2013)

Es claro su significado: Si a un número triangular le sumamos el

anterior multiplicado por el número de orden del primero, resulta un

piramidal triangular centrado.

Así 3+2*1=5, 6+3*3=15, 10+4*6=34…

En forma de tabla:

Page 209: Números y formas - Aprender y divertirse con hoja de …hojamat.es/publicaciones/numform.pdf · Este es uno de los capítulos de las Matemáticas en los que se aprecia ... también

209

Se ha destacado en rojo el cálculo: multiplicar el anterior por el número

de orden y sumar el actual triangular.

Se demuestra con un simple desarrollo:

T(n)+nT(n-1)=n(n+1)/2+n*(n-1)*(n)/2=n/2*(n+1+n2-

n)=n/2(n2+1)=(n3+n)/2

Llegamos a la misma expresión ya conocida.

Piramidales centrados de cuatro lados

Como procedimos con los triangulares, partiremos de los números

poligonales cuadrangulares centrados:

1, 5, 13, 25, 41, 61, 85, 113, 145, 181, 221, 265, 313, 365, 421, 481,

545, 613, 685, 761, 841, 925, 1013, 1105, 1201, 1301, 1405, 1513,

1625, 1741, 1861, 1985, 2113, 2245, 2381…http://oeis.org/A001844

Los puedes repasar en la siguiente entrada de este blog:

http://hojaynumeros.blogspot.com.es/2018/01/poligonales-centrados-

2.html

Realizamos la acostumbrada construcción de sumas parciales:

1, 1+5=6. 1+5+13=19, 1+5+13+25=44,…

Así conseguimos los números piramidales cuadrangulares centrados:

Page 210: Números y formas - Aprender y divertirse con hoja de …hojamat.es/publicaciones/numform.pdf · Este es uno de los capítulos de las Matemáticas en los que se aprecia ... también

210

1, 6, 19, 44, 85, 146, 231, 344, 489, 670, 891, 1156, 1469, 1834, 2255,

2736, 3281, 3894, 4579, 5340, 6181,… http://oeis.org/A005900

Como el uso del interpolador lineal ha sido ya explicado, sólo

insertamos el esquema de cálculo:

Resulta el polinomio

1+5(x-1)+4(x-1)(x-2)+2(x-1)(x-2)(x-3)/3

Se simplifica mediante la web de Wolfram|Alpha y obtenemos:

Es decir:

𝑃𝐼𝑅𝐶4(𝑛) =𝑛(2𝑛2 + 1)

3

Podíamos haberlo deducido de la fórmula general de Elena Deza y

Michel Deza:

𝑃𝐼𝑅𝐶𝑚(𝑛) =𝑚𝑛3 + (6 − 𝑚)𝑛

6

Page 211: Números y formas - Aprender y divertirse con hoja de …hojamat.es/publicaciones/numform.pdf · Este es uno de los capítulos de las Matemáticas en los que se aprecia ... también

211

Para m=4 queda

𝑃𝐼𝑅𝐶4(𝑛) =4𝑛3 + (6 − 4)𝑛

6=

2𝑛3 + 𝑛

3

Estos números coinciden con los octaédricos, como puedes comprobar

en http://oeis.org/A005900. Por tanto, las propiedades que siguen las

comparten ambos tipos de números. Puedes encontrar la causa si

comparas las dos figuras siguientes. La primera corresponde al

número piramidal cuadrangular centrado de orden 3. Por tanto, estará

formado por las capas 1+(1+4)+(1+4+8)=19, tal como vimos en la

sucesión general.

En la segunda figura hemos desplazado capas hacia abajo, y colocado

el centro en la mayor:

Así vemos perfectamente formado el octaedro, que se puede generar

con la suma 1^2+2^2+3^2+2^2+1^2=19, como cabía esperar.

Page 212: Números y formas - Aprender y divertirse con hoja de …hojamat.es/publicaciones/numform.pdf · Este es uno de los capítulos de las Matemáticas en los que se aprecia ... también

212

Este resultado se puede generalizar sin problemas al término n, lo que

nos da la primera propiedad de estos números.

Propiedades

Suma de cuadrados

Los términos de la sucesión que estamos estudiando se pueden

calcular mediante la expresión:

𝑃𝐼𝑅𝐶4(𝑛) = 12 + 22 + 32 + ⋯ + (𝑛 − 1)2 + 𝑛2 + (𝑛 − 1)2 + ⋯ + 32 + 22

+ 12

Este desarrollo los identifica con los números octaédricos, como ya

hemos visto.

Tomamos algún ejemplo:

85=1+4+9+16+25+16+9+4+1

231=1+4+9+16+25+36+49+36+25+16+9+4+1

Productos de sumandos impares

A continuación vemos una interesante propiedad debida a Jon Perry

PIRC4(n) coincide con la suma de todos los productos posibles p*q con

p y q impares tales que p+q=2n

Es una consecuencia directa de la primera propiedad:

𝑃𝐼𝑅𝐶4(𝑛) = 12 + 22 + 32 + ⋯ + (𝑛 − 1)2 + 𝑛2 + (𝑛 − 1)2 + ⋯ + 32 + 22

+ 12

Si recordamos que un cuadrado es suma de impares consecutivos,

obtendremos sumas repetidas que se podrán agrupar en productos:

Page 213: Números y formas - Aprender y divertirse con hoja de …hojamat.es/publicaciones/numform.pdf · Este es uno de los capítulos de las Matemáticas en los que se aprecia ... también

213

PIRC4(3)=19=1+1+3+1+3+5+1+3+1=1*5+3*3+5*1

PIRC4(4)=44=1+1+3+1+3+5+1+3+5+7+1+3+5+1+3+1=1*7+3*5+5*3+7

*1

Para el caso general no sería difícil ir agrupando los impares.

Como coeficiente de una potencia

Esta propiedad sólo la comprobaremos en algún caso concreto. Pues

supone pesados cálculos algebraicos que no hay por qué abordar

ahora. Lo dejamos para quién se atreva.

Estos números coinciden con el máximo coeficiente del desarrollo

de (1+x+x2+x3+…xk)4

Lo comprobamos para el 44:

Escribimos (1+x+x^2+x^3)^4 en la página de WolframAlpha:

Leemos su desarrollo más abajo:

Comprobamos que el mayor coeficiente es 44.

A continuación insertamos un recorte de pantalla de wxMaxima en el

que se comprueba la propiedad para exponente 5:

Page 214: Números y formas - Aprender y divertirse con hoja de …hojamat.es/publicaciones/numform.pdf · Este es uno de los capítulos de las Matemáticas en los que se aprecia ... también

214

También aquí el mayor coeficiente es 85, el siguiente elemento de la

lista.

Propiedad combinatoria

El número piramidal cuadrangular centrado de orden n equivale al

número de conjuntos ordenados (w,x,y,z) con todos sus términos en

{1,...,n} tales que w+x=y+z. (Clark Kimberling, Jun 02 2012)

No es difícil razonarlo. Las posibles sumas w+x son 2, 3,…,2n. Sus

posibilidades van creciendo al principio y disminuyendo al final. Así:

Suma 2 o suma 2n: w, x tiene 1 posibilidad , (1+1 o n+n); y+z otra:

luego sale 1=1^2

Suma 3 o suma 2n-1: w, x tiene 2 posibilidades, (1+2, 2+1 o n-1+n.

n+n-1); y+z otras 2, luego al combinar ambas posibilidades quedan 4=

2^2

Suma 4 o suma 2n-2: con un razonamiento similar llegaríamos a 3^2

Así seguiríamos, con lo que volvemos a la propiedad básica, y es que

el total sería igual a

𝑃𝐼𝑅𝐶4(𝑛) = 12 + 22 + 32 + ⋯ + (𝑛 − 1)2 + 𝑛2 + (𝑛 − 1)2 + ⋯ + 32 + 22

+ 12

Page 215: Números y formas - Aprender y divertirse con hoja de …hojamat.es/publicaciones/numform.pdf · Este es uno de los capítulos de las Matemáticas en los que se aprecia ... también

215

Como no hay prisa por resolver las cuestiones, comprobamos esta

propiedad con nuestra hoja Cartesius:

Con Cartesius

Descargamos la hoja desde

http://www.hojamat.es/sindecimales/combinatoria/herramientas/herrco

mb.htm#cartesius

Comprobamos la propiedad para el caso 6. Para ello planteamos:

Estamos pidiendo que se combinen cuatro elementos (corresponden a

w, x, y, z de la propiedad), que deberán estar contenidos en el rango

1..6 y que la suma de los dos primeros x1+x2 coincida con la de los

segundos x3+x4.

Desarrollamos el planteo con el botón Iniciar y, efectivamente, resultan

146 casos

Aunque no nos cabe en este documento, podemos, con la función SI

seleccionar los resultados cuyos dos primeros elementos sumen, por

ejemplo, 5. En el recorte de la imagen asignamos un 1 a los casos en

los que w+x=5

Page 216: Números y formas - Aprender y divertirse con hoja de …hojamat.es/publicaciones/numform.pdf · Este es uno de los capítulos de las Matemáticas en los que se aprecia ... también

216

Después bastaría contar los unos y nos resultarían 16=4^2, tal como

razonamos más arriba.

Piramidales pentagonales centrados

Formación

Lo explicamos de forma esquemática, pues es un procedimiento que

hemos desarrollado anteriormente. Insertamos enlaces para una mejor

comprensión. Procederemos de la misma forma en los siguientes tipos.

Tomamos los números poligonales pentagonales centrados:

1, 6, 16, 31, 51, 76, 106, 141, 181, 226, 276, 331, 391, 456, 526, 601,

681, 766, 856, 951, 1051, 1156, 1266, 1381, 1501, 1626, 1756, 1891,

2031, 2176, 2326, 2481, 2641, 2806, 2976,… http://oeis.org/A005891

http://hojaynumeros.blogspot.com.es/2018/01/poligonales-centrados-

2.html

Sobre ellos acumulamos sumas parciales

1, 1+6=7, 1+6+16=23, 1+6+16+31=54,…

Y nos queda

1, 7, 23, 54, 105, 181, 287, 428, 609, 835, 1111, 1442, 1833, 2289,

2815, 3416, 4097, 4863, 5719, 6670, 7721, 8877,

10143,…http://oeis.org/A004068

Extraemos la expresión genera con nuestro interpolador:

Page 217: Números y formas - Aprender y divertirse con hoja de …hojamat.es/publicaciones/numform.pdf · Este es uno de los capítulos de las Matemáticas en los que se aprecia ... también

217

Copiamos los coeficientes de abajo para obtener el polinomio

interpolador, y resulta:

P(x)=1+6(x-1)+5(x-1)(x-2)+5(x-1)(x-2)(x-3)/6

Simplificamos en la página de WolframAlpha:

O bien

Es decir:

𝑃𝐼𝑅𝐶5(𝑛) =𝑛(5𝑛2 + 1)

6

A partir de la fórmula de Deza también se obtiene:

Page 218: Números y formas - Aprender y divertirse con hoja de …hojamat.es/publicaciones/numform.pdf · Este es uno de los capítulos de las Matemáticas en los que se aprecia ... también

218

𝑃𝐼𝑅𝐶5(𝑛) =5𝑛3 + (6 − 5)𝑛

6=

5𝑛3 + 𝑛

6

Puedes ir engendrando así los términos de la sucesión o usar nuestra

calculadora Calcupol. Ahora cambiamos el método. Ábrela y concreta

en su parte derecha que deseas usar piramidales centrados y marca 5

como orden:

Después escribe un 1 en pantalla y ve usando la tecla PROX paso a

paso, y obtendrás la sucesión 1, 7, 23, 54, 105, 181, 287,…Después,

con la tecla ANT los puedes recorrer descendiendo hasta el 1.

También puedes encontrar un término más alejado. Por ejemplo, con la

secuencia de teclas 5 PIRC 30 = obtendrás el término 30, que resulta

ser 22505. Si ahora usas PROX y ANT puedes descubrir los términos

más cercanos a él.

Propiedades de estos números

Si los piramidales cuadrados centrados los interpretamos como

octaedros, estos pentagonales los podemos convertir en decaedros, es

decir en poliedros de diez caras. Así lo interpreta la sucesión de OEIS

A004068, como ves en su inicio:

A004068 Number of atoms in a decahedron with n shells.

0, 1, 7, 23, 54, 105, 181, 287, 428, 609, 835, 1111, 1442, 1833, 2289,

2815, 3416, 4097, 4863, 5719, 6670, 7721, 8877, 10143, 11524,

Page 219: Números y formas - Aprender y divertirse con hoja de …hojamat.es/publicaciones/numform.pdf · Este es uno de los capítulos de las Matemáticas en los que se aprecia ... también

219

13025, 14651, 16407, 18298, 20329, 22505, 24831, 27312, 29953,

32759, 35735, 38886, 42217, 45733, 49439,…

Como es una cuestión geométrica y el sentido de la palabra decaedro

es ambiguo, dejamos esta interpretación en este punto.

Otra interpretación de la fórmula

La expresión general del valor de estos números se puede escribir de

otra forma:

𝑃𝐼𝑅𝐶5(𝑛) =5𝑛3 + 𝑛

6= 𝑛3 −

𝑛3 − 𝑛

6= 𝑛3 −

(𝑛 + 1)𝑛(𝑛 − 1)

6

Esta, a su vez equivale a

𝑃𝐼𝑅𝐶5(𝑛) = 𝑛3 − (𝑛 + 1

3)

Llegamos a algo interesante, y es que la fórmula se reduce a un cubo y

a un número combinatorio.

Relación con la Combinatoria

La última expresión de la fórmula se puede interpretar como una

diferencia entre combinaciones con repetición de n elementos tomados

de 3 en 3 y las combinaciones de n+1 elementos también de 3 en 3.

Esta consideración nos lleva a una interpretación combinatoria similar

a otra que descubrimos para los piramidales centrados de 4 lados.

Page 220: Números y formas - Aprender y divertirse con hoja de …hojamat.es/publicaciones/numform.pdf · Este es uno de los capítulos de las Matemáticas en los que se aprecia ... también

220

a(n+1) equivale al número de tripletas (w,x,y) con términos

comprendidos en {0,...,n} y tales que x+y>=w. Esta propiedad también

es debida a Clark Kimberling.

Antes de razonar nada, lo desarrollaremos mediante nuestra

herramienta Cartesius, que construye productos cartesianos

condicionados

(http://www.hojamat.es/sindecimales/combinatoria/herramientas/herrco

mb.htm#cartesius)

Usaremos este planteo para el caso n=3:

En la primera línea pedimos un producto de tres factores. Después,

que pertenezcan al intervalo (0,..3) y, finalmente, que el primero sea

menor o igual que la suma de los otros dos.

El resultado es igual a 54 casos, que es el cuarto término de la

sucesión. Veamos en detalle los valores de x1:

El valor x1=0 aparece sin restricciones, ya que es menor o igual que

cualquier otro elemento. En total 16 veces:

Page 221: Números y formas - Aprender y divertirse con hoja de …hojamat.es/publicaciones/numform.pdf · Este es uno de los capítulos de las Matemáticas en los que se aprecia ... también

221

El valor x1=1 ya tiene una restricción, que es (1, 0, 0), luego se

presentará 16-1 veces:

De igual forma, x1=2 aparece 16-3=13 veces, donde 3 son las

combinaciones que forman las excepciones: (2,0,0), (2,1,0) y (2,0,1)

Page 222: Números y formas - Aprender y divertirse con hoja de …hojamat.es/publicaciones/numform.pdf · Este es uno de los capítulos de las Matemáticas en los que se aprecia ... también

222

Por último, el 3 sólo aparecerá en 16-6=10 casos

Se ve, y se puede generalizar fácilmente, que lo que se le va restando

a cada 16 un número triangular:

16+16-1+16-3+16-6=64-1-3-6=54

Para n=4 obtendríamos:

5^3=125, luego la expresión que acabamos de obtener se convertiría

en

25+25-1+25-3+25-6+25-10=125-(1+3+6+10)=105, como era de

esperar. Los números triangulares representan las combinaciones de

dos en dos que representan a las excepciones.

La suma de triangulares equivale al número piramidal triangular o

tetraedro, tal como puedes comprobar en nuestra entrada

Page 223: Números y formas - Aprender y divertirse con hoja de …hojamat.es/publicaciones/numform.pdf · Este es uno de los capítulos de las Matemáticas en los que se aprecia ... también

223

http://hojaynumeros.blogspot.com.es/2017/04/numeros-piramidales-2-

tetraedros.html

En ella vemos que equivale a un número combinatorio

Ajustando índices nos queda la expresión ya vista para los piramidales

que estamos estudiando:

𝑃𝐼𝑅𝐶5(𝑛) = 𝑛3 − (𝑛 + 1

3)

Así que la propiedad es cierta.

Todo este estudio nos da otra interpretación geométrica para estos

piramidales centrados de orden 5, y es que son la diferencia entre un

número cúbico e lado n y un tetraedro de lado n-1.

Suma de valores de un polinomio

Traducimos una propuesta de Reinhard Zumkeller, Nov 11 2012:

Otra expresión para estos números es

𝑃𝐼𝑅𝐶5(𝑛) = ∑ 𝑛2 − 𝑛𝑘 + 𝑘2

𝑛

𝑘=1

En efecto, si sumamos estos términos, obtenemos los piramidales

centrados pentagonales:

Page 224: Números y formas - Aprender y divertirse con hoja de …hojamat.es/publicaciones/numform.pdf · Este es uno de los capítulos de las Matemáticas en los que se aprecia ... también

224

Para demostrarlo recordemos que la suma de n naturales es n(n+1)/2 y

la de sus cuadrados n(n+1)(2n+1)/6. Por tanto, al sumar n^2+nk+k^2

obtendremos:

PIRC(5,n)=n(n+1)(2n+1)/6-n*n*(n+1)/2+n*n^2

Lo simplificamos en la página de WolframAlpha y nos queda

comprobado:

Volvemos a la expresión inicial.

Otros números piramidales centrados

Hexagonales

Con estos números, como veremos, el inicio del estudio seguirá un

camino más simple:

Partimos de los poligonales hexagonales centrados:

1, 7, 19, 37, 61, 91, 127, 169, 217, 271, 331, 397, 469, 547, 631, 721,

817, 919, 1027, 1141, 1261, 1387, 1519, 1657, 1801, 1951, 2107,

2269, 2437, 2611, 2791, 2977, 3169, 3367, 3571, 3781, 3997

(http://oeis.org/A003215

Y

Page 225: Números y formas - Aprender y divertirse con hoja de …hojamat.es/publicaciones/numform.pdf · Este es uno de los capítulos de las Matemáticas en los que se aprecia ... también

225

http://hojaynumeros.blogspot.com.es/2018/01/poligonales-centrados-

2.html)

En esta entrada nuestra incluimos su expresión, que es una diferencia

de cubos consecutivos

Por tanto, si para construir los piramidales debemos ir formando las

sumas parciales, resultarán cubos. En efecto:

1, 1+7=8, 1+7+19=27, 1+7+19+37=64

Luego la sucesión será:

1, 8, 27, 64, 125, 216, 343, 512, 729, 1000, 1331, 1728, 2197, 2744,

3375, 4096, 4913, 5832, 6859, 8000, 9261, 10648, 12167,…

En el boceto siguiente está representado el número 27, que a su vez

contiene el 7 y el 1, en sus tres capas, luego 27=1+1+6+1+6+12

Los términos de la sucesión claramente son cubos. No hay que usar

interpolador para verlo. Si también acudimos a la fórmula de Deza lo

comprobaremos, para n=6

Page 226: Números y formas - Aprender y divertirse con hoja de …hojamat.es/publicaciones/numform.pdf · Este es uno de los capítulos de las Matemáticas en los que se aprecia ... también

226

𝑃𝐼𝑅𝐶6(𝑛) =6𝑛3 + (6 − 6)𝑛

6=

6𝑛3 + 0

6= 𝑛3

Así que estos números, además de ser piramidales centrados,

representarán una figura cúbica. Aclara mucho la equivalencia si vas

tomando grupos de tres unidades en la imagen anterior y te los

imaginas alineados en una trama cúbica:

Al coincidir estos números con los cubos, todas sus propiedades se

desprenderán de ese carácter, lo que les quita interés.

Suma de grupos de impares consecutivos

Añadimos esta propiedad porque se puede interpretar como un número

trapezoidal. Cada número heptagonal centrado equivale a la suma de

uno de estos grupos:

{1}, {3, 5},{7, 9, 11}, {13, 15, 17, 19},…

1=1

8=3+5

27=7+9+11

64=13+15+17+19

Page 227: Números y formas - Aprender y divertirse con hoja de …hojamat.es/publicaciones/numform.pdf · Este es uno de los capítulos de las Matemáticas en los que se aprecia ... también

227

Las sumas se pueden representar mediante trapecios. Por ejemplo, la

última formaría esta imagen:

Para comprobarlo algebraicamente, usaremos, como en casos

anteriores, los números triangulares. Sabemos que la suma de impares

equivale al cuadrado de su número, pero estos grupos se han ido

eligiendo siguiendo los triangulares, por lo que su valor coincidirá con

la diferencia de cuadrados de dos triangulares consecutivos. Así:

𝑛2(𝑛 + 1)2

4−

𝑛2(𝑛 − 1)2

4= 𝑛3

Heptagonales

Partimos de los poligonales centrados de siete lados

1, 8, 22, 43, 71, 106, 148, 197,… http://oeis.org/A069099

Acumulamos:

1, 1+8=9, 9+22=31, 31+43=74,…y obtenemos:

1, 9, 31, 74, 145, 251, 399,…

Están publicados en http://oeis.org/A004126

Su expresión es fácil de obtener con la fórmula de Deza:

Page 228: Números y formas - Aprender y divertirse con hoja de …hojamat.es/publicaciones/numform.pdf · Este es uno de los capítulos de las Matemáticas en los que se aprecia ... también

228

𝑃𝐼𝑅𝐶7(𝑛) =7𝑛3 + (6 − 7)𝑛

6=

7𝑛3 − 𝑛

6

Propiedades

Como suma de triangulares

Casi todos los números figurados presentan relaciones sencillas con

los números triangulares. En este caso es:

El piramidal heptagonal centrado de orden n equivale a la suma de n

triangulares comenzando por T(n)

Por ejemplo:

31=6+10+15

74=10+15+21+28

Para el caso n basta recordar que la suma de los primeros números

triangulares equivale a n(n+1)(n+2)/6, luego la suma de sólo cuatro

será la diferencia entre la suma de los 2n-1 primeros menos la suma

de los n-1 primeros. Lo desarrollamos:

(2n-1)2n(2n+1)/6-(n-1)n(n+1)/6

Simplificando en Wolfran-Alpha:

Obtenemos:

Page 229: Números y formas - Aprender y divertirse con hoja de …hojamat.es/publicaciones/numform.pdf · Este es uno de los capítulos de las Matemáticas en los que se aprecia ... también

229

Coincide con la expresión obtenida más arriba, luego la propiedad es

verdadera.

Fórmula combinatoria

La propiedad anterior se puede expresar así:

𝑃𝐼𝑅𝐶7(𝑁) = (2𝑛 − 1

3) − (

𝑛 − 13

)

Octogonales

Los poligonales octogonales centrados, que no llegamos a estudiarlos

en este blog, equivalen a los cuadrados de los números impares, como

puedes ver en OEIS:

También los puedes recorrer con nuestra calculadora Calcupol

(http://www.hojamat.es/sindecimales/aritmetica/herramientas/herrarit.ht

m#figurados)

Eliges el tipo Centrado de orden 8

Page 230: Números y formas - Aprender y divertirse con hoja de …hojamat.es/publicaciones/numform.pdf · Este es uno de los capítulos de las Matemáticas en los que se aprecia ... también

230

Escribes un 1 en pantalla y vas pulsando la tecla PROX, con lo que

aparecerán en pantalla los cuadrados de los impares.

Acumulamos esos cuadrados mediante sumas parciales

1+9=10

1+9+25=35

1+9+25+49=84

Obtendremos la sucesión

1, 10, 35, 84, 165, 286, 455, 680, 969, 1330, 1771, 2300, 2925, 3654,

4495, 5456, 6545, 7770, 9139, 10660,

12341,…(http://oeis.org/A000447)

Estos serán los piramidales octogonales centrados.

De la fórmula de Deza se deduce:

𝑃𝐼𝑅𝐶8(𝑛) =8𝑛3 + (6 − 8)𝑛

6=

8𝑛3 − 2𝑛

6=

4𝑛3 − 𝑛

3=

𝑛(4𝑛2 − 1)

3

También se puede escribir como

𝑃𝐼𝑅𝐶8(𝑛) =𝑛(2𝑛 − 1)(2𝑛 + 1)

3

Se puede comprobar:

PIRC8(3)=3*5*7/3=35; PIRC8(4)=4*7*9/3=84

Coincidencia con tetraedros

Page 231: Números y formas - Aprender y divertirse con hoja de …hojamat.es/publicaciones/numform.pdf · Este es uno de los capítulos de las Matemáticas en los que se aprecia ... también

231

Si aplicamos la fórmula obtenida a los números impares nos resultará:

𝑃𝐼𝑅𝐶8(2𝑛 − 1) =(2𝑛 − 1)(4𝑛 − 1)(4𝑛 − 3)

3= (

4𝑛 − 33

)

Los números combinatorios de orden 3 coinciden con los piramidales

triangulares o tetraedros.

http://hojaynumeros.blogspot.com.es/2017/04/numeros-piramidales-2-

tetraedros.html

Así que estos estos octogonales que estamos estudiando coinciden

con los tetraedros en los lugares impares:

Piramidales octogonales centrados:

1, 10, 35, 84, 165, 286, 455, 680, 969, 1330, 1771, 2300, 2925, 3654,

4495, 5456, 6545, 7770,…

Piramidales triangulares:

1, 4, 10, 20, 35, 56, 84, 120, 165, 220, 286, 364, 455, 560, 680, 816,

969, 1140, 1330, 1540, 1771, 2024, 2300, 2600, 2925,…

Cada dos de estos coinciden con los de arriba.

Este tema se ha alargado mucho. Es el momento de cortar y dejar el

resto para investigar.

1, 10, 35, 84, 165

Page 232: Números y formas - Aprender y divertirse con hoja de …hojamat.es/publicaciones/numform.pdf · Este es uno de los capítulos de las Matemáticas en los que se aprecia ... también

232

PITÁGORAS Y SUS TERNAS

EL SUEÑO DE LEWIS CARROLL

La lectura de la biografía de Lewis Carroll me ha sugerido el proponer

la siguiente búsqueda, inspirada en un problema que le impidió dormir

una noche:

¿Qué números enteros equivalen al área de un triángulo

rectángulo de lados también enteros, de tres formas distintas?

La primera solución es 840, porque las tres ternas

15, 112 y 113

24, 70 y 74

40, 42 y 58

pertenecen a lados de triángulos rectángulos de área 840.

¿Cuáles son las siguientes soluciones?

Área 840 Triángulos de lados 15,112 y 113 24,70 y 74 40,42 y 58

Aquí tenéis las siguientes:

Área 3360 Lados 30,224 y 226 48, 140 y 148 80,84 y 116

Área 7560 Lados 45, 336 y 339 72,210 y 222 120,126 y 17

Área 10920 Lados 56,390 y 394 105,208 y 233 120,182 y 218

Área 13440 Lados 60,448 y 452 96,280 y 296 160,168 y 232

Área 21000 Lados 75,560 y 565 120,350 y 370 200,210 y 290

Los siguientes son 30240, 31920, 41160 y 43680. Quedáis invitados a

encontrar los tres triángulos correspondientes a cada uno. Una vez que

Page 233: Números y formas - Aprender y divertirse con hoja de …hojamat.es/publicaciones/numform.pdf · Este es uno de los capítulos de las Matemáticas en los que se aprecia ... también

233

se conocen las soluciones ya no es difícil encontrar los catetos

apropiados.

OBLONGOS Y PITAGÓRICOS

Una cuestión que ha dado juego desde los tiempos de Girard y Fermat

y que permite recorrer alternativas de cálculo es la siguiente:

De todos los triángulos rectángulos de lados enteros ¿Cuáles

cumplen que la diferencia entre los catetos es la unidad?

El primero que la cumple es el popular de lados 3, 4 y 5, pues la

diferencia entre 3 y 4 es una unidad. ¿Habrá más? ¿Cómo abordamos

el cálculo? Casi todos los caminos nos llevan a una ecuación diofántica

de segundo grado, pero hay que ver cuál y cómo resolverla. También

podemos intentar una búsqueda con hoja de cálculo.

Quizás fuera prudente comenzar con esta última posibilidad. Si lo

intentas descubrirás al menos estas soluciones:

3, 4 y 5 20, 21 y 29 119, 120 y 169 696, 697 y 985 4059, 4060 y 5741 23660,23661 y 33461 137903, 137904 y 195025 ¿Cómo organizar una búsqueda de soluciones para

x2+(x+1)2 =y2

con hoja de cálculo?

Presentamos dos propuestas:

Elemental:

Page 234: Números y formas - Aprender y divertirse con hoja de …hojamat.es/publicaciones/numform.pdf · Este es uno de los capítulos de las Matemáticas en los que se aprecia ... también

234

Rellena una columna con los primeros números naturales

consecutivos, y en la columna de su derecha auméntalos en una

unidad. Supongamos que has comenzado en las celdas B4 y C4

respectivamente. En ese caso puedes rellenar la celda D4 con la

fórmula B4^2+C4^2, y en la E4 una condición que nos devuelva la

palabra “Vale” si es cuadrado perfecto:

SI(D4=(ENTERO(RAIZ(D4))^2; “Vale”;””).

De esta forma descubriremos las soluciones, con algo de paciencia,

tiempo y muchas filas de hoja de cálculo:

Con Basic

La misma idea de construir una lista para X, otra para X+1 y una

tercera en la que buscamos los cuadrados perfectos se puede construir

en Basic. X lo almacenamos en la variable i, X+1 en la j, y la

hipotenusa en k. Una sentencia IF nos presenta las soluciones en las

que k es un entero.

Con este código se buscan las soluciones para números inferiores a

1000000.

Sub busquedas Dim i,j,k for i=1 to 1000000 j=i+1 k=i^2+j^2 if k=Int(sqr(k))^2 then

Page 235: Números y formas - Aprender y divertirse con hoja de …hojamat.es/publicaciones/numform.pdf · Este es uno de los capítulos de las Matemáticas en los que se aprecia ... también

235

msgbox(i) msgbox(j) msgbox(sqr(k)) end if next i End Sub

Estudio algebraico

La ecuación x2+(x+1)2 =y2 se puede desarrollar de esta forma: x2+(x+1)2 =y2; 2x2+2x+1=y2; (2x+1)2 + 1 = 2y2 ; (2x+1)2 - 2y2 = -1, por lo que llamando z=2x+1 desembocamos en una ecuación de Pell con segundo miembro igual a -1

Z2-2y2 = -1

Utilizamos la hoja de cálculo pell.ods o pell.xls contenidas en la dirección

http://www.hojamat.es/sindecimales/aritmetica/herramientas/herrarit.htm

con el resultado que indica la imagen:

en la que valdrán las soluciones correspondientes a -1

Z=1; Y=1; Imposible, pues X sería negativo

Z=7; Y=5 X=3; X+1=4; Y=5

Z=41; Y=29 X=20; X+1=21; Y=29

Z=239; Y=169 X=119; X+1=120; Y=169

Z=1393; Y=985 X=696; X+1=697; Y=985

Este método tiene el inconveniente de que depende de la precisión que tenga la hoja de cálculo en los cálculos con coma flotante, lo que hará que se rompa en algún momento la periodicidad de los cocientes, en

Page 236: Números y formas - Aprender y divertirse con hoja de …hojamat.es/publicaciones/numform.pdf · Este es uno de los capítulos de las Matemáticas en los que se aprecia ... también

236

este caso el 2. Por ello se puede completar con una fórmula recursiva que obtenga soluciones exactas conociendo las primeras.

En este ejemplo cada elemento de las distintas celdas cumple la fórmula

an+2 = 2an+1 + an

pero como las soluciones aparecen de forma alternada, deberemos reiterar dos veces, y nos quedará:

an+4 = 2an+3 + an+2 = 2(2an+2 + an+1)+ 2an+1 + an = 4an+2 + 4an+1+ an = 6an+2 - an

Con esta fórmula recursiva se van obteniendo las soluciones sin errores a partir de las dos primeras:

Z0 = 1; Z2 = 7; Z4 = 6*7-1 = 41; Z6 = 6*41-7 =239;…

Y0 = 1; Y2 = 5; Y4 = 6*5-1 = 29; Y6 = 6*29-5 =169;…

Pero no olvidemos que Z es una variable auxiliar Z=2X+1 y que después debemos despejar X

La siguiente lista de ternas, que coincide con la primera que propuso Girard, se ha obtenido mediante esta técnica

1 0 1

5 3 4

29 20 21

169 119 120

985 696 697

5741 4059 4060

33461 23660 23661

195025 137903 137904

1136689 803760 803761

6625109 4684659 4684660

38613965 27304196 27304197

225058681 159140519 159140520

1311738121 927538920 927538921

7645370045 5406093003 5406093004

44560482149 31509019100 31509019101

Page 237: Números y formas - Aprender y divertirse con hoja de …hojamat.es/publicaciones/numform.pdf · Este es uno de los capítulos de las Matemáticas en los que se aprecia ... también

237

259717522849 183648021599 183648021600

1513744654945 1070379110496 1070379110497

8822750406821 6238626641379 6238626641380

51422757785981 36361380737780 36361380737781

299713796309065 211929657785303 211929657785304

Un reto: Fermat propuso una fórmula de recurrencia para generar ternas de este tipo a partir de otras similares. Dada la terna (x,x+1,y), se puede generar otra similar (x’,x’+1,y’)

mediante las fórmulas x’=2x+3y+1 y y’=4x+3y+2.

¿Sabrías demostrarlo? ¿Engendra todas las ternas posibles a partir de 3, 4,5?

Notas

(1) Dados dos catetos X y X+1 de la serie anterior, los siguientes se obtienen sumando los tres lados y restando después un cateto u otro del doble de esa suma:

Por ejemplo, de 3, 4,5 obtenemos suma 12, su doble 24, y restando 3 y 4 por separado obtenemos 20 y 21, que es la siguiente solución junto con 29.

En efecto, de las fórmulas de Fermat

X’ = 3X+2Y+1; Y’ = 4X+3Y+2, junto con la de la suma de los tres, S= 2X+1+Y

obtenemos 2S-X = 4X+2+2Y-X = 3X+2Y+2 = X’+1, que es el nuevo

cateto mayor, y de forma similar obtendríamos X’ como 2S-(X+1)

(2) Según G. Hutton, si llamamos pr/qr a la r-ésima convergente de √2

en su desarrollo mediante fracciones continuas, las ternas pitagóricas

que hemos presentado con catetos del tipo X y X+1 vendrían dadas

por

Xr=pr*pr+1 Yr=2qr*qr+1

En efecto, la siguiente tabla obtenida con Excel nos permite comprobar

esta propiedad:

Page 238: Números y formas - Aprender y divertirse con hoja de …hojamat.es/publicaciones/numform.pdf · Este es uno de los capítulos de las Matemáticas en los que se aprecia ... también

238

(3) La terna 3, 4,5 está engendrada por las fórmulas clásicas 2uv, u2-v2

y u2+v2 para u=2 y v=1. Si sustituimos u y v por u, v+2u se mantendrá

la misma diferencia entre catetos.

Basta ver que si engendramos los nuevos catetos y los restamos (en

orden contrario) resultará: 2u(v+2u) - (v+2u)2+u2= 2uv+4u2-v2-4u2-

4uv+u2 = u2-v2-2uv, que es la diferencia original.

Esto nos permite engendrar de nuevo la lista que estamos

considerando, tomando, n primer lugar u=2 v=1, y generando con ella

la primera terna 3,4 y 5. Después se aplica la fórmula de recurrencia

un = 2un-1+vn-1 vn = un-1 y se vuelve a generar una terna con ella, que

resultará tener la misma diferencia pero con signo cambiado. Así

hemos generado la lista con hoja de cálculo:

u v x y z

2 1 4 3 5

5 2 20 21 29

12 5 120 119 169

29 12 696 697 985

70 29 4060 4059 5741

169 70 23660 23661 33461

408 169 137904 137903 195025

985 408 803760 803761 1136689

2378 985 4684660 4684659 6625109

5741 2378 27304196 27304197 38613965

13860 5741 159140520 159140519 225058681

Page 239: Números y formas - Aprender y divertirse con hoja de …hojamat.es/publicaciones/numform.pdf · Este es uno de los capítulos de las Matemáticas en los que se aprecia ... también

239

VIAJE DE IDA Y VUELTA A LA GEOMETRÍA

Según leo en el libro “Cardano y Tartaglia. Las matemáticas en el

Renacimiento italiano” de Francisco M. Casalderrey, a Fibonacci le

interesó mucho el estudio del triángulo de lados con medidas enteras

13, 14 y 15, porque la altura correspondiente al lado que mide 14

también tiene medida entera, 12, así como los dos segmentos que

forma en la base, 5 y 9 respectivamente.

¿Existirán más triángulos con esa propiedad?

La respuesta es afirmativa. Existen muchos, y no es difícil encontrarlos.

Uno de ellos, con la misma superficie que el anterior, está formado por

los lados 17, 21 y 10.

¿Podrías encontrar alguno más con medidas inferiores a 50?

Más difícil es que esta propiedad la presenten dos alturas.

Existen algunos triángulos en los que aparece por simetría, como el de

la imagen. Los lados son 25, 25 y 30, las alturas 24, 24 y 20, y los

segmentos en las bases 7, 15 y 18. Todos son enteros.

Page 240: Números y formas - Aprender y divertirse con hoja de …hojamat.es/publicaciones/numform.pdf · Este es uno de los capítulos de las Matemáticas en los que se aprecia ... también

240

¿Existirán triángulos en los que dos alturas presenten segmentos de

medida entera sin acudir a la simetría? Pues también la respuesta es

afirmativa. En la imagen tienes uno:

Los lados miden 70, 65 y 75 respectivamente, una altura de 56 divide

al 75 en dos segmentos de medidas 33 y 42, y la otra altura, de 60,

divide a 70 en 25 y 45.

El hecho de que este triángulo sea semejante al de Fibonacci y posea

una propiedad más amplia nos demuestra que estas cuestiones no son

geométricas, sino aritméticas. Todo depende de si una medida se

expresa como entera o como fraccionaria. Una altura que en el primero

medía 11,2. en este mide 5 veces más, lo que la convierte en entera:

56.

Con lo explicado en el párrafo anterior puedes encontrar triángulos en

los que todos los lados, alturas, segmentos formados por estas en las

bases, perímetro y área sean enteros.

Para conseguir un triángulo con lados, alturas y segmentos en la base

todos enteros puedes seguir estos pasos:

(1) Elige dos ternas pitagóricas, preferentemente primitivas, como 20,

21,29 y 8, 15,17.

(2) Multiplícalas ambas por un valor entero adecuado a fin de unificar

las medidas de sus dos catetos mayores (el que sean los mayores no

Page 241: Números y formas - Aprender y divertirse con hoja de …hojamat.es/publicaciones/numform.pdf · Este es uno de los capítulos de las Matemáticas en los que se aprecia ... también

241

es necesario, pero te garantiza que el triángulo sea acutángulo)

Puedes buscar el MCM de ambos valores. En nuestro ejemplo se

convertirían en 56, 105,119 y 100, 105,145

(3) Arma un primer triángulo tomando como altura el cateto común:

Con esto te garantizas que el seno y el coseno de

los ángulos opuestos a la altura 105 sean números

racionales, y como consecuencia que también lo

sean los del tercer ángulo ¿Por qué?

También tienes garantizada una medida racional

para las alturas y segmentos que quedan, pero no

necesariamente enteros.

(4) En efecto, usando la fórmula (a2+b2-c2)/(2a) para todos los pares de

lados nos resultarán las medidas de los segmentos, necesariamente

racionales. Puedes verlo en un desarrollo con Wiris

A la vista del desarrollo encontrarás los factores por los que hay que

multiplicar (para conseguir una semejanza de triángulos) a fin de que

todas las medidas sean enteras. En este caso por 29 y 17.

Con esto llegamos a la meta:

Page 242: Números y formas - Aprender y divertirse con hoja de …hojamat.es/publicaciones/numform.pdf · Este es uno de los capítulos de las Matemáticas en los que se aprecia ... también

242

Sólo nos queda calcular las alturas y tendremos el triángulo completo:

Puedes analizar también el área, el perímetro y el radio de la

circunferencia inscrita. Sus valores son: A= 1990571310, P=207060 y

R=19227

En definitiva, de la Geometría sólo hemos usado fórmulas, por lo que el

resultado constituye un regreso a la Aritmética de números enteros y

racionales, que es su verdadero sitio, aunque lo hayamos representado

como un triángulo. Quizás por eso le gustaba a Fibonacci.

CENTRADOS Y PITAGÓRICOS

¿Por qué los números cuadrados centrados pueden ser siempre los

términos mayores (la hipotenusa) de ciertas ternas pitagóricas?

Por ejemplo: 612 =602 +112

Es fácil demostrarlo: todo cuadrado centrado se descompone en dos

cuadrados consecutivos:

Page 243: Números y formas - Aprender y divertirse con hoja de …hojamat.es/publicaciones/numform.pdf · Este es uno de los capítulos de las Matemáticas en los que se aprecia ... también

243

La figura se explica por sí misma: el cuadrado centrado de orden 4,

con valor 25, se descompone en cuatro triangulares, dos iguales y los

otros dos consecutivos, que recombinados forman los cuadrados de

orden 3 y 4 respetivamente

Por tanto, CC4 = C4+C3 (CC: Cuadrado centrado, C: cuadrado normal),

lo que nos lleva a la fórmula para los cuadrados centrados CC4 =

n2+(n-1)2 = 2n2 – 2n + 1.

Para demostrar que puede ser el término mayor de una terna

pitagórica elevamos su fórmula al cuadrado y podemos dividirla en dos

cuadrados:

(2n2 – 2n + 1)2 = 4n4 - 8n3 + 8n2 -4n + 1 = (2n2 – 2n)2 + (2n – 1)2

con lo que descubrimos que la terna tiene la estructura X, Y, Y+1, con

X2 = 2y+1

En la siguiente tabla están contenidos los desarrollos de los 10

primeros cuadrados centrados. Algunas ternas son muy populares:

N CCN=Y+1 Y X

2 5 4 3

3 13 12 5

4 25 24 7

5 41 40 9

6 61 60 11

7 85 84 13

8 113 112 15

9 145 144 17

10 181 180 19

¿Se puede demostrar el teorema inverso: Si en una terna pitagórica la

hipotenusa se diferencia en una unidad del cateto mayor, dicha

hipotenusa es un número cuadrado centrado?

Page 244: Números y formas - Aprender y divertirse con hoja de …hojamat.es/publicaciones/numform.pdf · Este es uno de los capítulos de las Matemáticas en los que se aprecia ... también

244

Solución: Y2=(Y-1)2+X2 equivale a que 2Y-1 = X2 es decir Y=(X2-1)/2.

Como X ha de ser impar, sustituimos por 2N-1 y queda que Y = 2n2 –

2n + 1 y por tanto es cuadrado centrado.

CUADRADOS CON TROZOS CONSECUTIVOS

Acabo de leer en (http://maanumberaday.blogspot.com/) que el número

573 tiene la propiedad de que su cuadrado está representado en el

sistema decimal con los dígitos de dos números consecutivos:

5732 = 328329

He puesto a trabajar a mi hoja de cálculo y me ha devuelto siete de

esos números entre 1 y 50000.

Es este un problema interesante, porque el algoritmo se simplifica

mucho si se concibe a la contra: en lugar de recorrer los números uno

a uno y después someterlos a la prueba del cuadrado formado por dos

consecutivos, se cambia la perspectiva, es decir, se construyen los

candidatos a cuadrados y después se prueba si lo son o no. En caso

afirmativo se les extrae la raíz cuadrada para ver la solución.

Sub cuadrado_consecutivos

Input n ‘se concreta hasta dónde llegará la búsqueda. for i=1 to n j=numcifras(i+1) ‘se cuentan las cifras del número i+1 k=i*10^j+i+1 ‘ se forma el posible cuadrado con dos números consecutivos. if escuad(k)=1 then ‘si es cuadrado perfecto se comunica msgbox(sqr(k)) ‘solución msgbox(k) ‘cuadrado end if next i end sub

Page 245: Números y formas - Aprender y divertirse con hoja de …hojamat.es/publicaciones/numform.pdf · Este es uno de los capítulos de las Matemáticas en los que se aprecia ... también

245

He conseguido 7 soluciones hasta el 50000 y una falsa (el 1000): 428,

573, 727, 846, 7810, 36365, 63636

Los tienes en http://oeis.org/A030467

Es muy elegante la versión en PARI. Analizándola puede que desees

estudiar este lenguaje un poco mejor:

{for(x=1,10^7,m=eval(concat(Str(x),Str(x+1)));if(issquare(m,&p),pri

nt(p)))}

DIFERENCIA ENTRE CATETOS

En otro momento estudiamos las ternas pitagóricas en las que la

diferencia entre catetos era igual a 1. Nos podemos plantear también

qué números, aparte del 1, pueden ser diferencia entre catetos en esas

ternas. Estudiaremos algunos aspectos de este problema

(1) Afirmamos que todo número puede ser diferencia entre catetos

en una terna pitagórica.

En efecto, dado cualquier número k, a partir de la terna 3, 4, 5

podemos construir 3k, 4k, 5k con diferencia de catetos k

(2) Más difícil es demostrar que todo número es diferencia de

catetos de infinitas formas distintas. Para ayudarte puedes

demostrar previamente lo siguiente:

Si u y v engendran una terna pitagórica mediante las fórmulas 2uv, u2-

v2 y u2+v2, los valores 2u+v y v engendran otra terna con la misma

diferencia de catetos.

No es difícil. Dados dos valores u, v primos entre sí y de distinta

paridad, engendran la terna pitagórica primitiva u2-v2, 2uv, u2+v2, con

diferencia de catetos igual a u2-v2-2uv.

Page 246: Números y formas - Aprender y divertirse con hoja de …hojamat.es/publicaciones/numform.pdf · Este es uno de los capítulos de las Matemáticas en los que se aprecia ... también

246

Los nuevos valores 2u+v, v son de distinta paridad y primos entre sí

(fácil de ver) y engendran la terna (2u+v)2-u2, 2u(2u+v), (2u+v)2+u2, con

diferencia

(2u+v)2-u2-2u(2u+v)= 4u2+v2+4uv-u2-4u2-2uv = v2-u2+2uv, que es la

misma con signo cambiado.

La nueva terna será primitiva, porque se cumplen las condiciones.

Si lo anterior es cierto, reiterando el procedimiento obtendremos

infinitas ternas con la misma diferencia (salvo signo u orden). Si la

primera es primitiva, todas las demás lo serán ¿Por qué?

Por ejemplo, de u=4, v=3, x=7, y=24, z=25, con diferencia entre catetos

igual a 17, podemos engendrar u=11, v=4, x=88, y=105, z=137, con

105-88 = 17 y después u=26, v=11, x=572, y=555 z=797, y así tantas

como queramos.

(3) Si sólo admitimos ternas primitivas, no todos los números pueden ser diferencia de catetos. Los únicos posibles son 1, 7, 17, 23, 31, 41, 47, 49, 71, 73, 79, 89, 97, 103, 113, 119, ...( http://oeis.org/A058529) La razón es que la diferencia D ha de ser impar, con fórmula D=u2-v2-

2uv = (u+v)2-2v2

Esto obliga a que el número 2 sea resto cuadrático respecto a todos

divisores de D.

Existe un criterio derivado del de Euler, que nos dice que 2 es resto

cuadrático módulo p (primo) si (p2-1)/8 es par (consultar bibliografía), y

esto sólo se cumple si p=8k+1 o p=8k-1.

Los factores primos de los números considerados son del tipo 8k±1.

Como esta estructura algebraica se conserva en el producto, todos los

números presentarán esta estructura

Esta condición se puede concretar aún más: Entre los factores primos

del tipo 8k±1 no pueden estar 2, 3 ni 5. Los primos mayores que 3,

como se demuestra fácilmente, presentan también la estructura 6k±1,

luego los números de la lista también presentarán esta estructura.

Page 247: Números y formas - Aprender y divertirse con hoja de …hojamat.es/publicaciones/numform.pdf · Este es uno de los capítulos de las Matemáticas en los que se aprecia ... también

247

Las dos condiciones han de cumplirse a la vez. Si las combinamos nos

resulta otra definitiva (la puedes obtener tachando los restos que no las

cumplen):

Todos los números de la lista sólo pueden presentar respecto al 24 los

restos 1, 7, 17 o 23.

DOBLADO PITAGÓRICO

Si tomamos un segmento de longitud 31 cm. y lo doblamos por cierto

punto en forma de ángulo recto, podemos completar un triángulo

rectángulo cuya hipotenusa tiene medida entera. No es difícil averiguar

por dónde se puede doblar: basta hacerlo con un segmento de medida

7, con lo que el otro trozo mediría 24 y la hipotenusa 25.

Existen otros números con la misma propiedad: 7, descompuesto en 3

y 4, 23, doblado por 8 y 15, y otros muchos.

Te proponemos una búsqueda elemental, mediante razonamiento, hoja

de cálculo o navegación por la Red:

Además de 7, 23 o 31, ¿qué otros números tienen la propiedad de

engendrar un triángulo rectángulo de medidas enteras con un simple

“doblado”?

Te dejamos este código por si deseas practicar:

(Dado un valor n)

Sub buscar(n)

for i=7 to n

for j=3 to i/2

k=i-j

if escuadrado(k*k+j*j)=1 then

msgbox(i)

msgbox(j)

msgbox(k)

Page 248: Números y formas - Aprender y divertirse con hoja de …hojamat.es/publicaciones/numform.pdf · Este es uno de los capítulos de las Matemáticas en los que se aprecia ... también

248

end if

next j

next i

end sub

Si lo resuelves te llevarás una sorpresa: las soluciones son las mismas de la entrada anterior (salvo el 1) 7, 17, 23, 31, 41, 47, 49, 71, 73, 79, 89, 97, 103, 113, 119, ... y todos sus múltiplos. Lo puedes ver en esta tabla:

7 3 4

14 6 8

17 5 12

21 9 12

23 8 15

28 12 16

31 7 24

34 10 24

35 15 20

41 20 21

42 18 24

46 16 30

47 12 35

49 9 40

49 21 28

51 15 36

56 24 32

62 14 48

63 27 36

68 20 48

69 24 45

70 30 40

71 11 60

73 28 45

La razón estriba en que ambos problemas están relacionados con la

ecuación 2x2-y2=k.

Page 249: Números y formas - Aprender y divertirse con hoja de …hojamat.es/publicaciones/numform.pdf · Este es uno de los capítulos de las Matemáticas en los que se aprecia ... también

249

¿EN CUÁNTAS SUMAS DE CUADRADOS?

Todo comenzó con Fermat

Hay números que se pueden descomponer en suma de dos

cuadrados, pero ¿de cuántas formas? Esta cuestión ha sido ya

abordada en otros blogs de Matemáticas, pero aquí añadiremos

técnicas y algoritmos de hoja de cálculo.

Para conseguir una respuesta a la pregunta formulada se necesitaron

esfuerzos de varios matemáticos, pero todo comenzó con Fermat y su

Teorema de Navidad (lo comunicó a Mersenne el 25 de Diciembre de

1640, pero no lo demostró), y que actualmente expresamos así:

Un número primo se puede descomponer en suma de dos cuadrados

x2+y2 de números enteros si y sólo si es el número 2 o bien es

congruente con 1 módulo 4 (es decir, si es de la forma 4n+1).

El teorema directo es difícil de demostrar, y lo ha sido a lo largo de

siglos mediante diversas técnicas (descenso infinito, enteros

gausianos, etc.), siendo Euler el primero que lo logró. El inverso está a

nuestro alcance. Inténtalo:

Un número primo congruente con 3 módulo 4 no puede

descomponerse en suma de dos cuadrados de números enteros.

Gauss, en la sección 182 de sus Disquisitiones arithmeticae destacó

que esa descomposición es única, salvo orden y signo. Los dos

números x e y han de ser primos entre sí ¿por qué?

De este hecho podemos obtener un criterio marginal: Si un número de

la forma 4n+1 no se puede descomponer en dos cuadrados o bien lo

puede de más de una forma, no es primo.

Esta propiedad de poder descomponerse en suma de dos cuadrados

se mantiene si multiplicamos dos números primos de este tipo, y

Page 250: Números y formas - Aprender y divertirse con hoja de …hojamat.es/publicaciones/numform.pdf · Este es uno de los capítulos de las Matemáticas en los que se aprecia ... también

250

además se puede duplicar el número de posibles sumas. Así, si 13 =

22+32 y 5 = 22+12, al multiplicarlos obtenemos:

65 = 13*5 = 82+12 = 72+42

Esta propiedad se desprende de la famosa identidad:

(a2+b2)(c2+d2)=(ac+bd)2+(ad-bc)2 = (ac-bd)2+(ad+bc)2

que nos viene a decir que este producto también es suma de dos

cuadrados y además de dos formas distintas (si los sumandos son

distintos):

65 = (2*2+3*1)2 +(2*1-3*2)2 = 72+42 (obsérvese que en el cálculo se ha

obtenido -4 y no 4)

65 = (2*2-3*1)2 +(2*1+3*2)2 = 82+12

Ocurre lo mismo si se multiplica el número primo por 2 (elemental

¿no?)

Fórmula de Gauss

Estas propiedades se resumen en un criterio que no vamos a

desarrollar aquí, y es que sólo se pueden descomponer en cuadrados

los números en los que los factores primos del tipo 4n+3 figuren en su

descomposición con exponente par. Gauss fue más allá en esa

sección 182, pues dio una fórmula para contar el número de formas

diferentes en las que se descompone un número en suma de dos

cuadrados con base no negativa:

]

donde ES significa “mínimo entero igual o superior” y los factores que

le siguen se corresponden con los exponentes de los factores del tipo

4n+1 aumentados en una unidad. La fórmula, como advierte Gauss,

sólo es válida si los factores del tipo 4n+3 forman un cuadrado

perfecto.

Así, por ejemplo, el número 325=52*13 se deberá descomponer en

Page 251: Números y formas - Aprender y divertirse con hoja de …hojamat.es/publicaciones/numform.pdf · Este es uno de los capítulos de las Matemáticas en los que se aprecia ... también

251

N=ES((2+1)(1+1)/2)=ES(3*2/2)=ES(3)=3

En efecto, 325=12 + 182 = 62 + 172 = 102 + 152 (tres formas distintas)

Y el número 6664 sólo de una forma, pues 6664 = 23*72*17 y aplicando

la fórmula nos daría

N=ES(1+1)/2 = ES(1)=1, y su descomposición única es 6664=422+702

Actualmente se prefiere considerar todas las sumas de cuadrados

posibles, incluyendo bases negativas y teniendo en cuenta el orden.

Esto multiplica por 8 el número de soluciones cuando x es distinto de y

y ambos son no nulos, y por 4 en caso contrario. Así, el 13 presentaría

ocho soluciones:

13= 22+32 = (-2)2+32 = 22+(-3)2 = (-2)2+(-3)2 = 32 +22 =(-3)2 +22 = 32

+(-2)2 = (-3)2 +(-2)2

Y el 16, cuatro: 16 = 42+02 = (-4)2+02 =02 + 42 = 02 + (-4)2

Igualmente, 8 presentaría también 4: 8 = 42+42 = (-4)2+42 =42 + (-4)2 =

(-4)2 + (-4)2

Aparece el número π

En la sección anterior se presentaba una fórmula para encontrar el

número de descomposiciones distintas en suma de dos cuadrados que

puede presentar un número entero positivo. Vimos dos orientaciones:

buscar sólo sumandos positivos o admitir también los negativos

teniendo en cuenta además el orden

Para un resultado inesperado que obtendremos más adelante vamos a

elegir la segunda opción: encontrar, dado un número entero positivo N,

todos los pares x, y de números enteros tales que x2+y2=N. Al número

de esos pares lo podemos considerar como función de N, lo que nos

permite definir NSC(N)=Número de pares de enteros x, y tales que

x2+y2=N

Para implementar esta función en la hoja de cálculo podemos usar un

código similar al siguiente (comentarios en letra normal):

Page 252: Números y formas - Aprender y divertirse con hoja de …hojamat.es/publicaciones/numform.pdf · Este es uno de los capítulos de las Matemáticas en los que se aprecia ... también

252

Public function nsc(n) dim i,a,b,ns if n=0 then ns=1 Tenemos en cuenta que n puede valer 0 else ns=0 Se inicia la suma for i=0 to sqr(n) Busca el primer sumando a=n-i*I Calcula el Segundo sumando if a=int(sqr(a))^2 then El segundo sumando es un cuadrado if i*i<=a then Esta línea es para no tener en cuenta el orden de los sumandos b=sqr(a) Base del segundo cuadrado if b>0 and i>0 and b<>i then Si ambas bases son positivas y distintas hay 8 posibilidades ns=ns+8 else Si una es cero o son iguales, sólo hay 4 ns=ns+4 end if end if end if next i end if nsc=ns Se recoge el resultado end function

Esta función, si se declara Public se puede usar en la hoja de cálculo y

formar una tabla que compare N con NSC(N):

N NSC(N)

0 1

1 4

2 4

3 0

4 4

5 8

6 0

7 0

8 4

Page 253: Números y formas - Aprender y divertirse con hoja de …hojamat.es/publicaciones/numform.pdf · Este es uno de los capítulos de las Matemáticas en los que se aprecia ... también

253

9 4

10 8

11 0

12 0

13 8

14 0

15 0

16 4

17 8

18 4

19 0

20 8

Aunque su distribución parece ser muy irregular, nos espera una

sorpresa y es que si acumulamos los resultados y vamos calculando el

promedio de NSC conforme crece N, este promedio tiene como límite

π

En la siguiente tabla puedes observar que para N=20 ya se percibe

esta tendencia al límite:

N NSC(N) Acumulada Promedio

0 1 1 1,0000

1 4 5 5,0000

2 4 9 4,5000

3 0 9 3,0000

4 4 13 3,2500

5 8 21 4,2000

6 0 21 3,5000

7 0 21 3,0000

8 4 25 3,1250

9 4 29 3,2222

10 8 37 3,7000

11 0 37 3,3636

12 0 37 3,0833

13 8 45 3,4615

14 0 45 3,2143

Page 254: Números y formas - Aprender y divertirse con hoja de …hojamat.es/publicaciones/numform.pdf · Este es uno de los capítulos de las Matemáticas en los que se aprecia ... también

254

15 0 45 3,0000

16 4 49 3,0625

17 8 57 3,3529

18 4 61 3,3889

19 0 61 3,2105

20 8 69 3,4500

Para N=500 el promedio oscila ya de una forma clara alrededor de

3,14:

498 0 1565 3,1426

499 0 1565 3,1363

500 16 1581 3,1620

501 0 1581 3,1557

502 0 1581 3,1494

y para N=8000 su valor es 3,14213. ¡No nos libramos del número π!

Puedes descargarte las hojas de cálculo en las que hemos

implementado la fórmula de Gauss y la función NSC que cuenta todas

las sumas considerando signos y orden en la dirección

http://hojamat.es/blog/sumacuad.zip

Problema del círculo de Gauss

En el anterior apartado nos aparecía el número π de forma algo

sorprendente. En esta veremos que de sorpresa nada. Todo está

relacionado, y se basa en la solución del llamado Problema del círculo

de Gauss.

No entraremos demasiado en la parte teórica, que podéis consultar en

las páginas

http://mathworld.wolfram.com/GausssCircleProblem.html

Page 255: Números y formas - Aprender y divertirse con hoja de …hojamat.es/publicaciones/numform.pdf · Este es uno de los capítulos de las Matemáticas en los que se aprecia ... también

255

http://en.wikipedia.org/wiki/Gauss_circle_problem

o en el Blog “Juan de Mairena”

http://demairena.blogspot.com/2008/01/1363-el-problema-del-crculo-

de-gauss.html

Lo que presentaremos aquí es su tratamiento con hoja de cálculo, pero

con una pequeña introducción.

Anteriormente desarrollamos los números enteros positivos como

sumas de dos cuadrados de base entera. Estamos en el terreno del

Teorema de Pitágoras, y si representamos todas las soluciones para

un número N dado como catetos de un triángulo, los puntos

representados por ellos se situarán todos en el círculo de radio la raíz

cuadrada de N.

Si con una hoja de cálculo creamos una lista de valores X e Y tales que

X2+Y2<=N, según lo explicado, se rellenarán puntos dentro de un

círculo, lo que representará perfectamente el círculo de Gauss. En la

imagen puedes ver el gráfico correspondiente

a N=22

Para conseguir esta imagen necesitaremos el

algoritmo que encuentre todas las soluciones

para que X2+Y2<=N Una vez conseguida la

lista de soluciones bastará con crear un

gráfico del tipo XY para conseguir la

aproximación al círculo.

Se puede usar un código en el Basic de OpenOffice.org similar al

siguiente:

Sub desarrollo(n) dim i,j,s,t,fi,a,b,x fi=5 StarDesktop.CurrentComponent.sheets(0).GetCellByPosition(3,fi).value=0 StarDesktop.CurrentComponent.sheets(0).GetCellByPosition(4,fi).value=0

Page 256: Números y formas - Aprender y divertirse con hoja de …hojamat.es/publicaciones/numform.pdf · Este es uno de los capítulos de las Matemáticas en los que se aprecia ... también

256

for x=1 to n i=0 a=sqr(x) while i<=a j=x-i*i if j=int(sqr(j))^2 or j=0 then b=sqr(j) for s=-1 to 1 step 2 for t=-1 to 1 step 2 fi=fi+1 StarDesktop.CurrentComponent.sheets(0).GetCellByPosition(3,fi).value=i*s StarDesktop.CurrentComponent.sheets(0).GetCellByPosition(4,fi).value=b*t next t next s end if i=i+1 wend next x End Sub Puedes descargarte las versiones en Excel 2007 y OpenOffice.org 3 desde la dirección http://hojamat.es/blog/circulogauss.zip Reflexión intrascendente

Después de redactar los últimos apartados he recordado que en mis

clases de Matemáticas, al explicar los números reales, utilizábamos el

Teorema de Pitágoras para representar en la recta real los irracionales

cuadráticos. Así situábamos, por ejemplo, la raíz cuadrada de 10

mediante el uso de una recta graduada y un compás:

De igual forma representábamos las raíces cuadradas de 2, 13, 17,

etc.

Page 257: Números y formas - Aprender y divertirse con hoja de …hojamat.es/publicaciones/numform.pdf · Este es uno de los capítulos de las Matemáticas en los que se aprecia ... también

257

Cosa curiosa: en tantos años nadie me preguntó por la raíz de 7

¿Cómo se representa en la recta real? ¿Qué le

hubieras respondido tú?

Hay dos respuestas al menos: una es acumular

triángulos rectángulos a partir de uno con

hipotenusa la raíz de 2 adosándole un cateto de medida la unidad, con

lo que la hipotenusa equivaldría a la raíz de 3, y así sucesivamente,

mediante catetos 1 se irían generando todas la raíces.

Otra es acudir a una diferencia de cuadrados. En la imagen puedes ver

la representación de la raíz de 7 tomada como cateto de un triángulo

de hipotenusa 4 y el otro cateto 3:

Pero este método tiene un inconveniente, y es que sólo son

representables con diferencias de cuadrados los números impares y

los múltiplos de 4. Por tanto, el número 14 no se podría construir ni con

sumas de cuadrados ni con diferencias.

¿Sabrías indicar qué otras dos construcciones geométricas sobre un

triángulo rectángulo nos permitirían representar todos los irracionales

cuadráticos?

Así que hemos descubierto que la descomposición de un número en

sumas o bien en diferencias de cuadrados clasifica a los números

enteros positivos en cuatro clases.

Terminamos este estudio como lo comenzamos, con la sección 182 de

las Disquisitiones arithmeticae:

Todo número natural según Gauss se puede representar de la

siguiente forma:

sr c

s

ccb

r

bba qqqpppN ......2 211

21

2

21

Page 258: Números y formas - Aprender y divertirse con hoja de …hojamat.es/publicaciones/numform.pdf · Este es uno de los capítulos de las Matemáticas en los que se aprecia ... también

258

Donde pi son los factores del tipo 4h+3 y los qi del tipo 4h+1.

Con esa nomenclatura podemos afirmar:

Si a es par y todas la bi pares (contando el 0), N se puede

descomponer en suma de dos cuadrados y en diferencia de otros dos.

Igualando, N=a2+b2 = m2-n2 y produce de forma indirecta soluciones a

la ecuación x2+y2+z2=u2. Sería el caso del número 17 = 42+12= 92-82,

que da lugar a la identidad 42+12+82= 92

Si a es impar y todas la bi pares, N equivaldrá a sumas de cuadrados

pero no a diferencias. Ocurre esto con el número 10 = 32+12 que no

puede escribirse como diferencia de cuadrados a causa de no poder

expresarse como dos factores de la misma paridad.

Si a es par y alguna bi impar, admitirá una descomposición en

diferencias de cuadrados pero no en sumas (de dos). Así, 15=42-12 y

no se puede descomponer en suma por ser del tipo 4h+3.

Por último, no admitirán ninguna descomposición similar los que

presenten a impar y alguna bi impar. Es así el número 70 = 2*5*7, que

a causa del 2 y el 7 no admitirá ser expresado como suma o diferencia

de cuadrados. Insisto en la pregunta: ¿Cómo lo podríamos representar

en la recta real? Es una cuestión más bien elemental.

ESPIRAL DE NÚMEROS

A partir del número 3 se construye la siguiente sucesión de números

impares

3, 5, 13, 85, 157, 12325, 12461, 106285, 276341,…

¿Cómo se ha conseguido? Si consultas en la Red puedes descubrir

una definición algo complicada, que está contenida en una página muy

popular. Nosotros pedimos un procedimiento más simple mediante el

que se genere un 5 a partir del 3, y un 13 a partir del 5, y así el mismo

procedimiento en todos.

Page 259: Números y formas - Aprender y divertirse con hoja de …hojamat.es/publicaciones/numform.pdf · Este es uno de los capítulos de las Matemáticas en los que se aprecia ... también

259

Descubrimos la solución y, siguiendo nuestra afición a los giros, le daremos algunas vueltas. Para formar esta sucesión a partir del 3 se ha elegido en cada paso la “mínima hipotenusa que forma con el número una terna pitagórica”: 32+42=52; 52+122=132; 132+842=852… y así van saliendo 3, 5, 13, 85,… (a) La función mh(n) (“mínima hipotenusa para n”) está definida para todo número entero mayor que 2. En efecto, n2 ha de ser igual a una diferencia de cuadrados entre mh(n) y un cateto, llamémosles a y b: n2=a2-b2=(a+b)(a-b), siendo ambos factores de la misma paridad y diferentes entre sí. Esto siempre es posible: Si n es impar, n2 también lo será, y se podrá descomponer al menos como n2 =n2 *1, ambos impares. Si es par, n2 será múltiplo de 4, luego se puede escribir como n2 = 2*2m, ambos pares. Por tanto todo número mayor que 2 posee una o varias hipotenusas posibles y bastará elegir la mínima (¿por qué esto falla con el 1 y el 2?) (b) Una demostración fácil: Si n es primo, sólo existe una solución y viene dada por Mh(n)=(n2+1)/2. Además, en ese caso, la diferencia entre la hipotenusa y el otro cateto es la unidad. (c) Implementación de la función mh(n) Si toda solución pasa por una diferencia de cuadrados, deberemos encontrar dos factores de n con la misma paridad lo más cercanos uno de otro, a fin de que su semisuma (valor de a) sea mínima. Incluimos un posible código function minihip(n) dim i,a,b dim sigue as boolean a=n*n ‘ tomamos el cuadrado de n i=n ‘El primer factor probar es el mismo n sigue=true ‘Controla el bucle while while sigue i=i-1 ‘Vamos bajando el valor de i b=a/i ‘calculamos el otro factor

Page 260: Números y formas - Aprender y divertirse con hoja de …hojamat.es/publicaciones/numform.pdf · Este es uno de los capítulos de las Matemáticas en los que se aprecia ... también

260

if esmultiplo(a,i)=1 and esnumpar(i+b)=1 then sigue=false ‘Han de ser divisores de n2 y de la misma paridad wend b=(b+i)/2 ‘Se encuentra la semisuma de ambos factores minihip=b ‘y ese es el valor de mh(n) end function (d) Espiral de números Si reiteramos la aplicación de la función mh(n) a partir de un número entero, podremos construir un espiral con las hipotenusas (enteras) lo más pequeñas posible. De poco nos sirve, porque pronto comienzan a crecer. Por ejemplo, la que comienza con el 16 al principio va muy lenta, pero después salta: 16, 20, 25, 65, 97 y de pronto, 4705.

En cada tramo de la espiral el arcocoseno de n/mh(n) nos da una medida muy intuitiva de lo que aumenta el cateto al pasar a la hipotenusa, así como del giro que sufre ésta en cada paso. También podemos usar la razón mh(n)/n. Hay muchos números que comparten la misma razón, así mh(22)/22 = 122/22 = 61/11 y mh(121)/121 = 671/121 = 61/11, pero hay que tener cuidado, pues el carácter mínimo de mh(n) puede romper alguna proporcionalidad. (e) La función mh(n) no es inyectiva. De hecho, el número 925, es mínima hipotenusa de 43, 533, 740, 875, 888 y 924 (f) El que c sea la mínima hipotenusa para a, no significa que también lo sea para el otro cateto b. Hay veces que sí, como en el caso de a=52: su mínima hipotenusa es c=65, con lo que el otro cateto es b=39 y mh(39) = 65 de nuevo. En otros casos no se produce esa coincidencia: a=55, mh(a)=73, b=48 y mh(48)=50, que no coincide con 73. Piensa, ¿qué será más frecuente, el que coincidan o el que no? Pues en los mil primeros números son más frecuentes las coincidencias (entre 56% y 53,5%), pero va decreciendo ese porcentaje. Con más paciencia o instrumentos más rápidos podríamos conjeturar su límite.

Page 261: Números y formas - Aprender y divertirse con hoja de …hojamat.es/publicaciones/numform.pdf · Este es uno de los capítulos de las Matemáticas en los que se aprecia ... también

261

(g) Se señaló anteriormente que el arcocoseno es una buena medida de la razón n/mh(n). Su cota es pi/2. ¿Crees que podemos acercarnos a esa cota tanto como queramos eligiendo convenientemente n? La respuesta es afirmativa: Usando números primos la razón n/mh(n) = 2p/(p2+1) tendería a 0 para p suficientemente grande.

Page 262: Números y formas - Aprender y divertirse con hoja de …hojamat.es/publicaciones/numform.pdf · Este es uno de los capítulos de las Matemáticas en los que se aprecia ... también

262

NÚMEROS ESPECIALES

AUTONÚMEROS O NÚMEROS COLOMBIANOS

Estos números, llamados también de Devlali, fueron descritos por el

matemático indio Kaprekar (el de la constante 6174,

http://es.wikipedia.org/wiki/Constante_de_Kaprekar). Son muy

conocidos por haberlos presentado Martin Gadner en uno de sus

libros. En esta dirección puedes leer su artículo.

http://librosdemates.blogspot.com.es/2013/01/viajes-por-el-tiempo-y-

otras.html

Su definición es algo extraña, porque son aquellos números enteros

positivos que no pueden ser expresados como la suma de otro

entero con la suma de sus cifras (Kaprekar llamó a esta operación

digitadición). Por ejemplo, 20, no puede generarse con números más

pequeños a los que les sumamos sus cifras. Con los de una cifra se ve

que es imposible, y con los de dos: 11+1+1=13, 12+1+2=15,

13+1+3=17, 14+1+4=19, 15+1+5=21, y el resto tampoco daría como

resultado 20.

Con esta definición se comprende que existan infinitos tipos de

autonúmeros, dependiendo de la base elegida, y así se ha

demostrado. Nosotros nos limitaremos a la base 10. En este caso los

tienes en http://en.wikipedia.org/wiki/Self_number y son estos:

1, 3, 5, 7, 9, 20, 31, 42, 53, 64, 75, 86, 97, 108, 110, 121, 132, 143,

154, 165, 176, 187, 198, …

También los puedes estudiar en http://oeis.org/A003052

Estos números proceden de una criba. Puedes ir calculando todos los

resultados posibles si se suma cada número con la suma de sus

dígitos en la base dada. Resultarían estos:

Page 263: Números y formas - Aprender y divertirse con hoja de …hojamat.es/publicaciones/numform.pdf · Este es uno de los capítulos de las Matemáticas en los que se aprecia ... también

263

2, 4, 6, 8, 10, 11, 12, 13, 14, 15, 16, 17, 18, 19, 21, 22, 23, 24, 25, 26,

27, 28,…, que están contenidos en http://oeis.org/A176995 y los que

nos ocupan serían su complemento.

Algoritmo de fuerza bruta

Una cuestión interesante es cómo saber si un número es autonúmero o

no. El primer acercamiento a este tema puede consistir en recorrer

todos los números inferiores a N, añadirles la suma de sus cifras y

comprobar si el resultado es N. Desembocamos entonces en la

programación con hojas de cálculo. Necesitaríamos:

La función SUMACIFRAS

Un bucle que recorra los números k de 1 a N y que pare si el resultado

de k+sumacifras(k) es N.

Si la prueba anterior es positiva, declaramos que el número N no es

autonúmero, y si es negativa para todos los números inferiores a N,

diremos que sí lo es.

La función SUMACIFRAS debemos tenerla publicada en algún

documento. Por si no fuera así, la reproducimos en Basic de hoja de

cálculo:

Public Function sumacifras(n) 'No analiza si el número es entero positivo Dim h, i, s, m h = n ‘De la variable h se irán extrayendo las cifras s = 0 ‘Esta variable recogerá la suma de cifras While h > 9 ‘Bucle para extraer las cifras una a una i = Int(h / 10) m = h - i * 10 h = i s = s + m ‘La nueva cifra se suma a la variable Wend s = s + h ‘La cifra residual se suma a la variable sumacifras = s End Function

Page 264: Números y formas - Aprender y divertirse con hoja de …hojamat.es/publicaciones/numform.pdf · Este es uno de los capítulos de las Matemáticas en los que se aprecia ... también

264

Una vez tenemos la función sumacifras podemos organizar el test en

forma de función booleana:

Public Function esauto(n) Dim es As Boolean Dim k es = True ‘Se declara de entrada que el número es “colombiano” k = 0 ‘Comenzamos a recorrer los números menores que n While k < n And es ‘No paramos hasta llegar a n o hasta que es sea falso If k + sumacifras(k) = n Then es = False ‘Si hay igualdad, paramos y declaramos “False” k = k + 1 Wend esauto = es ‘La función recoge el valor de es End Function

Aplicamos esta función a los primeros números y obtendremos el valor

VERDADERO en los autonúmeros:

Por ejemplo, busquemos el primer autonúmero que sigue a 1000:

1 VERDADERO

2 FALSO

3 VERDADERO

4 FALSO

5 VERDADERO

6 FALSO

7 VERDADERO

8 FALSO

9 VERDADERO

10 FALSO

11 FALSO

12 FALSO

13 FALSO

14 FALSO

15 FALSO

16 FALSO

17 FALSO

18 FALSO

19 FALSO

20 VERDADERO

21 FALSO

22 FALSO

Page 265: Números y formas - Aprender y divertirse con hoja de …hojamat.es/publicaciones/numform.pdf · Este es uno de los capítulos de las Matemáticas en los que se aprecia ... también

265

Resulta ser el 1006.

Test de Kaprekar

El anterior algoritmo recorre demasiados números y es ineficiente para

enteros grandes. El mismo Kaprekar demostró un test en el que sólo

hay que analizar enteros no superiores al número de dígitos. Lo

copiamos de la página

http://www.numbersaplenty.com/set/self_number/

Lo hemos implementado como función para Excel. No detallamos el

código. Sólo lo copiamos con algún comentario:

Public Function esauto2(n) Dim nc, a, r, h, k Dim es As Boolean 'número de cifras a = 1: nc = 0 While a <= n a = a * 10: nc = nc + 1 Wend 'Se preparan las variables del test de Kaprekar

1000 FALSO

1001 FALSO

1002 FALSO

1003 FALSO

1004 FALSO

1005 FALSO

1006 VERDADERO

1007 FALSO

1008 FALSO

1009 FALSO

1010 FALSO

Page 266: Números y formas - Aprender y divertirse con hoja de …hojamat.es/publicaciones/numform.pdf · Este es uno de los capítulos de las Matemáticas en los que se aprecia ... también

266

If nc = 0 Then esauto2 = False: Exit Function r = 1 + ((n - 1) Mod 9) If r / 2 = r \ 2 Then h = r / 2 Else h = (r + 9) / 2 'Bucle del test es = True k = 0 While k <= nc And es If sumacifras(Abs(n - h - 9 * k)) = h + 9 * k Then es = False k = k + 1 Wend esauto2 = es End Function Hemos preparado un esquema en el que se puede elegir el inicio y compara las dos funciones, ESAUTO y ESAUTO2. Por si se hubiera deslizado algún error se han efectuado varias comprobaciones y coinciden ambas funciones, pero con gran lentitud en la primera.

Distribución de los autonúmeros

Es fácil ver que el incremento entre dos autonúmeros consecutivos es

frecuentemente 11. Por ello esperamos tramos lineales en su

distribución. Los veremos creando un gráfico con los primeros, pero si

elegimos un gráfico con muchos puntos, se nos presenta una

distribución prácticamente lineal, con pendiente 10,2, cercana al 11,

que como hemos visto, aparece en muchos tramos.

Page 267: Números y formas - Aprender y divertirse con hoja de …hojamat.es/publicaciones/numform.pdf · Este es uno de los capítulos de las Matemáticas en los que se aprecia ... también

267

Para ver mejor los tramos lineales elegimos un rango de números más

pequeño:

Normalmente el salto de 11 se produce porque equivale a rebajar en

una unidad las decenas cuando es posible. Así la suma total se rebaja

en 11. Si el primero es autonúmero puede obligar a que el segundo

también lo sea, pues si éste admitiera una suma, al rebajar esa unidad

podría no ser autonúmero el primero. Esto vale sólo para la mayoría de

los casos. No es una regla general. Ocurre algo parecido con 101,

1001, 10001, …

Hay una forma sencilla de engendrar autonúmeros (no todos), y es

comenzar por 9 y después usar la fórmula recurrente

Ck=8*10k-1+Ck-1+8

El siguiente número sería 8*10+9+8=97, el siguiente 800+97+8=905.

Con hoja de cálculo es fácil construir esta generación recurrente.

Inténtalo si quieres. En la imagen hemos añadido a la derecha la

función esauto.

Page 268: Números y formas - Aprender y divertirse con hoja de …hojamat.es/publicaciones/numform.pdf · Este es uno de los capítulos de las Matemáticas en los que se aprecia ... también

268

En párrafos anteriores descubrimos que todos los números se pueden

clasificar en generados por la digitadición de Kaprekar y autonúmeros,

que no pueden ser generados. Nos dedicaremos en primer lugar a los

generados, y con ellos descubriremos algo más sobre los

autonúmeros.

Números generados

Señalamos que los números 2, 4, 6, 8, 10, 11, 12, 13, 14, 15, 16, 17,

18, 19, 21, 22, 23, 24, 25, 26, 27, 28,… (http://oeis.org/A176995) son

los complementarios de los autonúmeros. Todos ellos se pueden

expresar como N+SUMACIFRAS(N), por lo que Kaprekar les llamó

“generados”. A este número N le podemos llamar generador del

mismo, pero desafortunadamente no es único, por lo que no podemos

considerarlo una función dependiente del número. En efecto, existen

números, como el 103, que poseen más de un desarrollo de este tipo,

ya que 103=92+9+2 y también 103=101+1+1.

Podíamos definir una función GENERADOR si para cada N

eligiéramos el mayor K que cumple que K+SUMACIFRAS(K)=N, y

asignar el valor 0 como generador de los autonúmeros. Así sí sería una

función. La función AUTO descrita más arriba y debidamente adaptada

nos servirá para este propósito:

Public Function generador(n) Dim k, g g = 0 While k < n If k + sumacifras(k) = n Then g = k k = k + 1 Wend

9 VERDADERO

10 97 VERDADERO

100 905 VERDADERO

1000 8913 VERDADERO

10000 88921 VERDADERO

100000 888929 VERDADERO

1000000 8888937 VERDADERO

10000000 88888945 VERDADERO

Page 269: Números y formas - Aprender y divertirse con hoja de …hojamat.es/publicaciones/numform.pdf · Este es uno de los capítulos de las Matemáticas en los que se aprecia ... también

269

generador = g End Function

En esta tabla puedes comprobar que si a cada número de la derecha le

sumas la suma de sus cifras, resulta el de la izquierda, salvo el caso

del autonúmero 97 al que le hemos asignado un cero.

Si construimos un diagrama de dispersión entre N y su generador,

obtenemos un resultado similar a este:

Abajo los puntos de valor 0 representan a los autonúmeros, y los de

arriba parecen presentar pautas lineales escalonadas, pero es una

ilusión, ya que esos tramos no se podrían solapar. Lo que ocurre es

que en este proceso los números consecutivos aparecen muy

cercanos, y dan la impresión de sucederse. Analizaremos estos

números con más detalle.

Si representamos un número N de k+1 cifras en base 10, lo estamos

generando mediante un polinomio del tipo

𝑁 = 𝑎0 + 𝑎1101 + 𝑎2102 + ⋯ + 𝑎𝑘10𝑘

N Generador(N)

90 81

91 77

92 82

93 78

94 83

95 79

96 84

97 0

98 85

99 90

100 86

101 100

Page 270: Números y formas - Aprender y divertirse con hoja de …hojamat.es/publicaciones/numform.pdf · Este es uno de los capítulos de las Matemáticas en los que se aprecia ... también

270

Si ahora le sumamos sus cifras (digitadición) se convertirá en

𝑁 = 2𝑎0 + 11𝑎1 + 101𝑎2 + 1001𝑎3 … + 100 … 01𝑎𝑘

Todos los números generados se podrán expresar de esta forma, y los

autogenerados, no.

Según esta fórmula, los dobles de las cifras 1…9 serán todos números

generados, y si dos generadores se diferencian sólo en una unidad en

las decenas, sus generados se diferenciarán en 11, como ya hemos

observado. Si se diferencian en una unidad de las centenas, sus

generados se diferenciarán en 101, y así. Estas correspondencias

también se dan en casi todos los autonúmeros, pues van la par de

estos. No se da en todos.

En este gráfico hemos descompuesto cada autonúmero en relación

con los menores de 100, y vemos una repetición de tramos lineales a

unas alturas de 1001, 2002, 3003 y 4004. Esto es consecuencia de la

fórmula que hemos desarrollado.

Fundamento del algoritmo de Kaprekar

La fórmula

𝑁 = 2𝑎0 + 11𝑎1 + 101𝑎2 + 1001𝑎3 … + 100 … 01𝑎𝑘

nos da una forma paralela a la de Kaprekar para decidir si N es

autonúmero en pocos pasos (es el mismo proceso con otra

Page 271: Números y formas - Aprender y divertirse con hoja de …hojamat.es/publicaciones/numform.pdf · Este es uno de los capítulos de las Matemáticas en los que se aprecia ... también

271

orientación). Observa que todos los coeficientes son congruentes con 2

módulo 9, con lo que si llamamos G al posible generador de N y SC(G)

a la suma de sus cifras, se cumplirá que N≡2SC(G) (mod 9. Para saber

esto no hacía falta la fórmula, pues como G es congruente con SC(G)

módulo 9 y se cumple que N=G+SC(G), es evidente que N será

congruente con el doble de SC(G).

Como 2 es primo con 9, en la ecuación N≡2SC(G) (mod 9 se podrá

encontrar una solución S, con lo que G=9*k+S para valores de k no

superiores al número de cifras. Lo vemos con el 30: Si N=30, será

30≡3≡2*SC(G). En este caso SC(G)≡6, porque 6+6≡3 (mod 9. De esta

forma G puede ser 6, 15 0 24. Recorremos de mayor a menor y

descubrimos que el 24 vale, porque 24+2+4=30, luego 30 no es

autonúmero, sino generado.

Probamos con un número mayor, como 4327, del que ya sabemos por

otros medios que es autonúmero:

Hallamos el resto módulo 9 de 4327 (puedes dividir mentalmente o

usar la función RESIDUO de Excel) y resulta ser 7. Planteamos

7≡2SC(G) (mod 9. También, mentalmente, vemos que SC(G) ≡8 (mod

9. Por tanto, vamos restando de 4327 números del tipo 9*k+8 hasta ver

si la suma de las cifras de la diferencia es ese número:

K=0: 4327-8=4321, y su suma de cifras no es 8.

K=1: 4327-17=4310 y no suman 17

K=2: 4327-26=4301. No suman 26,

K=3: 4327-35=4292. No vale

Acabamos con k=4, porque la suma de cifras ya no puede ser mayor:

K=4: 4327-44=4283, de suma 17, luego tampoco es válido.

Hemos comprobado, con la misma técnica de Kaprekar y distinta

presentación, que 4327 es autonúmero. No puede ser generado.

Para los aficionados a la programación, esta es la función que

podemos usar:

Public Function esauto3(n)

Dim nc, a, r, h, k

Page 272: Números y formas - Aprender y divertirse con hoja de …hojamat.es/publicaciones/numform.pdf · Este es uno de los capítulos de las Matemáticas en los que se aprecia ... también

272

Dim es As Boolean

'número de cifras

a = 1: nc = 0

While a <= n

a = a * 10: nc = nc + 1

Wend

'módulo 9

r = n Mod 9

For k = 0 To 8: If (r - 2 * k) Mod 9 = 0 Then h = k

k = 0: es = True

While k <= nc And es

If sumacifras(n - 9 * k - h) = 9*k+h Then es = False

k = k + 1

Wend

esauto3 = es

End Function

Hemos construido un esquema de hoja de cálculo en el que vemos la

equivalencia de las tres funciones que hemos. En la imagen tienes un

ejemplo:

N ESAUTO(N) ESAUTO2(N) ESAUTO3(N)

1928 FALSO FALSO FALSO

1929 FALSO FALSO FALSO

1930 VERDADERO VERDADERO VERDADERO

1931 FALSO FALSO FALSO

1932 FALSO FALSO FALSO

1933 FALSO FALSO FALSO

1934 FALSO FALSO FALSO

1935 FALSO FALSO FALSO

1936 FALSO FALSO FALSO

1937 FALSO FALSO FALSO

1938 FALSO FALSO FALSO

1939 FALSO FALSO FALSO

1940 FALSO FALSO FALSO

1941 VERDADERO VERDADERO VERDADERO

1942 FALSO FALSO FALSO

1943 FALSO FALSO FALSO

Page 273: Números y formas - Aprender y divertirse con hoja de …hojamat.es/publicaciones/numform.pdf · Este es uno de los capítulos de las Matemáticas en los que se aprecia ... también

273

Sucesiones recurrentes de números generados

Kaprekar estudió también las sucesiones que se forman si a un

número cualquiera le vamos aplicando la digitadición tanto a él como a

los resultados sucesivos. Por ejemplo, si comenzamos con el 12

tendríamos la sucesión 12, 15, 21, 24, 30,…Evidentemente es infinita,

creciente y todos sus elementos, salvo quizás el primero, son

generados. Kaprekar destacó que si restamos el último del primero y

sumamos las cifras del último, resulta la suma de cifras de todos ellos.

Por ejemplo, aquí, 30-12+3=21=1+2+1+5+2+1+2+4+3+0

Aunque el autor le dio mucha importancia, basta recorrer la generación

para darse cuenta de su validez: 12+3+6+3+6=30, luego al restar

resultarán las sumas de cifras menos la del último. Es una obviedad.

Page 274: Números y formas - Aprender y divertirse con hoja de …hojamat.es/publicaciones/numform.pdf · Este es uno de los capítulos de las Matemáticas en los que se aprecia ... también

274

SOLUCIONES

CUADRADOS

Sumas de los primeros cuadrados

Cuadrados que dan cuadrado

12+22+32+42+52+…+232+242 = 702

Cuadrados que dan triangular

12+22+32+42+52 = 55 = T10

12+22+32+42+52+62 = 91 = T13

12+22+32+42+52+…+842+852 = 208335 = T645

(http://oeis.org/A039596)

Triangulares que dan cuadrados

1+3 = 22

1+3+6+10+…1176 = 19600 =1402

Triangulares que dan triangular

1+3+6 = 10 = T4

1+3+6+10+15+21+28+36 = 120 = T15

1+3+6+10+… + T20 = 1540 = T55

1+3+6+10+… + T34 = 7140 = T119

(http://oeis.org/A027568)

Cuadrados en progresión aritmética

Page 275: Números y formas - Aprender y divertirse con hoja de …hojamat.es/publicaciones/numform.pdf · Este es uno de los capítulos de las Matemáticas en los que se aprecia ... también

275

La condición que ha de cumplir la media aritmética de los dos

cuadrados, es la de ser ella también un cuadrado perfecto, y en hoja

de cálculo se puede expresar mediante esta condición:

N=(ENTERO(RAÍZ(N))^2

El número 10404= 102^2 forma progresión aritmética con 1764= 42^2;

y la media aritmética de ambos 6084 = 78^2

Las diferencias entre tres cuadrados en progresión aritmética son

siempre múltiplos de 24. Lo razonaremos en dos fases:

(A) Las diferencias son múltiplos de 3

Partimos de a2+b2=2c2 como condición de estar en progresión

aritmética. Los restos cuadráticos módulo 3 son 0 y 1. Nunca pueden

ser 2.

Si el resto de c2 es 0, el de 2c2 también lo es, luego a2 y b2 deberán

presentar también resto 0 (es la única posibilidad). Por tanto sus

diferencias serán múltiplo de 3.

Si el resto de c2 es 1, el de 2c2 será 2 y eso obliga a que a2 y b2 tengan

también resto 1, luego las diferencias vuelven a ser múltiplos de 3. Con

esto queda demostrado.

(A) Las diferencias son múltiplos de 8

Los restos cuadráticos módulo 16 son 0, 1, 4 y 9. Podemos construir

una tabla de sumar entre ellos para que comprendas mejor el

razonamiento:

+ 0 1 4 9

0 0 1 4 9

1 1 2 5 10

4 4 5 8 13

9 9 10 13 2

Con razonamientos similares al caso de 3, tendremos:

Page 276: Números y formas - Aprender y divertirse con hoja de …hojamat.es/publicaciones/numform.pdf · Este es uno de los capítulos de las Matemáticas en los que se aprecia ... también

276

Si el resto de c2 es 0, el de 2c2 también lo es, luego a2 y b2 deberán

presentar también resto 0. Por tanto sus diferencias serán múltiplos de

16 y con mayor razón de 8.

Si el resto de c2 es 1, el de 2c2 es 2, luego a2 y b2 deberán presentar

ambos resto 1 o resto 9. Por tanto su diferencia será al menos múltiplo

de 8 si no lo es de 16 (en el caso 9-1)

Si el resto de c2 es 4, el de 2c2 es 8, luego a2 y b2 deberán presentar

también resto 4. Por tanto sus diferencias serán múltiplos de 16.

Si el resto de c2 es 9, el de 2c2 será 2, y ya hemos estudiado ese caso.

Acabamos de razonar que las diferencias son múltiplos de 16 salvo en

un caso en el que sólo lo son de 8.

Resumiendo: Las diferencias mutuas son múltiplos de 24.

La mitad, cuadrado, el tercio, cubo

Si N = 2m2 = 3n3, deberán figurar, tanto en m como en n, los factores 2

y 3. Luego m2 = 36*k2 y n3 = 216*h3.

N = 72*k2 = 648*h3 y h ha de ser cuadrado perfecto para que se pueda

dividir entre k2

Queda, pues, que N=648p6

Así que las soluciones se obtendrán de los valores de p2

p2=1, 648, p2=4, 41472, p2=9, 472392, p2=16, 2654208

En el caso N= 2n2 = 5m5

deberá ser N = 200*k2 = 500000*h5

y una solución 500000

En el caso N= 3n3 = 5m5

será 10125*k3 = 500000*h5

y una solución posible es 922640625

Page 277: Números y formas - Aprender y divertirse con hoja de …hojamat.es/publicaciones/numform.pdf · Este es uno de los capítulos de las Matemáticas en los que se aprecia ... también

277

Exploración matemática

(1) Si 2n2 + 1 = m2, ambos naturales, m ha de ser impar, luego m=2k+1

con k natural, con lo que se cumple si 2n2 + 1 = 4k2+4k+1.

Simplificando y dividiendo entre 8: n2/4 = k(k+1)/2, luego se cumple que

n2/4 es cuadrado y triangular.

(2) Partimos de la igualdad x2=k(k+1)/2. Multiplicamos todo por 8 y

queda: 8x2 =4k2+4k+1-1=(2k+1)2-1. Si llamamos y a 2k+1, nos queda

la ecuación de Pell 8x2+1=y2

(3) (Para no complicar la edición, usaremos la palabra RAÍZ para

representar la raíz cuadrada). Como (3+RAÍZ(8))n = yn+RAÍZ(8)xn,

basta ir multiplicando por sí mismo y sumando por separado los

números naturales y los coeficientes de RAÍZ(8): (3+RAÍZ(8)),

(17+6*RAÍZ(8)), (99+35*RAÍZ(8))… van resultando los valores de x 1,

6, 35,…

(4) Basta usar (yn-1+RAÍZ(8)xn-1)* (3+RAÍZ(8))= yn+RAÍZ(8)xn con lo que

resultan las fórmulas pedidas. Para reducir a una sola con x, se puede

acudir a esta transformación:

yn=3xn-1+8yn-1, xn=3xn-1+yn-1

xn=3xn-1+yn-1 = 9xn-2+3yn-2+(8xn-2+3yn-2) = 17 xn-2 + 6 yn-2 = 6(3xn-2 + yn-2)-

xn-2 = 6 xn-1 - xn-2

Jugamos con los triangulares

(1) La tabla a construir puede ser similar a la siguiente:

N T(N) Suma cuad. X

1 1

2 3 10 4

3 6 45 9

4 10 136 16

5 15 325 25

6 21 666

Page 278: Números y formas - Aprender y divertirse con hoja de …hojamat.es/publicaciones/numform.pdf · Este es uno de los capítulos de las Matemáticas en los que se aprecia ... también

278

7 28 1225

En la primera columna figura el número de orden, en la segunda el

triangular correspondiente, y a continuación la suma de cuadrados

entre dos consecutivos. Fácilmente se descubre que los resultados son

números triangulares de índice X, que resulta ser el cuadrado del

índice. Por tanto conjeturamos que la suma del triangular de índice n

con el siguiente de índice n+1 da como resultado al triangular

correspondiente a (n+1)2

(2) Si recordamos que T(n)=n(n+1)/2, será T(n+1)=(n+1)(n+2)/2.

Desarrollamos sus cuadrados y sumamos, resultando la expresión

(n+1)2(n2+(n+2)2)/4 =(n+1)2(2 n2+4n+4)/4 =(n+1)2((n+1)2+1)/2 , que

corresponde al triangular de índice (n+1)2.

Con la calculadora Wiris se comprueba fácilmente:

(3) No necesita comentarios

Cubos y gnomones

(1) Todo cubo n3 de base natural n equivale a la diferencia de los

cuadrados de los números triangulares Tn y Tn-1. Basta desarrollar la

expresión (n(n+1)/2)2-(n(n-1)/2)2 y comprobar que el resultado es n3.

(2) la suma de cubos propuesta equivale a T2k+r

- T2

k-1. Puedes

desarrollarla de varias formas simplificando algebraicamente.

El número 30500

(1) Se presentan todas las cifras, porque las terminaciones de

cuadrados de primos mayores que 2 sólo pueden ser 1 y 9 (ó -1).

Page 279: Números y formas - Aprender y divertirse con hoja de …hojamat.es/publicaciones/numform.pdf · Este es uno de los capítulos de las Matemáticas en los que se aprecia ... también

279

Según la distribución del 1 y el 9 en las cuatro terminaciones

tendremos: 1+1+1+1 termina en cuatro; 1+1+1+9 en 2; 1+1+9+9 en 0;

1+9+9+9 en 8 y 9+9+9+9 en 6, luego los números considerados

recorrerán las cifras 2, 4, 6, 8 y 0

(2) Los cuadrados de los números primos salvo el 2 tienen la forma

4k+1, luego al sumar cuatro resultará 4m+4, que es múltiplo de 4

(3) 23500=71^2+73^2+79^2+83^2

(5)

sub sumaprimcuad

dim n,a,b,c,d,e,r,vale

dim s$

s$=inputbox("Número")

n=val(s$)

a=2:b=3:c=5:d=7

r=sqr(n/4)

vale=0

while a<=r and vale=0

e=a^2+b^2+c^2+d^2

if e=n then msgbox(str$(a)+str$(b)+str$(c)+str$(d)):vale=1

a=b:b=c:c=d:d=primprox(d)

wend

if vale=0 then msgbox("No es suma de cuadrados consecutivos")

end sub

(6) Por las mismas razones que en (1), a partir de un resultado todos

tendrán una de las formas: 1+1+1 que termina en 3, 1+1+9 en 1,

1+9+9 en 9 y 9+9+9 en 7, y nunca se produce la terminación en 5

En el caso de dos cuadrados tendríamos: 1+1 terina en 2, 1+9 en 0 y

9+9 en 8. No aparecen ni el 6 ni el 4.

P I TÁG ORAS Y SUS TE RNA S

Page 280: Números y formas - Aprender y divertirse con hoja de …hojamat.es/publicaciones/numform.pdf · Este es uno de los capítulos de las Matemáticas en los que se aprecia ... también

280

El sueño de Lewis Carroll

30240, 31920, 41160 y 43680

Las ternas pedidas son:

30240 90, 672 y 678 144, 420 y 444 240,252 y 348

31920 80, 798 y 802 105, 608 y 617 190, 336 y 386

41160 105, 784 y 791 168, 490 y 518 280, 294 y 406

43680 112, 780 y 788 210, 416 y 466 240, 364 y 436

Se pueden obtener con OpenOffice.org Calc con este código:

Código de la macrobúqueda Sub busquedas Dim fila,i,j,k,m,l,n dim a,b,c dim vale as boolean fila=10 ‘Lleva a cuenta de las filas de presentación de resultados for i=1 to 50000 Rango de búsqueda de áreas n=2*i Doble del área o producto de catetos c=5 Lleva la cuenta de la columna de presentación l=0 Cuenta las ternas válidas a=int(sqr(n)) for j=1 to a Busca catetos if esdivisible(n,j) then b=n/j Calcula el otro cateto m=j*j+b*b Busca la terna pitagórica if escuadrado(m) then Se trata de una terna pitagórica l=l+1 c=c+1 Se presenta el resultado

Page 281: Números y formas - Aprender y divertirse con hoja de …hojamat.es/publicaciones/numform.pdf · Este es uno de los capítulos de las Matemáticas en los que se aprecia ... también

281

StarDesktop.CurrentComponent.sheets(0).GetCellByPosition(4,fila).value=iStarDesktop.CurrentComponent.sheets(0).GetCellByPosition(c,fila).value=j c=c+1 StarDesktop.CurrentComponent.sheets(0).GetCellByPosition(c,fila).value=b c=c+1 StarDesktop.CurrentComponent.sheets(0).GetCellByPosition(c,fila).value=sqr(m) c=c+1 end if end if next j if l>2 then fila=fila+1 Si se han conseguido tres ternas, se avanza una fila next i End Sub Oblongos y pitagóricos

Solución

Cuando se buscan las soluciones de la ecuación de Pell las recurrencias vienen dadas por las fórmulas de recurrencia zn+1=zn*z0+D*yn*y0 yn+1=zn*y0+yn*z0, pero en el caso z2-2y2 = -1 las soluciones surgen de forma alternada. Así, como en este caso z0=1, y0=1, tendremos: zn+1=zn+2yn; yn+1=zn+yn y reiterando dos veces

Z’’=Z’+2Y’=(Z+2Y)+2(Z+Y) = Z+2Y+2Z+2Y = 3Z+4Y

Y’’=Z’+Y’ = Z+2Y+Z+Y = 2Z+3Y

Teniendo en cuenta que Z=2X+1, y que Y=X+1, nos resulta

Y’’=2Z+3Y=2(2X+1)+3Y = 4X+3Y+2, que es la segunda fórmula de Fermat

Page 282: Números y formas - Aprender y divertirse con hoja de …hojamat.es/publicaciones/numform.pdf · Este es uno de los capítulos de las Matemáticas en los que se aprecia ... también

282

De Z’’=3Z+4Y podemos obtener (2X’’+1)=3(2X+1)+4Y; 2X’’ = 6X+4Y+2;

X’’ = 3X+2Y+1, que es la primera

Aplicamos estas dos fórmulas al cateto menor y a la hipotenusa y obtenemos los mismos resultados a partir de 3, 4 y 5

X''=3X+2Y+1 X+1 Y''=4X+3Y+2

3 4 5

20 21 29

119 120 169

696 697 985

4059 4060 5741

23660 23661 33461

137903 137904 195025

803760 803761 1136689

4684659 4684660 6625109

27304196 27304197 38613965

159140519 159140520 225058681

Doblado pitagórico

Los catetos pueden ser 2uv y u2-v2 con u, v primos entre sí y de distinta

paridad. Por tanto basta plantear 2uv+u2-v2=k y transformarla en

(u+v)2-2v2 = k, que se puede expresar como x2-2y2=k o como 2x2-y2=-k

con lo que obliga a que 2 sea resto cuadrático respecto a los divisores

de k y llegamos a la misma conclusión que en la anterior cuestión.

Espiral de números

El siguiente es 339709. La sucesión se ha formado eligiendo para cada

número la menor hipotenusa que forma con él una terna pitagórica:

32+42 = 52; 52+122 = 132; 132+842=852; 852+1322=1572, etc.

5, 13, 85, 157, 12325, 12461, 106285, 276341, 339709